Sei sulla pagina 1di 160

!

!
!
!
!

B47AA

Introduction to the
Process Industries A

Heriot-Watt University

Edinburgh EH14 4AS, United Kingdom


!
!

Produced by Heriot-Watt University, 2013

Copyright © 2013 Heriot-Watt University

All rights reserved. No part of this publication may be reproduced, stored in a


retrieval system or transmitted in any form or by any means without express
permission from the publisher.

This material is prepared to support the degree programmes in Chemical and


Petroleum Engineering.

Distributed by Heriot-Watt University


Introduction to the Process Industries A
!

Acknowledgements
Thanks are due to the members of Heriot-Watt, School of Engineering and Physical Science
who planned and generated this material. Thanks are also due to the members of Heriot-
Watt’s SCHOLAR team in preparing the printed materials.

We would like to acknowledge the assistance and contributions from colleagues across the
University and students in preparing this and support material.!
i

Contents

1 Units Conversion and Dimensional Analysis 1


1.1 Introduction . . . . . . . . . . . . . . . . . . . . . . . . . . . . . . . . . . 3
1.2 Fundamental & Derived SI Units . . . . . . . . . . . . . . . . . . . . . . 4
1.3 Unit Family . . . . . . . . . . . . . . . . . . . . . . . . . . . . . . . . . . 6
1.4 SI Units for Chemical Engineers . . . . . . . . . . . . . . . . . . . . . . 7
1.5 Conversion Factors . . . . . . . . . . . . . . . . . . . . . . . . . . . . . . 9
1.6 Temperature Scales . . . . . . . . . . . . . . . . . . . . . . . . . . . . . 13
1.7 Density and Specific Gravity Units . . . . . . . . . . . . . . . . . . . . . 14
1.8 Pressure Units . . . . . . . . . . . . . . . . . . . . . . . . . . . . . . . . 15
1.9 Energy Units . . . . . . . . . . . . . . . . . . . . . . . . . . . . . . . . . 19
1.10 Dimensional Consistency . . . . . . . . . . . . . . . . . . . . . . . . . . 20
1.11 Dimensionless Quantities & Groups . . . . . . . . . . . . . . . . . . . . 22
1.12 Tutorial Topic 1 . . . . . . . . . . . . . . . . . . . . . . . . . . . . . . . . 27
1.13 Units and conversion tables . . . . . . . . . . . . . . . . . . . . . . . . . 29

2 Engineering Chemistry 31
2.1 Introduction . . . . . . . . . . . . . . . . . . . . . . . . . . . . . . . . . . 33
2.2 Calculations in Chemistry . . . . . . . . . . . . . . . . . . . . . . . . . . 38
2.3 The Basis . . . . . . . . . . . . . . . . . . . . . . . . . . . . . . . . . . . 40
2.4 Limiting and Excess Reactants . . . . . . . . . . . . . . . . . . . . . . . 41
2.5 Conversion, Yield and Selectivity . . . . . . . . . . . . . . . . . . . . . . 43
2.6 Mole & Mass Fractions . . . . . . . . . . . . . . . . . . . . . . . . . . . . 46
2.7 Average Molar Mass of Mixture . . . . . . . . . . . . . . . . . . . . . . . 48
2.8 Other Concentration Units . . . . . . . . . . . . . . . . . . . . . . . . . . 49
2.9 Changing Concentration Units . . . . . . . . . . . . . . . . . . . . . . . 50
2.10 Reactions in Parallel . . . . . . . . . . . . . . . . . . . . . . . . . . . . . 51
2.11 Tutorial Topic 2 . . . . . . . . . . . . . . . . . . . . . . . . . . . . . . . . 54
2.12 Bibliography . . . . . . . . . . . . . . . . . . . . . . . . . . . . . . . . . . 55

3 Physical Properties and Gas Laws 57


3.1 Introduction . . . . . . . . . . . . . . . . . . . . . . . . . . . . . . . . . . 59
3.2 Density, Specific Volume and Specific Gravity . . . . . . . . . . . . . . . 60
3.3 Specific Heat Capacity . . . . . . . . . . . . . . . . . . . . . . . . . . . . 61
3.4 Mixture Specific Heat Capacity . . . . . . . . . . . . . . . . . . . . . . . 63
3.5 Sensible and Latent Heat Effects . . . . . . . . . . . . . . . . . . . . . . 65
3.6 Phase Diagrams and Gibbs Phase Rule . . . . . . . . . . . . . . . . . . 67
3.7 Equations of State . . . . . . . . . . . . . . . . . . . . . . . . . . . . . . 70
3.8 Partial Pressure . . . . . . . . . . . . . . . . . . . . . . . . . . . . . . . 73
3.9 Vapour Pressure and Antoine Equation . . . . . . . . . . . . . . . . . . 76
ii CONTENTS

3.10 Henry’s Law and Raoult’s Law . . . . . . . . . . . . . . . . . . . . . . . 79


3.11 Tutorial Topic 3 . . . . . . . . . . . . . . . . . . . . . . . . . . . . . . . . 82
3.12 Bibliography . . . . . . . . . . . . . . . . . . . . . . . . . . . . . . . . . . 84

4 Material Balance on Single-Stage Systems 85


4.1 Introduction . . . . . . . . . . . . . . . . . . . . . . . . . . . . . . . . . . 87
4.2 Basic Principles . . . . . . . . . . . . . . . . . . . . . . . . . . . . . . . . 88
4.3 General Balance Equation . . . . . . . . . . . . . . . . . . . . . . . . . . 90
4.4 Non-Reacting Systems . . . . . . . . . . . . . . . . . . . . . . . . . . . 93
4.5 Combustion Reaction Material Balances . . . . . . . . . . . . . . . . . . 105
4.6 Extent of Reaction . . . . . . . . . . . . . . . . . . . . . . . . . . . . . . 118
4.7 Tutorial Topic 4 . . . . . . . . . . . . . . . . . . . . . . . . . . . . . . . . 121
4.8 Bibliography . . . . . . . . . . . . . . . . . . . . . . . . . . . . . . . . . . 123

5 Material Balance on Multi-Stage Systems 125


5.1 Introduction . . . . . . . . . . . . . . . . . . . . . . . . . . . . . . . . . . 126
5.2 Multiple Units with no Recycle . . . . . . . . . . . . . . . . . . . . . . . . 127
5.3 Recycle without Chemical Reaction . . . . . . . . . . . . . . . . . . . . 129
5.4 Recycle with Chemical Reaction . . . . . . . . . . . . . . . . . . . . . . 130
5.5 Material Balance Strategies . . . . . . . . . . . . . . . . . . . . . . . . . 131
5.6 Tutorial Topic 5 . . . . . . . . . . . . . . . . . . . . . . . . . . . . . . . . 143
5.7 Bibliography . . . . . . . . . . . . . . . . . . . . . . . . . . . . . . . . . . 145

Glossary 146

©H ERIOT-WATT U NIVERSITY
1

Topic 1

Units Conversion and Dimensional


Analysis

Contents
1.1 Introduction . . . . . . . . . . . . . . . . . . . . . . . . . . . . . . . . . . . . . . 3
1.2 Fundamental & Derived SI Units . . . . . . . . . . . . . . . . . . . . . . . . . . 4
1.3 Unit Family . . . . . . . . . . . . . . . . . . . . . . . . . . . . . . . . . . . . . . 6
1.4 SI Units for Chemical Engineers . . . . . . . . . . . . . . . . . . . . . . . . . . 7
1.5 Conversion Factors . . . . . . . . . . . . . . . . . . . . . . . . . . . . . . . . . . 9
1.5.1 Coherent and Non-Coherent Systems . . . . . . . . . . . . . . . . . . . 11
1.5.2 Chemical Engineering Scale . . . . . . . . . . . . . . . . . . . . . . . . 12
1.6 Temperature Scales . . . . . . . . . . . . . . . . . . . . . . . . . . . . . . . . . 13
1.7 Density and Specific Gravity Units . . . . . . . . . . . . . . . . . . . . . . . . . 14
1.8 Pressure Units . . . . . . . . . . . . . . . . . . . . . . . . . . . . . . . . . . . . 15
1.8.1 Manometric Pressure . . . . . . . . . . . . . . . . . . . . . . . . . . . . 17
1.9 Energy Units . . . . . . . . . . . . . . . . . . . . . . . . . . . . . . . . . . . . . 19
1.10 Dimensional Consistency . . . . . . . . . . . . . . . . . . . . . . . . . . . . . . 20
1.11 Dimensionless Quantities & Groups . . . . . . . . . . . . . . . . . . . . . . . . 22
1.12 Tutorial Topic 1 . . . . . . . . . . . . . . . . . . . . . . . . . . . . . . . . . . . . 27
1.13 Units and conversion tables . . . . . . . . . . . . . . . . . . . . . . . . . . . . . 29

Prerequisite knowledge

• Basic knowledge of dimensions.

• Knowledge of scientific units.

• Ability to manipulate algebraic expressions.

• Some familiarity with system of units.

Learning Objectives
By the end of this topic, you should be able to:

• Distinguish between fundamental and derived units.


2 TOPIC 1. UNITS CONVERSION AND DIMENSIONAL ANALYSIS

• Recognise SI system of units.

• Identify Chemical Engineering SI units.

• Convert between SI and non-SI units.

• Comprehend and apply dimensional consistency.

• Identify and use dimensionless groups.

©H ERIOT-WATT U NIVERSITY
TOPIC 1. UNITS CONVERSION AND DIMENSIONAL ANALYSIS 3

1.1 Introduction
Most European schools, colleges and universities have fully embraced metric units and
most have adopted a family of consistent metric units called the "SI system" - this should
be familiar to all school science students.
Tthe SI system of units should be used in all laboratory reports, assignments and
projects. Examinations will generally be framed in SI units, but students will be asked to
convert non-SI units.
Metrication is less well advanced in the USA because many large US multi-national
companies still use "US Customary Units", which developed originally from English units
which, in turn, were later standardised into the "Imperial Units" family.
In addition to the wide influence of US companies within the Chemical & Process
industries, there are other reasons why unit conversion may be needed:

• Some plants are still equipped with older pre-metrication instrumentation and it
may be expensive to upgrade such systems.

• Many textbooks are still framed in terms of US Customary units.

• Published physical property data whether charts, tables or correlations may be


given in a wide variety of non-SI units.

• Other non-SI metric systems also exist - "CGS" and "MKS" are two such families -
and these often feature in older books and literature.

Even in a European context, some non-SI units are still widely used - for instance " ◦ C"
and "bar" are often used in place of "K" (kelvin) and "Pa" (pascal):

• Chemical concentration can appear per "L" (litre) instead of per "m 3 ".

• Kinetic and thermal data can appear in terms of "cal" (calorie) instead of "J" (joule).

For the above reasons it is vitally important to be able to convert and manipulate units.

©H ERIOT-WATT U NIVERSITY
4 TOPIC 1. UNITS CONVERSION AND DIMENSIONAL ANALYSIS

1.2 Fundamental & Derived SI Units


There are seven fundamental SI units - these are the basic units from which all other
units are derived:

Quantity Dimension SI Unit


mass [M] kg (kilogram)
length [L] m (metre)
time [T] s (second)
temperature [θ] K (kelvin)
amount of substance [N] mol (mole)
electric current [I] A (ampere)
luminous intensity [J] cd (candela)

Only the first five are commonly used in Chemical Engineering. Derived SI units can
be broken down into products and quotients of these fundamental units raised to some
power:

• Using a simple analogy - fundamental units are a little like elements.


• Based on the same analogy - derived units are a little like compounds.

Due to their importance some derived SI units are given special names - these are
usually to honour famous scientists (who would have worked in a closely related field).
The only exception below is "angle" - the "radian" is not a scientist’s name:

Quantity SI Name SI Symbol Fundamental SI


units
force newton N kg m/s2
pressure pascal Pa (N/m 2 ) kg/m s 2
energy joule J (N m) kg m2 /s2
power watt W (J/s) kg m2 /s3
frequency hertz Hz 1/s
angle radian rad -

The radian is the ratio of two distances - the length of an arc subtended by an angle,
divided by the radius of the circle. The units therefore cancel so that the "rad" is a pure
number with no units.
Do not confuse "dimensions" with "units" - dimensions are independent of the system
of units used. The following table lists some more derived SI units together with their
associated dimensions. Notice the convention of using square brackets for dimensions:

©H ERIOT-WATT U NIVERSITY
TOPIC 1. UNITS CONVERSION AND DIMENSIONAL ANALYSIS 5

Quantity Fundamental Basic Grouped


SI Units Dimensions Dimensions
velocity m/s [L]/[T] [L][T] -1
volume m3 [L][L][L] [L] 3
density kg/m3 [M]/[L] 3 [M][L] -3
specific volume m3 /kg [L] 3 /[M] [L] 3 [M] -1
force kg m/s 2 [M][L]/[T] 2 [M][L][T] -2
pressure kg/m s 2 [M]/[L] [T] 2 [M][L] -1 [T] -2
energy kg m2 /s 2 [M][L] 2 /[T] 2 [M][L] 2 [T] -2
power kg m2 /s 3 [M][L] 2 /[T] 3 [M][L] 2 [T] -3

All the quantities listed in the left-hand column, for instance velocity, volume, density
will have exactly the same dimensions whether expressed in SI Units or US Customary
units - this is the power of "dimensional analysis".
Examine the grouped dimensions of density ρ and then examine the grouped
dimensions of specific volume ν. Clearly there must be some sort of reciprocal
relationship between them.
The exact relationship between ρ and ν might not be known but dimensional analysis
shows that it must somehow be a reciprocal relation with no other dimensional quantity
involved.
"Force", which can be thought of as either "mass times acceleration", or as "rate of
change of momentum" - notice the dimensions of each are exactly the same:

Force = ma = [M][L][T]-2

Force = m(v-u)/t = [M][L][T]-1 [T]-1 = [M][L][T]-2

©H ERIOT-WATT U NIVERSITY
6 TOPIC 1. UNITS CONVERSION AND DIMENSIONAL ANALYSIS

1.3 Unit Family


A "Unit Family" or a "Unit Set" is a set of units where the derived units are related to the
fundamental units in a systematic way - some important unit sets are listed below:

• SI units - these are based on fundamental units already given in section 1.2.

• Customary US units - these are used widely throughout US process and chemical
industries and have only been kept alive by the size of this sector.

• Field units - these are used within sections of the oil and gas industry.

• "CGS" units are a sub-set of the metric "MKS" units - CGS units are used only for
small-scale scientific and/or laboratory work.

US Customary system emerged from the "English/Imperial" system and its fundamental
units are: length is the foot (ft); mass is the pound-mass (Ibm ); time is the second (s);
temperature is the degree Fahrenheit ( ◦ F); amount of a substance is the pound-mole (Ib
mol). In addition:

• Absolute, thermodynamic, temperature is the degree Rankine ( ◦ R).

• Area is the foot squared (ft2 ) - alternatively acres are used for land area.

• Volume is either cubic feet (ft3 ) or US gallons (US gal) - dry/wet respectively.

• Force is the pound-force (Ibf ) - that force exerted by 1 Ibm in the Earth’s
gravitational field.

• Pressure is the pound-force per square foot (Ib f /ft2 ) or (Ibf /in2 ) also known as (psi).

• Mechanical and heat energy are the (ft Ib f ) and the (BTU) respectively.

• Power is the horse power (h.p.) - notional pulling power of one horse.

The "CGS" system is a small-scale metric set of units. The fundamental units are:
length is the centimetre (cm); mass is the gram (g); time is the second (s); temperature
is degree kelvin (K); amount of a substance is the mole (mol):

• Force is the dyne and 1 dyn = 1 g cm/s 2 .

• Pressure is the dyne per square centimetre and 1 dyn/cm 2 = 1 g/cm s2 .

• Energy is the erg and 1 erg = 1dyn cm = 1g cm 2 /s2 .

• Power is erg/s and 1erg/s = 1g cm 2 /s3 .

Field units will be covered later in a Petroleum Engineering module.

©H ERIOT-WATT U NIVERSITY
TOPIC 1. UNITS CONVERSION AND DIMENSIONAL ANALYSIS 7

1.4 SI Units for Chemical Engineers


In section (1.2) seven fundamental SI quantities were presented. In the same section
several derived SI quantities were also presented - these derived units force, pressure,
energy, power and frequency are important enough to be named after famous scientists.
Other derived quantities arise frequently in Chemical Engineering. They do tend to have
symbols which are widely recognised within the profession - see below:

Quantity Symbol Fund-SI units


area A m2
volume V m3
density ρ kg/m 3
specific volume ν m 3 /kg
velocity (average) u m/s
mass flow ṁ kg/s
specific heat capacity Cp J/kg K or ( m 2 /s 2 K)
viscosity µ N s/m 2 or (kg/m s)
thermal conductivity k W/m K or (kg m/s 3 K)

In order to extend the range "prefixes" are used before the units - for instance:

Prefix Symbol Multiplier


"kilo" as in kilogram kg 10 3 g
"milli" as in milligram mg 10 -3 g
"micro" as in microgram µg 10 -6 g

The "CGS" system of units was based on the fundamental units "cm-g-s" while the
"MKS" system was based on the fundamental units "m-kg-s". Both are metric systems,
however, the units defined by the "CGS" system are really too small for widespread use.
The "MKS" system, through time, evolved into the present "SI" system which has now
been almost universally adopted throughout the world.
The SI system of units is a consistent set of units (CGS is also consistent) - by consistent
it is meant that the definition of key derived units eliminates the need for constants. For
instance, consider the SI definitions below:
In SI the unit of force is the "newton" and is defined as:

• 1 newton (N) is that force needed to accelerate a mass of 1 kg by 1 m/s 2 .

©H ERIOT-WATT U NIVERSITY
8 TOPIC 1. UNITS CONVERSION AND DIMENSIONAL ANALYSIS

In SI the unit of energy is the "joule" and is defined as:

• 1 joule (J) is work(energy) needed to displace a force of 1 N through 1 m.

In SI the unit of power is the "watt" and is defined as:

• 1 watt is that power developed when 1 J of work (energy) is done in 1 s.

The "FPS" system is not a consistent set of units because 1 Ib f is defined as that force
needed to accelerate 1 Ib m by 32.174 ft/s2 (standard gravitational acceleration - g). This
constant can re-appear in later equations as g c
In SI "heat" and "work" have the same units as "energy", expressed in J - this is true
because heat and work are just different types of energy. In the "FPS" system heat and
work have different units - although, as expected, they can be converted one into the
other.
See the end of this topic for an extended list of "SI Units for Chemical Engineers". These
tables will be supplied in examination papers, so students should become familiar with
them.

©H ERIOT-WATT U NIVERSITY
TOPIC 1. UNITS CONVERSION AND DIMENSIONAL ANALYSIS 9

1.5 Conversion Factors


Consider the following example: a heat exchanger is to be fitted with 3/4 in O.D. (outside
diameter) tubes. Convert this into "m" - notice that "mm" is not an SI unit.
There are several ways of doing this - one possibility is as follows:
1 in = 25.4 mm

∴ 0.75 in = 25.4×0.75 mm = 19.05 mm

Now converting from "mm" into "m"

1000 mm = 1 m

1 mm = 1/1000 m

19.05 mm = 19.05/1000 m = 0.01905 m

Although technically correct the above approach is tedious - a Chemical Engineer would
convert the units in a "chain-wise" procedure as follows:

! "! "
25.4 mm 1 m
OD of Tube = 0.75 in = 0.01905 m
1 in 1000 mm

Notice how "in" and "mm" cancel out top and bottom leaving the correct answer in "m" -
if the student had erroneously multiplied by "1000" instead of dividing then "mm" would
not have cancelled correctly - and "m" would have not ended in the numerator.
Conversion tables are supplied at the end of this topic. Do become familiar with the
conversion table at the end of this topic which will be supplied in examinations. There
are other standard textbooks with conversion tables - (Felder and Rousseau, 2008) and
(Himmelblau and Riggs, 2012).

Example : 1.5.1
Problem:
Convert 20 ft/hr into m/s.
Solution:
First the individual conversions are needed:
1 ft = 0.3048 m
1 hr = 60 min
1 min = 60 s
! "! "! "
ft 0.3048 m 1 hr 1 min
Velocity = 20 = 0.00169 m/s
hr 1 ft 60 min 60 s
..........................................

©H ERIOT-WATT U NIVERSITY
10 TOPIC 1. UNITS CONVERSION AND DIMENSIONAL ANALYSIS

Example : 1.5.2
Problem:
Convert 101.325 kN/m 2 into mm Hg.
Solution:
First the individual conversions are needed:
1 kN = 1000 N
1 mm Hg = 133.32 N/m 2

! "! "
kN 1000 N 1 mm Hg
Pressure = 101.325 2 = 760 mm Hg
m 1 kN 133.32 N/m2

..........................................

Example : 1.5.3
Problem:
Convert 14.8 BTU/Ib mol ◦ F into kJ/kmol K
Solution:
First the individual conversions are needed:
1 BTU = 1.05506 kJ
1 Ib mol = 0.45395 kmol
1.8 ◦ F = 1 K (temperature difference)
! "! "! "
BTU 1.05506 kJ 1 Ib mol 1.8 o F
Molar CP = 14.8
Ib mol o F 1 BTU 0.45395 kmol 1 K
! "
kJ
∴ Molar CP = 61.9
kmol K

..........................................

©H ERIOT-WATT U NIVERSITY
TOPIC 1. UNITS CONVERSION AND DIMENSIONAL ANALYSIS 11

1.5.1 Coherent and Non-Coherent Systems


SI and CGS units are known as coherent systems because the unit of force is defined
exactly in terms of fundamental units without any constants:

1 newton (N) = 1 kg ×1 m/s2

1 dyne (dyn) = 1 g ×1 cm/s2

With US units the Pound Force (Ib f ) is defined as that force that arises when 1 Ib m is
accelerated under one standard gravity, which at sea level, 45 ◦ latitude, is 32.174 ft/s2 :

1 Ibf = 1 Ibm × 32.174 ft/s2

The symbol gc is used as a conversion factor to convert fundamental units into derived
units as follows

! " ! "
32.174 Ibm ft/s2 1 kg m/s2
gc = =
1 Ibf 1 N

Example : 1.5.4
Problem:
Calculate the Potential Energy (PE) (ft Ib f ) of a block, 200 Ibm suspended 15 ft in the air
- take the local acceleration due to gravity as 32.2 ft/s2 :
Solution:
! "
# $ 1 Ibf
PE = mgh = 200 × 32.2 × 15 = 96, 600 (Ib m ) ft/s2 (ft)
32.174 Ibm ft/s2
PE = 3, 002 ft Ibf

Notice that in the formula for Potential Energy all the units are fundamental units
(Ibm ft/s2 ), while it is the derived unit (Ibf ) that is needed - this is where gc comes in,
it converts the fundamental units into the derived unit (Ib f ).
In SI no such conversion factor is needed since 1 N equates exactly to 1kg x 1m/s 2 .
..........................................

©H ERIOT-WATT U NIVERSITY
12 TOPIC 1. UNITS CONVERSION AND DIMENSIONAL ANALYSIS

1.5.2 Chemical Engineering Scale


Just as the (kg) is used in place of the (g), so in Chemical Engineering the (kmol) is used
in place of the (mol), the (kN) is used in place of the (N), the (kJ) in place of the (J), the
(kN/m2 ) is used in place of the (N/m2 ) and the (kW) is used in place of the (W).
When measuring production rates it is more likely that units of production will be
kg/annum or tonnes/annum rather than kg/s - note the following conversions:

a) There are 1000 kg in a metric "tonne".

b) A "long ton" is 2,240 Ib m = 1,016.06 kg.

c) A "short ton" is 2,000 Ibm = 907.18 kg.

The normal ton is the "long ton" - if the "short ton" is inferred it must be stated explicitly,
Oil companies use "field units" - thus, oil flow is measured in STB/day (stock tank barrels
per day) and gas in SCF/day (standard cubic feet per day).
Volume measurement also tends to be on a large scale - in field units STB/day and
SCF/day - however, be careful as the US gallon and the UK gallon are different, as are
the number of gallons in a "barrel":

a) 1 US gallon = 0.3785 m 3 .

b) 1 UK gallon = 0.4546 m 3 .

c) 1 US Barrel = 42 US gallons = 34.97 UK gallons.

The "CGS" system of units is totally inappropriate for general Chemical Engineering
usage, since it is built around "g-cm-s" units and these are far too small for industrial
use.

©H ERIOT-WATT U NIVERSITY
TOPIC 1. UNITS CONVERSION AND DIMENSIONAL ANALYSIS 13

1.6 Temperature Scales


There are a variety of temperature scales - with empirical scales arbitrary numbers are
placed against a pair of reproducible reference states (the freezing point & boiling point
of pure water at atmospheric pressure are good reference states to use):

Reproducible
Centigrade Difference Difference Fahrenheit
Reference
Scale Centigrade Fahrenheit Scale
State
Boiling Point of ↑ ↑
100◦ C 212◦ F
Pure Water 100 degrees 180 degrees
Freezing Point
0◦ C ↓ ↓ 32◦ F
of Pure Water

Thus, there are 100 "degrees" in the centigrade scale and 180 "degrees" in the
Fahrenheit scale, therefore, each 1 ◦ C is equivalent to 180/100 = 1.8 ◦ F; also, there
is an offset of 32 degrees between the scales so that:

◦F = (1.8 x ◦ C) + 32 ........................(1.1)

An absolute scale places the zero point at the absolute zero of temperature which
corresponds to -273.15◦C, on the centigrade scale, and -459.67 ◦F on the Fahrenheit
scale; these scales are called "Kelvin" and "Rankine" respectively (note rounding below):

Reproducible
Centigrade Fahrenheit
Reference Kelvin Scale Rankine Scale
Scale Scale
State
Boiling Point of
100◦ C 373 K 212◦ F 672◦ R
Pure Water
Freezing Point
0◦ C 273K 32◦ F 492◦ R
of Pure Water
"Absolute Zero" -273◦ C 0K -460◦ F 0◦ R

The size of the "degree" is the same for both the centigrade and Kelvin scales and the
same (but larger by factor of 1.8) for both the Fahrenheit and Rankine scales.
Rounding absolute zero to -273 ◦ C and -460 ◦ F gives the followign conversions:
K = ◦ C + 273

◦R = ◦ F + 460

©H ERIOT-WATT U NIVERSITY
14 TOPIC 1. UNITS CONVERSION AND DIMENSIONAL ANALYSIS

1.7 Density and Specific Gravity Units


As stated before there is a simple reciprocal relationship between density and specific
volume (specific volume is more widely used in Thermodynamics):
Quantity Symbol Fund-SI Units Dimensions
density ρ kg/m3 [M][L]-3
specific volume ν m3 /kg [L]3 [M]-1

Example : 1.7.1
Problem:
Look up tables and find the approximate density of water in (kg/m 3 ) and then from
conversion tables convert this into (Ibm /ft3 ) - see end of topic for conversion tables.
Solution:
% &
1 Ibm /ft3
Approx density water (US units) = 1000 kg/m3 = 62.44 Ibm /ft3
16.016 kg/m3

Notice how the conversion factor

1 Ibm /ft3 = 16.016 (kg/m3 )


! "
16.016 kg/m3
is re-arranged into a factor . Next decide whether to multiply or divide
1 Ibm /ft3
by this factor - in this case dividing cancels out the SI density leaving the correct value
in US density units.
Specific gravity "sg" is often used - it is the ratio of two densities as follows:

Density of Substance
sgsubstance = ........................(1.2)
Density of Water

The density of any substance depends on temperature and pressure (note, the pressure
dependence of liquids is very weak) - the temperature dependence is strong, so that "sg"
is usually expressed as, for instance, sgETOH (20◦ C/20◦ C):

• The above means that the densities of "Ethanol" (substance) & "Water" in equation
(1.2) are both at 20 ◦ C

• It is not necessary to use the same temperature for both - for instance, one could
write sgsubstance (20◦ C/4◦ C).

(Perry and Green, 2008) provides accurate density tables - often densities are measured
at 101.325 kN/m2 .
..........................................

©H ERIOT-WATT U NIVERSITY
TOPIC 1. UNITS CONVERSION AND DIMENSIONAL ANALYSIS 15

1.8 Pressure Units


The SI unit of pressure is "pascal", but 1 Pa = 1 N/m 2 . Note that N/m2 is more often
used in this department:

• 1 pascal is that pressure produced by a force of 1 newton acting uniformly over an


area of 1 m2 .

• The force must act perpendicularly to surface. Pressure is also known as a


"Normal Stress" - normal meaning perpendicular and stress meaning force/unit
area.

• The pressure at the base of a liquid column h (m), density ρ (kg/m 3 ) is given by
the well-known expression

P = ρgh........................(1.3)

Where,
g = acceleration due to gravity or 9.81 (m/s2 ).

Pressure can either be measured as "Absolute Pressure" or "Gauge Pressure":

• If "0" on the pressure scale is assigned to a complete vacuum then the


measurement is known as "Absolute Pressure" - see LHS of diagram below.

• If "0" on the pressure scale is assigned to atmospheric pressure then the


measurement is known as "Gauge Pressure" - see RHS of diagram below.

• Distinguish between absolute and gauge pressure by appending "absolute" or


"gauge" after unit. In this case 501.325 kN/m 2 absolute is same as 400 kN/m2
gauge:

©H ERIOT-WATT U NIVERSITY
16 TOPIC 1. UNITS CONVERSION AND DIMENSIONAL ANALYSIS

The expression "Gauge Pressure" probably originates from a time when a pressure
gauge invariably read out on such a scale; now they can also be supplied to read out in
terms of "Absolute Pressure".
The relation between the two pressure scales, see also previous diagram, is as follows:

PGauge = PAbsolute - PAtmospheric ........................(1.4)

Where,

PAbsolute = absolute pressure (N/m2 absolute) or (kN/m2 absolute)

PGauge = gauge pressure (N/m2 gauge) or (kN/m2 gauge)

Since the zero of the absolute pressure scale is a complete vacuum such a pressure
reading is, therefore, completely independent of the current atmospheric pressure.

• Notice the same absolute pressure in a vessel will read as different gauge
pressures as the atmospheric pressure varies from day-to-day.

"US Customary" and "English" units measure pressure in (Ib f /in2 ) - shorted to (psia) or
(psig). Note, 1 atmosphere ≈ 14.7 psia.
Pressure measurements below atmospheric pressure (partial vacuum) occur in many
items of process plant. For instance, say a pressure gauge reads 2 (psi) below
atmospheric pressure, which in turn is 14.7 (psia):

• The pressure inside the vessel is then either 12.7 psia or -2 psig. Notice that
equation (1.4) can still be used to find gauge pressure - another way specifying
this pressure is to say that it corresponds to "2 psi of vacuum".

PAtmospheric = 14.7 psia

PAbsolute = 12.7 psia

PGauge = -2 psig

©H ERIOT-WATT U NIVERSITY
TOPIC 1. UNITS CONVERSION AND DIMENSIONAL ANALYSIS 17

1.8.1 Manometric Pressure


A common instrument used for measuring pressure is the manometer. This is simply a
U-tube filled with a liquid of known density.
If one end is connected to a vessel or pipe and the other to the atmosphere (as below),
then the pressure within the vessel may be calculated in terms of (N/m 2 gauge):

Patm

Pv

25cm

The manometric fluid is often water or mercury depending on the fluid within the
vessel. The pressure difference ∆P between the left hand leg P v and the right hand
leg Patm (in this case it is open to atmosphere) is given by a similar equation to (1.3)

∆P = ρgh........................(1.5)

Where,

∆p = difference in fluid pressure between tapings (N/m 2 ).

ρ = manometer fluid density (kg/m3 ).

g = acceleration due to gravity or 9.81 (m/s2 ).

h = deflection of the manometer fluid (m).

The pressure difference can also be expressed in terms of "liquid depth", i.e. (mm H 2 0)
or (mm Hg) but the manometer liquid must be clearly stated so that correct density may
be used in equation (1.5) to convert back to mainstream pressure units (N/m 2 ).
With the right hand leg open to atmosphere (as above) the pressure calculated from
(1.5) will have units (N/m2 gauge) - use (1.4) to convert to (N/m2 absolute). If both
legs are connected to process equipment then (1.5) simply gives (N/m 2 ) of pressure
difference.

©H ERIOT-WATT U NIVERSITY
18 TOPIC 1. UNITS CONVERSION AND DIMENSIONAL ANALYSIS

Example : 1.8
Problem:
Calculate the absolute pressure 30 m below the surface of a tank containing a fluid with
density 1380 kg/m3 . The tank is not pressurised and is open to the atmosphere. Given
the above determine the following:

a) Calculate the pressure in (Pa) and then convert to (m H 2 O) - use equation (1.3) to
find the pressure at any depth within a body of fluid.

b) Calculate the pressure directly in terms of (m H2 O).

Take atmospheric pressure to be 10.38 m H 2 O and density of water to be 1000 kg/m 3 .


Solution:
(a) Use equation (1.3) to find the pressure at any depth. Students must fill in the
missing parts of the calculations.

P = ρgh = 1380 x 9.81 x 30 = N/m 2 gauge

This is gauge pressure - so next calculate atmospheric pressure

∴ P atm = P = ρgh = 1000 x 9.81 x 10.38 = N/m 2

Now use equation (1.4) to find absolute pressure

Pabsolute = Pgauge + Patmospheric = + N/m2 absolute

∴ Pabsolute = 507,961.8 N/m 2 absolute

This is about 508 (kN/m2 ) or 508 (kPa) or 5.08 (bar). The answer is also required in
(m H2 O) - thus re-arranging equation (1.3) leads to:

P 507, 961.8
h= = = m H2 O absolute.
ρg 1000 × 9.81

(b) To calculate directly in (m H2 O) the depth of fluid must be converted to an equivalent


depth of water

1380 x 9.81 x 30 = 1000 x 9.81 x h water

∴h= m H2 O gauge

∴ h = 41.4 + 10.38 = m H 2 O absolute


..........................................

©H ERIOT-WATT U NIVERSITY
TOPIC 1. UNITS CONVERSION AND DIMENSIONAL ANALYSIS 19

1.9 Energy Units


Energy must be conserved - it may be converted into different forms but can never be
created or destroyed:

• In Chemical Engineering common forms of energy are electrical energy, kinetic


and potential energy, energy in the form of work, energy in the form of heat and
chemical energy.

Since energy can be converted into different forms, it follows that the units (of the
different forms) must either be the same or, if different, conversion factors must be
available:

• In SI all forms of energy are expressed in "joule" (J). Recall that 1 joule of work is
done when a force of 1 newton is moved through a distance of 1 m. Thus we can
write 1 (J) = 1 (N m) = 1 (kg m2 /s2 ).

SI employs the "joule" (J) for all forms of energy. However, "CGS" and "US" units are
different - in these cases "work" and "heat" are expressed in different units, although
conversion factors must be available, see below:

Unit Set Work (Energy) Heat (Energy)


SI 1 J - the amount of work done when JP Joule found by careful stirring
a force of 1 N is moved through a measurements "The Mechanical
distance of 1 m. Equivalence of Heat".
1 kcal = 4.1868 kJ exactly.
CGS 1 erg - the amount of work done 1 cal, or calorie - the amount of heat
when a force of 1 dyne is moved needed to raise 1 g of water by 1 o C.
through a distance of 1 cm.
1 cal = 4.1868 10 7 erg = 4.1868 J
1 erg = 10 -7 J
US 1 ft Ib f - the amount of work done 1 BTU or British Thermal Unit - the
when a force of 1 Ib f is moved amount of heat needed to raise 1 Ib
through a distance of 1 ft. of water by 1 o F.
1 ft Ib f = 1.3558 J 1 BTU = 778.18 ftIb f = 1.05506 kJ

Energy in SI units can also be expressed as (power x time):

1 kW h = 3600 kW s = 3600 (kJ/s) s = 3600 kJ = 3.6 MJ


1 thm (therm) = 105 BTU = 105.51 MJ

©H ERIOT-WATT U NIVERSITY
20 TOPIC 1. UNITS CONVERSION AND DIMENSIONAL ANALYSIS

1.10 Dimensional Consistency


Consider any general equation linking the three unspecified quantities x, y and z
together as follows:

z =x+y

Because of the addition sign all the quantities must have the same fundamental units;
for instance, all may be mass, all may be volume or all may be velocity - but it is not
possible for any of them to have different fundamental units.
If each of the quantities has units of mass then it makes no sense if x is given in (kg)
while y is given in (Ibm ) - neither can be in (kg) and y in (g).
Circumstances are a little different when multiplying and/or dividing quantities. Consider
the following example

(a + b + cd)
z=
w

The quantities a, b and cd must have same units since these are added together and the
a b cd
quantities , and must all have the same units and these must be identical to the
w w w
units of z.
For liquids and low-pressure gases, the pressure drop ∆P in a pipe is given by

L ρu2
∆P = 4f ........................ (1.6)
d 2

By studying the units on the RHS of this equation it is possible to identify the correct
units for the pressure drop ∆P .

Example : 1.10
Problem:
For a Newtonian fluid flowing under strictly laminar conditions within a circular pipe the
Hagen-Poiseuille equation describes how the pressure drop of the fluid varies with
volumetric flowrate. Given this equation and a definition of all the terms check the
equation for dimensional consistency.
Solution:
The Hagen-Poiseuille equation for Newtonian fluids flowing in laminar flow through
circular pipes is given by:

8LµQ
∆P =
π r4

©H ERIOT-WATT U NIVERSITY
TOPIC 1. UNITS CONVERSION AND DIMENSIONAL ANALYSIS 21

Look up the table at the end of this topic and note the SI units for each of the variables
that appear in the above equation. List these units in the table below. Change the SI
units into fundamental dimensions. Students must complete this table:

Variable Description Units Dimensions


∆P Pressure drop
L Length of pipe
µ Fluid Viscosity
Q Volumetric Flow
r Pipe radius
8LµQ should give ∆P
π r4

Check that the fundamental dimensions of the first and last entries in the "dimensions"
column match.
Also check that the units of the first and last entries also match - it will be necessary to
replace any derived SI units with fundamental SI units - see immediately below.
If there are problems converting derived units into fundamental units and associated
dimensions - then notice how it is done for the unit of force (newton):
1 N = 1 kg×1 m/s2 - thus force has fundamental dimensions = [M][L][T] -2 .
..........................................

©H ERIOT-WATT U NIVERSITY
22 TOPIC 1. UNITS CONVERSION AND DIMENSIONAL ANALYSIS

1.11 Dimensionless Quantities & Groups


The derived unit the "rad" (radian) is a dimensionless quantity since it actually has
dimensions of [L]/[L] or, in terms of units, (m)/(m) - either way these cancel out. Notice
that "rad" is simply a pure number with no dimensions or units.
L
In equation (1.6) , the terms 4f and are actually dimensionless. Thus, the units of
d
∆P must be derived exclusively from the units of ρu2 . Knowing this, as an exercise, go
back two pages and check the units of ∆P .
In Chemical Engineering scaling-up pilot scale plant to full scale requires that the ratio
of certain forces be the same - this is one reason why dimensionless groups are useful.
One very important dimensionless group is Reynolds number Re, defined as

duρ
Re = ........................ (1.7)
µ

Where,
Re = Reynolds number (-)

d = pipe diameter (m)

u = average fluid velocity (m/s)

ρ = fluid density (kg/m3 )

µ = fluid viscosity (N s/m2 )

Now work out all the dimensions (below) for each term in equation (1.7) above. First
note down fundamental units for each variable and then change units into dimensions:

Variable Description Units Dimensions


d pipe diameter
u average velocity
ρ fluid density
µ fluid viscosity

Next substitute the above dimensions into equation (1.7) and then gather together like
dimensions top and bottom, which leads to:

Re =

It should be clear from the above that Re has no dimensions (they all cancel top and

©H ERIOT-WATT U NIVERSITY
TOPIC 1. UNITS CONVERSION AND DIMENSIONAL ANALYSIS 23

bottom) - Reynolds Number Re is said to be dimensionless.


The Reynolds number has physical meaning: it is the ratio of "inertial forces" to "viscous
forces" - if Re is the same between pilot plant and the real plant then the flow profile is
the same and the hydraulic performance in each case should be comparable.
It is common to find equations (called correlations) containing a combination of
dimensionless groups, together with various constants and exponents, such as:

G1 = k Ga2 Gb3

The various Gi are the dimensionless groups, a and b are the exponents (powers)
and k is a dimensionless constant. There are distinct advantages in terms of putting
experimental data together in this way:

1. There are fewer groups to correlate experimental data against than there were
original variables (groups are made up of variables).

2. The form of the correlations and the numerical values of the constants and
exponents are the same no matter which system of units is used.

3. The groups themselves have physical meaning and are used so that the results
from small-scale trials apply to much larger plants.

Chemical Engineers use these dimensionless correlations extensively to design heat,


mass and momentum transfer equipment. They help determine how fast heat, mass
and momentum are transferred.

Example : 1.11.1
Problem:
A typical correlation for the "heat transfer coefficient", between the fluid and the wall of
a stirred tank, is shown below:
Solution:

! " ! "b ! "c


hd L2 N ρ k
=a
k µ CP µ

©H ERIOT-WATT U NIVERSITY
24 TOPIC 1. UNITS CONVERSION AND DIMENSIONAL ANALYSIS

Where,

d = vessel diameter

h = heat transfer coefficient

k = thermal conductivity of fluid

µ = viscosity of fluid

N = stirrer speed (r.p.m)

L = stirrer length

Cp = heat capacity of fluid.

ρ = density of fluid

As an exercise for the student, for each variable fill in the "units" column and then work
out the dimensions:
Variable Description SI Units Dimensions
h heat transfer coefficient
d vessel diameter
k thermal conductivity
L stirrer length
N stirrer speed
µ viscosity of fluid
ρ density of fluid
Cp heat capacity of fluid

! "
hd
= (fill in dimensions)
k
! "
L2 N ρ
= (fill in dimensions)
µ
! "
k
= (fill in dimensions)
CP µ

..........................................

If equations are dimensionless there is no need to convert them to work with different
system of units - unfortunately not all equations are dimensionless, so that sometimes it
is necessary to convert constants so that equation will work with different units.

©H ERIOT-WATT U NIVERSITY
TOPIC 1. UNITS CONVERSION AND DIMENSIONAL ANALYSIS 25

A simple example would be the hypothetical expression below:

d = 3t + 4 ........................ (A)
Where,

d = distance (km)

t = time (hr)

In order to be dimensionally consistent the constants 3 & 4 must have units of 3 (km/hr)
and 4 (km) respectively.
Suppose the equation must be altered, keeping the same form, but is now required to
produce the correct answer with SI Units - that is the equation should still take the form:

d" = x t" + y

But now,

d" = distance (m)

t" = time (s)

Find x and y as follows - first the relations between old/new d and t are needed:

d" t"
d= &t=
1000 3600

To find the new constants substitute the two expressions above into (A) to get:

d" t"
=3 +4
1000 3600

Or,

d" = 0.833 t" + 4000 ........................ (B)

As a student exercise, check this out: by equation (A) after 0.5 hr the distance d =
km
Whereas, equation (B) after 1800 s yields a distance d " = m

©H ERIOT-WATT U NIVERSITY
26 TOPIC 1. UNITS CONVERSION AND DIMENSIONAL ANALYSIS

Example : 1.11.2
Problem:
Convert the correlation below written in terms of US units so that has the same form,
but will work with SI units:
Solution:


370 (1 + 0.00067 T ) v
h= √
d
Where,

h = heat transfer coefficient (BTU/hr ft2 ◦ F)

T = temperature (◦ F)

v = fluid velocity (ft/s)

d = diameter (in)
Check the relation between old US and new SI variables are as given below - the new
SI variables are shown with a "prime", the old US variables without a "prime":

h" v"
h= & v=
5.6783 0.3048
100 d"
T = 1.8 T " + 32 & d=
2.54

Now substitute these expressions into (A) and simplify - students should now work out
the two constants a and b that should produce equivalent answers when working with SI
Units.


") v"
a (1 + b T
h" = √
d"
a b

Finally students should check results for the data given below:
T = 60 ◦ F T" = ◦C

v = 8 ft/s v" = m/s


d = 1.5 in d" = m

h= BTU/hr ft2◦ F h" = W/m2 K


..........................................

©H ERIOT-WATT U NIVERSITY
TOPIC 1. UNITS CONVERSION AND DIMENSIONAL ANALYSIS 27

1.12 Tutorial Topic 1


1. Convert the following to the units shown

Value Unit Convert to


1800 US gal/hr m3 /s
25 lb/hr kg/min
52 cal/oz ◦ F J/kg K

2. The pressure inside a vessel is measured by three pressure gauges.

Gauge 1 2 3
Reading 10 0.7 50
Units on gauge psi bar mm Hg

One of these is suspected to give false readings. By converting the values into a
consistent form, determine which gauge is faulty.
3. Three suppliers have quoted the following prices for a particular chemical:

Supplier Price
A £180 / tonne
B £0.185 / kg
C £181 / long ton

Which is the cheapest source of supply?


4. Given that the SI units of pressure and heat transfer coefficient are (N/m 2 ) and
2
(W/m K) respectively, determine the dimensions of these quantities.
5. The feed to a chemical reactor must be heated from 60 ◦ F to 170◦ F. Express these
temperatures and the temperature increases in a) centigrade, b) Rankine, c) Kelvin.
6. At what value do the Centigrade and Fahrenheit scales coincide? Express this
temperature in both Rankine and Kelvin.
7. 10 kg of salt are added to a tank containing 2000 litres of water. Express the
concentration of salt in the water as a) concentration in kg/m 3 , b) lb/ft3 , c) mole fraction.
8. Nitrobenzene has a density of 74.9 lb/ft 3 and benzene a density of 880 kg/m 3 .
Calculate the density of a mixture formed by mixing 4.4 UK gallons of nitrobenzene and
3.0 cubic feet of benzene - assume that the volume of the mixture is the sum of the two
feed component volumes.
9. The pressure of a gas in each of the vessels shown below is indicated by means of
a mercury filled open ended manometer, (specific gravity of mercury = 13.6). Express

©H ERIOT-WATT U NIVERSITY
28 TOPIC 1. UNITS CONVERSION AND DIMENSIONAL ANALYSIS

the pressure in each vessel as:


a) N/m2 gauge.
b) N/m2 absolute.

(Atmospheric pressure in each case is 755 mm Hg)


In what other way might the pressure in b) be expressed?

a) b)

25cm 40cm

10. A spherical tank 10 ft in diameter holds 20 kg of gas which has an average


molar mass of 32 kg/kmol. The temperature inside the tank is 40 ◦ C. Assuming ideal gas
behaviour, calculate the pressure inside the tank in the following units:
a) psia
b) psig

©H ERIOT-WATT U NIVERSITY
EPS Chemical Engineering, Heriot-Watt University SI Unit Conversion Sheet
1.13

SI UNITS FOR CHEMICAL ENGINEERS


Common Dimensionless Groups
The Basic SI Units Euler number En P
Quantity Name Symbol
u2
Length metre m
Mass kilogram kg Froude number Fr
u2
Time second s gl
Temperature Kelvin K
Grashof number Gr
Electrical current ampere A L3 g T 2 / µ2
Luminous intensity candela cd Nusselt number Nu hd / k
Note: “kilo” is in small “k”. K stands for Kelvin Reynolds number Re ud / µ
Peclet number for heat transfer Pe Re.Pr
Derived Units
Peclet number for mass transfer Pe Re.Sc
Quantity Name Symbol
2 Prandtl number Pr Cp µ / k
Force newton N = kgm/s

©H ERIOT-WATT U NIVERSITY
Work, energy, heat joule J = Nm Schmidt number Sc µ
Power watt W = J/s D
Electrical charge coulomb C = As Sherwood number Sh h dd / D
Electrical potential volt V = W/A
Stanton number for heat transfer St Nu/(RePr)
Electrical capacitance farad F = As/V
Stanton number for mass transfer St Nu/(ReSc)
Electrical resistance ohm =V/A 0.67
-1 j factor for heat transfer jh StPr
Frequency hertz Hz = s 0.67
j factor for mass transfer jd StSc
Magnetic flux Weber Wb = Vs
2
Magnetic flux density tesla T=Wb/m
Common symbols
Inductance henry H = Vs/A
2 Cp = Specific heat capacity at constant pressure (J/kg-K)
Pressure pascal Pa = N/m 2
g= Acceleration due to gravity (m/s )
2
h= Film heat transfer coefficient (W/m -K)
Prefixes for Unit Multiples
hd or kc= Film mass transfer coefficient (m/s)
Factor name symbol factor name symbol
Units and conversion tables

-12 k= Thermal conductivity (W/m-K)


10 pico p 10 deca da
-9 2 d or l = Characteristic size (diameter or length)
10 nano n 10 hecto h 2
-6 3 P= Pressure (N/m )
10 micro µ 10 kilo k
-3 6 u= Velocity (m/s)
10 milli m 10 mega M 2
-1 9 U= Overall heat transfer coefficient (W/m -K)
10 deci d 10 Giga G Temperature difference (T2-T1)
T=
2
D= Diffusivity (m /s)
Approximate Physical Properties µ= Viscosity (kg/ms)
Water Air 3
3 3 3 = Density (kg/m )
Density 10 kg/m Density at s.t.p 1.2 kg/m
-3 2 -5 2
Viscosity (18°C) 10 Ns/m Viscosity at s.t.p. 1.7x10 Ns/m
3
Specific heat capacity 4.18 kJ/kg-K Specific heat capacity 1x10 J/kg-K Atomic Weights of Common Elements and names of Common Compounds
Thermal conductivity 0.6 W/m-K Thermal conductivity 0.024 W/m-K C Carbon 12 N Nitrogen 14
TOPIC 1. UNITS CONVERSION AND DIMENSIONAL ANALYSIS

Latent heat (1 atm) 2 MJ/kg Cl Chlorine 35.5 Na Sodium 23


Prandtl number (20°C) 7 Prandtl number 0.7 Cu Copper 63 I Iodine 127
F Fluorine 19 O Oxygen 16
Some Common Physical Constants He Helium 4 S Sulphur 32
26 -1
Avogadro’s Number NA=6.023x10 kmol
2
Gravitational Acceleration g=9.807 m/s O2 Oxygen HNO3 Nitric Acid CH4 Methane
Ideal gas constant R=8314 J/kmol-K N2 Nitrogen H2SO4 Sulphuric Acid C 2 H6 Ethane
(the same as 8.134 kJ/kmol-K) H2 Hydrogen NaOH Caustic Soda CH3OH Methanol
-8 2 4
Stefan-Boltzman constant = 5.6697x10 W/m -K NaCl Salt
3
Volume of 1 kmol of an ideal gas at s.t.p. 22.4 m
2
Standard temperature and pressure (s.t.p) 101.3 kN/m , 273.15 K

Revised July 2013 M.Barr & Dr G.White


29
30

EPS Chemical Engineering, Heriot-Watt University SI Unit Conversion Sheet

CALORIFIC VALUE 1 Btu/ft3 37.25 9 kJ/m3


1 Btu/lb 2.326 kJ/kg
CONVERSION FACTORS
VOLUMETRIC FLOW 1 ft3/s 0.028316 m3/s
TIME 1 min 60 s
1 ft3/h 7.8658 cm3/s
1 hr 3600 s
1 UK gal/h 1.2628 cm3/s
1 US gal/h 1.0515 cm3/s
LENGTH 1 in 25.4 mm
1 ft 0.3048 m
MASS FLOW 1 lb/h 0.12600 g/s
1 yd 0.9144 m
1 ton/h 0.28224 kg/s
1 mile 1.6093 km
1 micron 10-6 m
DENSITY 1 lb/in3 27.680 g/cm3
1 angstrom 10-10 m
1 lb/ft3 16.019 kg/m3
2 2 1 lb/UK gal 99.776 kg/m3
AREA 1 in 645.16 mm
1 lb/US gal 119.83 kg/m3
1 ft2 0.092903 m2
1 g/cm3 1000 kg/m3
3
VOLUME 1 in 16.387 cm3
MOLAR MASS 1 lb/lbmol 1 kg/kmol
1 ft3 0.02832 m3
(MOLECULAR WEIGHT) 1 g/gmol 1 kg/kmol
1 UK gal 4546.1 cm3
1 US gal 3785.4 cm3 3
MOLAR VOLUME 1 ft /lbmol 0.06243 m3/kmol
1 litre 10-3 m3
1 l/gmol 1 m/kmol
1 cm3 10-6 m3
1 barrel 0.15899 m3
MOMENT OF INERTIA 1 lb – ft2 0.042140 kg – m2
MOMENTUM 1 lb – ft/s 0.13826 kg – m/s
MASS 1 oz 28.352 g
PRESSURE 1 lb/in2 (psi) 6.8948 kN/m2
1 lb 0.45395 kg
1 lb/ft2 47.880 N/m2
1 ton 1016.06 kg
1 dyn/cm2 0.1 N/m2
1 short ton 907.18 kg
1 atm (standard) 101.325 kN/m2
1 tonne 1000 kg
1 bar 100 kN/m2
1 grain 0.0648 g
1 kg/cm2 (1 at) 98.0665 kN/m2
o 1 ft water 2.9891 kN/m2
TEMPERATURE F (oC) x 1.8 +32
o 1 in water 249.09 N/m2
C (o F - 32) / 1.8
o o 1 in Hg 3.3864 kN/m2
R F + 460
o 1 mm Hg (1 torr) 133.32 N/m2
K C + 273
POWER, HEAT FLOW 1 hp (British) 745.70 W
TEMPERATURE 1 deg F ) 1 C (or K)
1 erg/s 10-7 W
DIFFERENCE 1 deg R ) deg
1 .8 1 ft – lbf /s 1.3558 W
1 Btu/h 0.29307 W
VELOCITY 1 ft/s 0.3048 m/s
1 Chu/h 0.5275 W
1 mile/hr 0.44704 m/s
1 kcal/h 1.16306 W
1 ton of refrigeration 3516.9 W
ACCELERATION 1 ft/s2 0.3048 m/s2
DYNAMIC VISCOSITY 1 P (poise = 1 g cm-s) 0.1 N – s/m2
FORCE 1 pdl 0.13826 N
1 lb/ft – h 0.41338 mN – s/m2
1 lbf 4.4482 N
1 lb / ft – s 1.4882 N – s/m2
1 kgf 9.8067 N
1 dyn 10-5 N
KINEMATIC VISCOSITY 1 S (stokes = 1 cm2/s) 10 –4 m2 / s
1 ft2 / h 0.25806 cm2 / s
ENERGY, WORK, HEAT 1 ft – lbf 1.3558 J
1 ft –pdl 0.04214 J
SURFACE ENERGY 1 dyn/cm 10-3 J / m2
1 cal 4.1868 J
TOPIC 1. UNITS CONVERSION AND DIMENSIONAL ANALYSIS

SURFACE TENSION (= 1 erg/cm2)


1 erg 10-7 J
1 Btu 1.05506 kJ
THERMAL CONDUCTIVITY 1 Btu/hr-ft-°F 1.7307 W/m-K
1 Chu 1.8991 kJ
1 Kcal/hr-m-°C 1.163 W/m-K
1 kW-hr 3.6 MJ
1 therm 105.51 MJ
HEAT TRANSFER 1 Btu/hr-ft2-°F 5.6783 W/m2-K
COEFFICENT
SPECIFIC ENTHALPY 1 Btu/lb 2.326 kJ/kg
MASS TRANSFER 1 ft/sec 0.3048 m/s
(LATENT HEAT etc) 1 kcal/kg 4.1868 kJ/kg
COEFFICIENT
o
SPECIFIC HEAT CAPACITY 1 Btu/lb- F 4.1868 kJ/kg-K

SPECIFIC ENTROPY 1 cal/g-oC 4.1868 kJ/kg-K

Revised July 2013 M.Barr & Dr G.White

©H ERIOT-WATT U NIVERSITY
31

Topic 2

Engineering Chemistry

Contents
2.1 Introduction . . . . . . . . . . . . . . . . . . . . . . . . . . . . . . . . . . . . . . 33
2.1.1 Chemical Equation . . . . . . . . . . . . . . . . . . . . . . . . . . . . . . 34
2.1.2 Molar Mass . . . . . . . . . . . . . . . . . . . . . . . . . . . . . . . . . . 35
2.1.3 Chemical Formulae . . . . . . . . . . . . . . . . . . . . . . . . . . . . . 36
2.1.4 Kmol, Ib mol and Conversions . . . . . . . . . . . . . . . . . . . . . . . 36
2.2 Calculations in Chemistry . . . . . . . . . . . . . . . . . . . . . . . . . . . . . . 38
2.3 The Basis . . . . . . . . . . . . . . . . . . . . . . . . . . . . . . . . . . . . . . . 40
2.4 Limiting and Excess Reactants . . . . . . . . . . . . . . . . . . . . . . . . . . . 41
2.4.1 Fractional Excess and Percentage Excess . . . . . . . . . . . . . . . . . 42
2.5 Conversion, Yield and Selectivity . . . . . . . . . . . . . . . . . . . . . . . . . . 43
2.6 Mole & Mass Fractions . . . . . . . . . . . . . . . . . . . . . . . . . . . . . . . . 46
2.7 Average Molar Mass of Mixture . . . . . . . . . . . . . . . . . . . . . . . . . . . 48
2.8 Other Concentration Units . . . . . . . . . . . . . . . . . . . . . . . . . . . . . . 49
2.9 Changing Concentration Units . . . . . . . . . . . . . . . . . . . . . . . . . . . 50
2.10 Reactions in Parallel . . . . . . . . . . . . . . . . . . . . . . . . . . . . . . . . . 51
2.11 Tutorial Topic 2 . . . . . . . . . . . . . . . . . . . . . . . . . . . . . . . . . . . . 54
2.12 Bibliography . . . . . . . . . . . . . . . . . . . . . . . . . . . . . . . . . . . . . . 55

Prerequisite knowledge

• Fundamental knowledge of chemical reactions and moles.

• Fundamental knowledge of mass conservation principle.

• Basic understanding of molar mass.

• Some familiarity with reaction stoichiometry.

• Some understanding of molarity and molality.

Learning Objectives
By the end of this topic, you should be able to:

• Write down a balanced chemical reaction.


32 TOPIC 2. ENGINEERING CHEMISTRY

• Comprehend the importance of stoichiometric coefficients for reacting species.

• Construct “before” and “after” material balance tabulations.

• Identify and modify the basis of a material balance.

• Change between different concentration units.

• Solve simple open-ended problems in a systematic way.

• Deal with inert species and reactants species in excess.

• Distinguish between irreversible and reversible reactions.

©H ERIOT-WATT U NIVERSITY
TOPIC 2. ENGINEERING CHEMISTRY 33

2.1 Introduction
Chemical Engineering will open up a whole range of career choices, such as process
development, process design, plant operations, safety, marketing, etc.
Throughout this topic a range of process calculations and problem solving techniques
will be demonstrated. However, it is impossible to tackle every problem - the idea is to
show how general principles can be applied within a structured framework.
The process diagram for the manufacture of ammonia is very complex because it depicts
all the equipment needed to produce ammonia from methane and air. There is more
than one chemical reaction taking place:

CH4 + H2 O → CO + 3H2
CO + H2 O → CO2 + H2
N2 + 3H2 → 2NH3

One approach is to simplify the process by drawing a "block diagram" - this breaks
down the process into a sequence of simple “blocks” - each block may represent a
reaction step, a separation stage or a waste treatment process:
Air CO

NH3
CH4

Many things need to be known about the process but the first and most important is to
be able to balance all the material flows entering and leaving each block.
Even a simplified block diagram can become complex, so a starting point is to consider
only a single block - this topic deals with material balances around a "reactor block" and
will concentrate on the following items:

• The chemical equation used to describe a process - there is no need to know the
exact mechanism, only the overall reaction equation is needed.

• The amount of materials that enter the reactor and the amount of materials that
are used up and are formed within the reactor.

• The amount of materials that are unreacted (not all reactants are entirely
consumed) and the amount of materials that leave the reactor

The overall technique of accounting for mass or moles of material entering and leaving
any item of equipment is called a "Material Balance".

©H ERIOT-WATT U NIVERSITY
34 TOPIC 2. ENGINEERING CHEMISTRY

2.1.1 Chemical Equation


The first task is to write a balanced chemical equation. For example, suppose sulphur
dioxide is to be reacted with oxygen to form sulphur dioxide - as a first step simply write
down the chemical species taking part in the reaction

SO2 + O2 → SO3

Now balance this equation by making sure that the number of atoms, of each species,
are the same either side of the equation.

2SO2 + O2 → 2SO3

Balancing a chemical reaction is very important because it provides the correct


stoichiometric ratios in which reactants are consumed and products are produced:

• For every 2 mol of SO2 consumed, 1 mol of O 2 is consumed and 2 mol of SO 3 are
produced.

• The ratio in which the species SO 2 , O2 , SO3 react is, therefore, 2:1:2.

Stoichiometric coefficients (see below) also have a sign.

SO2 + 1/2O2 → SO3

Although the above is correct, where possible use whole numbers instead of fractions;
given that the stoichiometric coefficients are ratios it is easier to ratio integers than
fractions, however, the same answer will be obtained either way.
Components not taking part in the reaction need not be shown, for example N 2 is an
inert species in a combustion reaction. A further example may be when one or more of
the reactant species are in aqueous solution:

acid + alkali → salt + water

Water from the original reactant solutions does not take part in the reaction - so does
not appear on the left. The water formed by the reaction must, of course, appear on the
right -remember, the reaction equation only represents species reacting.
As a final point about stoichiometric coefficients consider the equation below:

υ 1 A1 + υ 2 A2 → υ 3 A3 + υ 4 A4 ....................(2.1)

©H ERIOT-WATT U NIVERSITY
TOPIC 2. ENGINEERING CHEMISTRY 35

Where,

υ i = stoichiometric coefficients of reactant & product species.

Ai = the various chemical species taking part in reaction.


There is a sign convention for each individual stoichiometric coefficient "υ i ":

• υ i is negative for reactant species.

• υ i is positive for product species.

• υ i is zero for inert species.

The sign associated with individual coefficients allows reactant species to be consumed
(negative υ i ), product species to be produced (positive υ i ) and any inert species to pass
through unchanged (zero υ i ). No sign is needed in the reaction equation.
Stoichiometric coefficients have a sign but do not have units - they are simply numbers,
positive, negative or zero - as seen they are in fact simple ratios. They should not be
confused with actual moles or kmoles of each species present.
The overall stoichiometric coefficient υ is also significant - it is found by summing
individual stoichiometric coefficients υi across the reaction as follows:

'
ν= νi = ν4 + ν3 − ν1 − ν2 =....................(2.2)

If this quantity is positive, more moles overall will be present "after" the reaction than
existed "before". If it is negative, then fewer moles overall will exist "after" than existed
"before" - either way overall moles "before" and "after" will not balance.
However, if the overall stoichiometric coefficient υ is zero, then there will be the
same number of moles overall "before" and "after" - only then will moles or kmole be
conserved.
Even if kmoles are not conserved kmoles entering and leaving (continuous plant) or
kmoles before and after (batch plant) may be balanced if the overall kmoles reacted
(produced or consumed) is known - this will be shown later in this topic.

2.1.2 Molar Mass


In order to complete a "material balance" it is necessary to know the molar mass of
each species (kg/kmol), however, molecules are a collection of atoms thus work out
molar mass from atomic masses - the molar mass of SO3 = (3×16) + 32 = 80 kg/kmol.
Although the periodic table lists atomic masses to a high degree of accuracy, in
engineering calculations only the approximations are needed.
For example a few common atomic masses are listed below:

©H ERIOT-WATT U NIVERSITY
36 TOPIC 2. ENGINEERING CHEMISTRY

H O N C Na S Cl Si
1 16 14 12 23 32 35.5 30

Over time students will come to remember some of more common atomic masses but,
in any chemical engineering exam, values will be provided. Many standard textbooks
are supplied with periodic tables (Perry and Green, 2008).

2.1.3 Chemical Formulae


Over time students will also come to remember some of the more common chemical
formulae but for now some are listed in the table below.
Students should learn the more common ones:

Alkanes Methane CH4


Ethane C2 H6
Alkenes Ethene - Ethylene C2 H4
Propene C3 H6
Alcohols Ethanol C2 H5 OH
Simple acids Sulphuric H2 SO4
Nitric HNO3
Hydrochloric HCl
Alkali’s Sodium Hydroxide NaOH
Salts Sodium Chloride NaCl
Gases Hydrogen H2
Oxygen O2
Nitrogen N2

2.1.4 Kmol, Ib mol and Conversions


The amount of a substance is given the dimension [N] has units of (mol) and is one of
only seven fundamental units in the SI system.

Mole definitions
A mole is the amount of any substance that contains the same number
mole
of entities (ions, electrons, atoms or molecules) as 12 gram of 12 C.
Thus, a mole equates to a number of atoms or molecules - the number
mole
is Avogadro’s number: N = 6.023 x 10 23 entities

1 gram mole (g mol) or (mol) of any substance is that quantity of the substance that has
a mass in grams numerically equal to the molar or atomic mass of the substance.

©H ERIOT-WATT U NIVERSITY
TOPIC 2. ENGINEERING CHEMISTRY 37

In chemical engineering, due to the size of processes, kilogram moles (kmol) are used
in place of gram mole (g mol).
Thus, for SO3 the gram mole and the kilogram mole (after checking molar mass) would
be as follows:
1 g mol of SO3 would weigh 80 grams.
1 kmol of SO3 would weigh 80 kg.
In calculations never mix g mol and kmol in the same problem; with SI units the kmol is
the most suitable - however, it is also possible to define the (Ib mol) or the (t mol):
1 Ib mol of SO3 would weigh 80 Ib.
1 t mol of SO3 would weigh 80 tonne.

Molar mass can be considered a conversion factor, for example:


1 kmol of SO3 weighs 80 kg → molar mass of SO 3 = 80 (kg/kmol).
1 Ib mol of SO3 weighs 80 Ib → molar mass of SO 3 = 80 (Ib/Ib mol).
There are more details in standard textbooks (Himmelblau and Riggs, 2012).

©H ERIOT-WATT U NIVERSITY
38 TOPIC 2. ENGINEERING CHEMISTRY

2.2 Calculations in Chemistry


Example : 2.2.1
Problem:
Take the complete combustion of 1 mole of methane with stoichiometric ratio of
oxygen. Notice that moles of methane are specified, so this is starting point.
Solution:
This reaction is irreversible with reactants present in their stoichiometric proportions.
The balanced chemical reaction is given by

CH4 + 2O2 → CO2 + 2H2 O

Check that the above chemical equation is properly balanced - the number of carbon,
hydrogen and oxygen atoms left and right should be the same (1, 4, 4 respectively).
1 mole of CH4 is specified and, according to reaction equation, reacts with 2 mole O 2
(present stoichiometrically). Now convert "before moles" to "before mass" using molar
mass of each species - students are required to fill in all the blanks in this table.
1 mole of CH4 produces 1 mole of CO 2 plus 2 moles H2 O. Now convert "after moles"
to "after mass" as before. It is important to lay out the calculations in a systematic way
using tabulation below - notice that all reactants are consumed:

Component Before Molar Before Reacting After After Mass


Moles Mass Mass (mol) Moles (g)
(mol) (g/mol) (g) (mol)
CH4 1 -1 0
O2 2 32 -2 0
H2 O 0 18 +2 2
CO2 0
Σ 3 - 0

Overall mass entering/leaving is kg (must be the same).


Check that overall moles entering/leaving are mol (may be same).
'
For mole balance check ν = νi = ν4 + ν3 − ν1 − ν2 = . Moles are conserved only
if this is zero.
..........................................

©H ERIOT-WATT U NIVERSITY
TOPIC 2. ENGINEERING CHEMISTRY 39

Example : 2.2.2
Problem:
96 kg of sulphur dioxide reacts stoichiometrically with oxygen to produce sulphur
trioxide . Notice mass of sulphur dioxide is specified so this is starting point.
Solution:
Write down properly balanced chemical reaction for this irreversible reaction.

2SO2 + O2 → 2SO3

Check that the above reaction equation is properly balanced (2 "S" and 6 "O" left and
right). Overall mass must balance (before and after), but overall kmoles cannot balance
(before and after) - students should consider why kmoles cannot be conserved in this
case.
The proportions of each species reacting are obtained from the reaction equation - so
first convert 96 kg of SO2 into kmoles, i.e. 96/64 = 1.5 kmol of SO 2 is present as "before
moles" and this reacts with 1.5 × (1/2) = 0.75 kmol O 2 .
Convert 0.75 kmol O2 back to "before mass" - thus, 0.75 x 32 = 24 kg O 2 "before mass".
Total mass before is kg.
According to reaction coefficients each mole of SO 2 produces equal moles of SO 3 .
Thus, SO3 produced = 1.5 kmol.

Before Molar Before After


Reacting After Mass
Component Mass Mass Moles Moles
(kmol) (kg)
(kg) (kg/kmol) (kmol) (kmol)
SO2 1.5 -1.5
O2 24 32 0.75 -0.75 0 0
SO3 0 80 0 +1.5 1.5 120
Σ - 2.25 -0.75

The final step is to convert "after moles" to "after mass" as before. All reactants are
consumed since reaction is irreversible using stoichiometric ratios:

• Students must fill in all the blank entries in the table.


• Check that overall mass balances before/after g.
• Overall moles does not balance: kmol before, but only
kmol after.
• The overall stoichiometric coefficient is .

..........................................

©H ERIOT-WATT U NIVERSITY
40 TOPIC 2. ENGINEERING CHEMISTRY

2.3 The Basis


Example : 2.3
Problem:
In the previous case 96 kg of sulphur dioxide was specified in the reactant mixture,
it is said that the "basis" of the calculation is 96 kg SO2 in reactant mixture. Now
change basis to 100 kg SO2 in reactant mixture - the basis must be clearly stated
at the beginning.
Solution:
Basis: 100 kg SO2 in reactant mixture.
Always check the reaction is properly balanced because the stoichiometric coefficients
provide the molar ratios with which reactants are consumed and products are produced.
Remember these coefficients have no units; they are just numbers with a sign.

2SO2 + O2 → 2SO3

The coefficients in the reaction equation have no sign because it is known which species
are products and which are reactants just by looking at the equation.
Use this new basis to produce the table below - as an exercise the student must
complete all missing entries in the table.
Hint, first change 100 kg of SO2 into kmoles. Although kmoles overall are not conserved
they can be balanced:

Before Molar Before After


Reacting After Mass
Component Mass Mass Moles Moles
(kmol) (kg)
(kg) (kg/kmol) (kmol) (kmol)
SO2 100 64 1.5625 -1.5625 0 0
O2 32
SO3 0 80 0 +1.5625 1.5625 125
Σ -

Using the present "basis" 100 kg SO 2 reacts with kg O2 to produce


125 kg SO3 . The "basis" is easily changed by scaling as follows:

• 96 kg must react with 25 x 96/100 = kg O 2 . To produce 125 x


96/100 = kg SO3
• - see results using previous "basis" to confirm scaling.

..........................................

©H ERIOT-WATT U NIVERSITY
TOPIC 2. ENGINEERING CHEMISTRY 41

2.4 Limiting and Excess Reactants


The reaction equation might suggest that all reactants will react to produce product;
in other words the reaction will go to 100% completion. In reality there may be some
unreacted reactants left over.
One method of increasing the conversion to one reactant is to supply another in excess
(in excess, that is, to the theoretical stoichiometric requirement).
If one reactant is in excess then the other one must be limiting. If there are an excess of
one reactant then there is a better chance that most of the limiting reactant will collide
and react, given a limited residence time within the reactor.

Example : 2.4
Problem:
500 kg of 70 mass% NaOH solution and 1,000 kg of 80 mass% H 2 SO4 are mixed
together in a batch process. Calculate which reactant is limiting, which is in excess
and the amount of Na 2 SO4 that is formed together with the composition of the final
mixture.
Solution:
Basis: 500kg of NaOH and 1,000kg of H 2 SO4 solution
Atomic mass of Na is 23 kg/kmol, O is 16 kg/kmol, H is 1 kg/kmol and S is 32 kg/kmol.
Thus, the molar masses of the various compounds are as follows:

• NaOH is 40 (kg/kmol).

• H2 SO4 is 98 (kg/kmol).

• H2 O is 18 (kg/kmol).

• Na2 SO4 is 142 (kg/kmol).

In this example, one of the reactants is supplied in excess. The objective is 2to find out
which one and how much product there would be assuming the reaction, shown below,
is 100% complete:

2NaOH + H2 SO4 → Na2 SO4 + 2H2 O

Since acid and alkali are in solution, the first step is to work out the actual kmol of acid,
alkali and water "before" the reaction takes place (it is a batch reactor).
500 kg of 70 mass% NaOH solution leads to NaOH = 500 × 0.7 = 350 kg NaOH and,
therefore, there must be 150 kg water in solution. Moles NaOH "before" the reaction
takes place = 350/40 = 8.75 kmol and moles of H 2 O "before" = 150/18 = 8.33 kmol.
Repeating this calculation for 1,000 kg of 80 mass% H 2 SO4 leads to 800 kg of H 2 SO4
and 200 kg of H 2 0 - converting to kmoles leads to: 8.163 kmol H 2 SO4 and 11.11 kmol
of H2 O:

©H ERIOT-WATT U NIVERSITY
42 TOPIC 2. ENGINEERING CHEMISTRY

Component Molar Mass Before Before Mass After Moles After Mass
Moles
(kg/kmol) (kg) (kmol) (kg)
(kg)
NaOH 40 8.75 350 0 0
H2 SO4 98 8.163 800 3.788 371.22
Na2 SO4 142 0 0 4.375 621.25
H2 O 18 19.44 350 28.19 507.42
Totals: - 36.353 1500 36.353 1499.9

Now 8.75 kmol of NaOH reacts with 8.75 × (1/2) = 4.375 kmol H 2 SO4 . Thus, unreacted
kmol H2 SO4 after reaction = 8.163 - 4.375 kmol = 3.788 kmol.
8.75 kmol of NaOH reacts to produce 8.75 × (1/2) = 4.375 kmol Na 2 SO4 and also
produce 8.75 × (2/2) = 8.75 kmol of water, which added to original 19.44 kmol of water
of solution gives 29.19 kmol of water "after".
Now simply convert from kmol to kg in the usual way and complete the table.
Therefore, NaOH is the limiting reactant since it is completely exhausted. H 2 SO4 is the
excess reactant since some of it is left after all the NaOH has reacted. The material
balance is satisfied with same mass in/out (neglecting rounding errors).
The amount of Na 2 SO4 formed is 621.25 kg. The composition is 0 mass% NaOH
(limiting reactant), 24.7 mass% H2 SO4 (excess reactant - some is left unreacted after
limiting reactant has been exhausted), 41.4 mass% Na 2 SO4 and 33.8 mass% H 2 O.
..........................................

2.4.1 Fractional Excess and Percentage Excess


When dealing with limiting and excess reactants, a useful measure is the term
fractional excess or percentage excess defined as follows:

Amount supplied in Excess of Stoichiometric Requirement ...(2.3)


Fractional Excess =
Stoichiometric Requirement

Percentage Excess = Fractional Excess x 100 ........................ (2.4)

Thus the fractional excess of H2 SO4 = (8.163 - 4.375)/4.375 = 0.866, or the percentage
excess of H2 SO4 = 86.6 %
By definition the calculation of percentage excess reactant is always based on the
assumption of complete conversion of the limiting reactant. That is, all the limiting
reactant is completely used up.

©H ERIOT-WATT U NIVERSITY
TOPIC 2. ENGINEERING CHEMISTRY 43

2.5 Conversion, Yield and Selectivity


Not all reactions go to 100% completion and to account for this the term "fractional
conversion" is widely used - it is defined as follows:

Moles of component reacted ..........(2.5)


Fractional Conversion =
Moles of component in feed

The term "percentage conversion" is simply the fractional conversion expressed as a


percentage. The fractional conversion may be based on either reactant, but usually it is
the fractional conversion to the limiting reactant that is of most interest.
Very often side or simultaneous reactions occur and to account for this "selectivity" is
needed - it is defined as follows:

Moles of Desired Product Produced ..........(2.6)


Selectivity =
Moles of Undesired Product

Finally the "yield" is sometimes quoted - it is defined as follows:

Moles of Product Produced ..........(2.7)


Yield =
Moles of Reactant Provided

Unfortunately there are a number of definitions of yield, so students are warned to


carefully take note which definition is being used in any given situation.
In addition, students should get into the habit of quoting what expression they are using
when writing reports or working on projects.

Example : 2.5.1
Problem:
Take the reaction:
A+B →C
The initial quantities of A, B and C are 1, 1.5 and 0 kmol respectively. If 0.3 kmol of A
is present in the final mixture calculate the following:

a) The final amounts of B and C present.

b) The % conversion of A.

c) The % conversion of B.

d) The yield of C on A.

e) The yield of C on B.

©H ERIOT-WATT U NIVERSITY
44 TOPIC 2. ENGINEERING CHEMISTRY

Solution:

a) If 0.3 kmol of A is left, then 0.7 kmol of A must have reacted. Likewise 0.7 kmol of
B must have reacted and 0.7 kmol of C must have been formed.
This means the final mixture must consist of 0.3 kmol of A, 0.8 kmol of B and 0.7
kmol of C.
b) Now apply the definition of percentage conversion to species A:
1 − 0.3
% Conversion of A = × 100 = 70%
1.0
c) Now apply the definition of percentage conversion to species B:
0.7
% Conversion of B = × 100 = 47%
1.5
d) Now apply the definition of yield of C on A:
0.7
Yield of C on A = = 0.7
1.0
e) Now apply the definition of yield of C on B:
0.7
Yield of C on A = = 0.47
1.5
..........................................

Example : 2.5.2
Problem:
Take the simultaneous reactions:
A+B →C
2A → D
The initial and final quantities of A, B, C and D are listed in the table below calculate
the following:

a) The total % conversion of A.


b) The % conversion of A to C.
c) The % conversion of B to C.
d) The yield of C on A.
e) The yield of C on B.
f) Selectivity of C with respect to D.

A B C D
Initial (kmol) 10 12 0 0
Final (kmol) 1 7 - -

©H ERIOT-WATT U NIVERSITY
TOPIC 2. ENGINEERING CHEMISTRY 45

Solution:
The first step is to find Cand D produced. Looking at these reactions A takes part in
both, however, B only takes part in the first reaction and notice that 5 kmol of B have
reacted in the first reaction.
Thus, in the first reaction only, 5 kmol of A have reacted and 5 kmol of C have been
produced. Since C does not appear in the second reaction, this means there must be 5
kmol of C in the final mixture - fill this into the table below:

A B C D
Initial (kmol) 10 12 0 0
Final (kmol) 1 7 5 -

Now turning to second reaction - it can be seen that 9 kmol of A in total have reacted,
but only 5 kmol of A reacted in first reaction, thus, 4 kmol of A must have reacted in
reaction two. If 4 kmol of A reacted then 2 kmol of D must have been produced:

A B C D
Initial (kmol) 10 12 0 0
Final (kmol) 1 7 5 2

a) Now apply the definition of total percentage conversion to species A:


10 − 1
% Conversion of A = × 100 = 90%
10
b) Now apply the definition of percentage conversion of A to C:
10 − 5
% Conversion of A to C = × 100 = 50%
10
c) Now apply the definition of percentage conversion of B to C:
12 − 7
% Conversion of B to C = × 100 = 42%
12
d) Now apply the definition of yield of C on A:
5
Yield of C on A = = 0.5 kmol C/kmol A
10
e) Now apply the definition of yield of Con B:
5
Yield of C on B = = 0.42 kmol C/kmol B
12
f) Now apply the definition of selectivity of C with respect to D:
5
Selectivity = = 2.5 kmol C/kmol D
2

..........................................

©H ERIOT-WATT U NIVERSITY
46 TOPIC 2. ENGINEERING CHEMISTRY

2.6 Mole & Mass Fractions


Reactions can involve one single component decomposing into two or more
components. Alternatively, several species may combine to form other species, as was
the case in Examples 2.2.1, 2.2.2 and 2.3.
Thus, there are very few situations where only a single pure component is present. Most
"streams" in a process plant are multi-component mixtures.
Not only is it important to know the overall mass or mole flow, it is just as important
to know the composition of each component in the mixture - with these two pieces of
information mass or moles of each component may be found.
Compositions may be expressed as mole fractions or as mass fractions as follows:

Moles of component “i” in mixture ..........(2.8)


Mole fraction of component “i” =
Total moles of mixture

Mass of component “i” in mixture ..........(2.9)


Mass fraction of component “i” =
Total mass of mixture

In some calculations mole fractions are required (for instance when predicting physical
properties of mixtures) in other types of calculation mass fractions will be needed:
• For instance moles ni kmol and mass mi kg of a component in a mixture may
be found from overall mole n T kmol and overall mass mT kg of the mixture as
follows:

ni = xi nT ....................(2.10)

mi = wi mT ....................(2.11)

Where xi and wi are mole and mass fractions respectively. Students should
be careful because often x i is used for both mole and mass fraction.

Example : 2.6
Problem:
A gas mixture contains 10 mol% hydrogen, 30 mol% methane, 50 mole% ethane and 10
mole% propane (note mol% can be found by multiplying mole fraction * 100). Calculate
the composition as mass% - remember mass% and mol% will NOT be the same.
Solution:
Basis: 100 kmol of mixture
Choosing this "basis" means that species mol% will be the same as kmoles of each
species.
Mole fractions and mass fractions must sum to one this can be written mathematically
as follows:

©H ERIOT-WATT U NIVERSITY
TOPIC 2. ENGINEERING CHEMISTRY 47

'
4
xi = x1 + x2 + x3 + x4 = 1....................(2.12)
1

'
4
wi = w1 + w2 + w3 + w4 = 1....................(2.13)
1

Again lay out the calculation in a tabular form working from what is given to what is
required - get in the habit of doing calculations like this systematically:
Mol% and kmoles of each component are the same because of the basis chosen.
Component mass (kg) for ethane = 50x30 = 1500 kg and weight% for ethane is simply
(1500/2440) x 100 = 61.48%:

Component Molar Component


Mole Mass
Component Moles Mass Mass
(%) (%)
(kmol) (kg/kmol) (kg)
H2 10 10 2 20 0.82
CH4 30 30 16 480 19.67
C2 H6 50 50 30 1500 61.48
C3 H8 10 10 44 440 18.03
Σ 100 100 - 2440 100

Component weight% is simply component weight fraction times 100. Students must
check that all entries in the above table are correct.
..........................................

©H ERIOT-WATT U NIVERSITY
48 TOPIC 2. ENGINEERING CHEMISTRY

2.7 Average Molar Mass of Mixture


Example : 2.7
Problem:
A gas mixture again contains 10 mol% hydrogen, 30 mol% methane, 50 mole% ethane
and 10 mole% propane. Calculate the average molar mass of the mixture.
Solution:
Basis: 100 kmol of mixture
The problem is the same as Example 2.6 thus the basis chosen is the same - notice
mole% and kmoles of each species must sum to one hundred.
There are two ways of solving this problem - first the average molar mass of the mixture
M may be found from mole fraction weighting the molar masses M i of each individual
component (species). Specifically, for these four components

'
n
M= xi Mi = x1 M1 + x2 M2 + x3 M3 + x4 M4 ..........(2.14)
1

∴ M = (0.10 × 2) + (0.3 × 16) + (0.5 × 30) + (0.1 × 44) = 24.4 kg/kmol

Mol% are easily converted to mole fraction. A second approach is to lay out the entire
calculation again - the average molar mass is then total mass divided by total moles:

Molar Component Component


Mole Mass
Component Mass Moles Mass
(%) (%)
(kg/kmol) (kmol) (kg)
H2 10 2 10 20 0.82
CH4 30 16 30 480 19.67
C2 H5 50 30 50 1500 61.48
C3 H8 10 44 10 440 18.03
Σ 100 - 100 2440 100

Using this approach average molar mass of mixture = 2440/100 = 24.4 kg/kmol.
Molar mass usually has units (g/mol), but this is same as (kg/kmol).
..........................................

©H ERIOT-WATT U NIVERSITY
TOPIC 2. ENGINEERING CHEMISTRY 49

2.8 Other Concentration Units


Mole fraction and mass fraction are two common ways of specifying concentration of a
component in a mixture - the confusing array of different concentration units is because
each tends to be used in different circumstances and/or processing equipment.
Mass concentration (kg/m3 ) of component i in a mixture is defined as follows:

Mass of i in mixture (kg) ..........(2.15)


Mass concentration of component i =
Total volume of mixture (m3)

Molar concentration of component i in a mixture (kmol/m 3 ) is defined as follows:

moles of i in mixture (kmol) ..........(2.16)


Molar concentration of component i =
Total volume of mixture (m3)

Molarity of a component i in a solution (mol/l) - often used in small-scale pilot


plant/laboratory work and is defined as follows:

moles of solute i in the solution (mol) ..........(2.17)


Molarity of solute i =
Total volume of solution (litre)

For instance a 0.5 molar aqueous solution of H 2 SO4 contains 0.5 mol of H2 SO4 per litre
of solution (mixture). Note, the volume is "solution" and not "solvent" - with mixing or
dissolution the "solution" volume may be larger/smaller than starting volumes.
Molality (less common than above) of component "i" in a solution (mol/kg) is defined:

moles of solute i in the solution (mol) ..........(2.18)


Molality of component i =
Mass of solvent (kg)

When only two species dissolve or mix together the terms "solute", "solvent" and
"solution" apply. With multi-component systems the terms "components" and "mixture"
apply.

©H ERIOT-WATT U NIVERSITY
50 TOPIC 2. ENGINEERING CHEMISTRY

2.9 Changing Concentration Units


Example : 2.9
Problem:
A 0.5 molar (sometimes written as 0.5 M) aqueous solution of H 2 SO4 has a density
of 1030 kg/m3 - this is the density of the resulting solution (mixture). Calculate the
following:

a) Mole fraction of H2 SO4 in mixture xi

b) Mass fraction of H2 SO4 in mixture wi .

c) Molality of H2 SO4 mi .

d) Mass concentration of H2 SO4 ρi .

Solution:
Basis: 1 litre (l) of solution or 1000 (cm3 )
This is an appropriate basis since, according to equation (2.17) a 0.5 molar solution of
H2 SO4 contains 0.5 mole H2 SO4 in every litre of solution.
Try tabulating such calculations - start with what is given and then find mass and moles
of each component (H 2 SO4 & H2 O) from this all else can be found.
Mass H2 SO4 = 98x0.5 = 49 g, mass of solution = 1000x1.030 = 1030 g. Thus, mass of
H2 O = 1030 - 49 = 981 g and moles of H 2 O = 981/18 = 54.5 mol.

H2 SO4 Solution H2 O
Moles Molar Molar
Mass Volume Density Mass Mass Moles
Mass Mass
(g) (cm3 ) (g/cm3 ) (g) (g) (mol)
(mol) (g/mol) (g/mol)
0.5 98 49 1000 1.030 1030 981 18 54.5

0.5 49
(a) xH2 SO4 = = 0.0091 (b) wH2 SO4 = = 0.0476
0.5 + 54.5 49 + 981

0.5 49
(c) mH2 SO4 = = 0.51 mol/kg (d) ρH2 SO4 = = 49 g/l or kg/m3
0.981 1
..........................................

©H ERIOT-WATT U NIVERSITY
TOPIC 2. ENGINEERING CHEMISTRY 51

2.10 Reactions in Parallel


Example : 2.10
Problem:
2 kg/s of a gas mixture comprising 70 mole% CH 4 and 30 mol% C 2 H6 is burned
completely with 10% excess air. Calculate the mass flow rate of air needed and the
composition (wt%) of the product gases. Take air to be 79 mol% N 2 and 21 mol% O 2 .
Solution:
Basis:100 kmol/s of gas being burnt
100 kmol/s is a good basis since it corresponds to 70 mol% CH 4 plus 30 mol% C2 H6 .
On this basis there will be 70 kmol/s CH4 and 30 kmol/s C2 H6 of fuel.

CH4 + 2O2 → CO2 + 2H2 O ..........(RXN 1)

2C2 H6 + 7O2 → 4CO2 + 6H2 O..........(RXN 1)

Tabulate the information starting with what is given and moving through to what is asked
for - remember the reaction goes to completion (irreversible), so all the fuel will be
consumed but, since air is in "excess", some O2 will be left and N2 is inert.
There must be zero (kmol/s) H2 O and CO2 at the "inlet" and zero CH 4 and C2 H6 at the
"outlet". Wherever there is zero mole flow there must also be zero mass flow (kg/s).
Next enter in molar mass of each species and what is given (this depends on the basis)
in this case 70 kmol/s CH4 and 30 kmol/s C2 H6 at inlet - enter this into table.
Proceed systematically filling out all the "inlet" mole flows first, then all the "outlet" mole
flows, finally convert from moles to mass (inlet & outlet) using molar masses.

Component Inlet Molar Inlet Reacting Outlet Outlet


Mole Flow Mass Mass Flow (kmol/s) Mole Flow Mass Flow
(kmol/s) (kg/kmol) (kg/s) (kmol/s) (kg/s)
CH4 70 16 1,120 -70 0 0
C2 H6 30 30 900 -30 0 0
O2 269.5 32 8,624 -245 24.5 784
N2 1,013.8 28 28,386.4 0 1013.8 28,386.4
H2 O 0 18 0 +230 230 4,140
CO2 0 44 0 +130 130 5,720
Σ 1,383.3 - 39,030.4 +15 1,398.3 39,030.4

Start with "inlet" mole flow of oxygen and - notice how the stoichiometric coefficients act
simple as ratios:

• Theoretical mole flow of O 2 needed = (2/1 x 70) + (7/2 x 30) = 245 kmol/s.

• This is stoichiometric case, actual O2 needed = 245 x 1.1 = 269.5 kmol/s.

©H ERIOT-WATT U NIVERSITY
52 TOPIC 2. ENGINEERING CHEMISTRY

• Inert N2 entering with this O2 as "air" = 269.5 x 79/21 = 1013.8 kmol/s

Fill these values into table then proceed to "outlet" mole flow. All the N 2 is inert thus it
passes straight through to outlet. Excess O2 left at outlet = 269.5 - 245 = 24.5 kmol/s.
Finish with "outlet" moles of H2 O and CO2 - notice once again how the stoichiometric
coefficients act simple as ratios. Use this approach for all chemical reactions:

• Theoretical mole flow of H 2 O at "outlet" = (2/1 x 70) + (6/2 x 30) = 230 kmol/s.

• Theoretical mole flow of CO 2 at "outlet" = (1/1 x 70) + (4/2 x 30) = 130 kmol/s.

Complete the table by simply converting from inlet/outlet moles to inlet/outlet mass; just
multiply each component mole flow by its molar mass. Final totals as follows:
Inlet Inlet Outlet Outlet
Reacting
Mole Flow Mass Flow Mole Flow Mass Flow
(kmol/s)
(kmol/s) (kg/s) (kmol/s) (kg/s)
TOTAL 1,383.3 39,030.4 +15 1,398.3 39,030.4

Notice overall mass entering/leaving combustion chamber = 39.034.4 kg/s - thus, mass
is conserved.
Inlet (kmol/s) is not equal to outlet (kmol/s) - thus moles are not conserved. However,
they can be balanced as follows 1,383.3 +15 = 1,398.3 kmol/s.
For convenience the basis chosen for this calculation was 100 kmol/s of gas being
burned. However, in the problem statement it was specified that only 2 kg/s of gas
mixture was being burned - thus the calculation must be scaled to this value.
From table 100 kmol/s of CH 4 plus C2 H6 is equivalent to 1,120 + 900 = 2,020 kg/s mass
flow. Thus, to scale down to 2 kg/s, the scale factor = (2/2020) = 0.00099. Remember,
the question asks for mass flow air (kg/s) and composition (mass%).
Thus, delete all unnecessary columns from the table and check that overall "inlet" mass
flow and overall "outlet" mass flow both balance at 38.65 kg/s - thus mass is conserved
as it should be.
Component Inlet Outlet Mass
Mass Flow Mass (%)
(kg/s) Flow
(kg/s)
CH4 1.11 0 0
C2 H6 0.89 0 0
O2 8.54 0.78 2.02
N2 28.11 28.11 72.73
H2 O 0 4.10 10.61
CO2 0 5.66 14.64
Σ 38.65 38.65 100

©H ERIOT-WATT U NIVERSITY
TOPIC 2. ENGINEERING CHEMISTRY 53

Notice that the inlet flow of both CH 4 and C2 H6 = 1.11 + 0.89 = 2 kg/s as specified in
problem specification - this shows scaling produces the correct result.
The mass% is calculated for N2 = (28.11/38.65)x100 = 72.73%. Notice the inert nitrogen
makes up close to 75% of total - this is why it is important to control "excess" air, since
all this inert N2 must be heated up to combustion temperature!
Answer, mass flow rate of air required = 8.54 + 28.11 = 36.65 kg/s.
..........................................

©H ERIOT-WATT U NIVERSITY
54 TOPIC 2. ENGINEERING CHEMISTRY

2.11 Tutorial Topic 2


1. 147 g of H2 SO4 is mixed with 80 g of NaOH. Calculate the mass (g) of Na 2 SO4
produced and then calculate the amount of unreacted acid or alkali.
Hint, the reaction is irreversible, but one reactant is in "excess" so while one
reactant will be completely consumed some of the component in excess will be
left over.
If 200 kg of H2 SO4 are mixed with 100 kg of NaOH calculate the mass of Na 2 SO4
produced.

2. Boron trichloride (BCl3 ) can be reacted with silicon (Si) to produce silicon
tetrachloride (SiCl4 ) and liberate free boron (B). Write down a properly balanced
chemical equation for this reaction.
4 mole/hr of BCl3 is fed into a reactor where 85% of the BCl 3 is converted to SiCl4
calculate the mol flow (mol/hr) of each component leaving the reactor.

3. During a routine de-pressurisation of a gas storage system, 700 kg/hr of methane


and 200 kg/hr of butane are released into a flare system, where these two
components are completely burned using excess air. Given that reaction goes to
completion, calculate the mass flow (kg/hr) of water and carbon dioxide produced.
If accidentally insufficient air were fed to the flare system and, as a consequence,
only 300 kg/hr of methane and 100 kg/hr of butane were actually burnt calculate
the mass flow kg/hr of unburned methane and butane present in the product gas,
also express the answer as a percentage based on:

a) Actual gas mass flow supplied.


b) Mass of unreacted gas as percentage of reacted gas.

4. During the production of ammonia, hydrogen is produced from the steam reforming
of methane. In the reformer methane gas is reacted with excess water and the
resulting mixture is found to contain 12 kg of hydrogen and 15 kg water. Assuming
all the methane is reacted calculate the amount of water used.

5. 450 kg/hr of ethene and 280 kg/hr of oxygen are fed to a reactor in which ethylene
oxide is formed by the following reaction:
2C2 H4 + O2 → 2CH4O

a) Determine which reactant is "limiting".


b) Calculate the percentage excess of the "excess" reactant.

6. The following two simultaneous reactions take place:


A+B→C+D
C+B→E+D
And, if
A B C D E
Initial
10 20 0 0 0
kmoles
Final
1 3
kmoles

©H ERIOT-WATT U NIVERSITY
TOPIC 2. ENGINEERING CHEMISTRY 55

a) For the remaining three components determine the final amounts (kmol)
present.
b) Calculate the final composition of the mixture if "A" is fully converted.

7. With the fermentation of a particular grain, yeast converts glucose from plant
matter into ethanol and propenoic acid by the following reactions:
C6 H12 O6 + → 2C2 H5 OH + 2CO2
C6 H12 O6 + → 2C2 H3 CO2 H + 2H2 0
In a batch process, a tank is charged with 3500 kg of a 12% glucose-water solution.
After the fermentation process 120 kg of carbon dioxide was produced together
with 90 kg of unreacted glucose.

a) Calculate the quantities of ethanol, acid and water produced from the
process.
b) Calculate the total amount of water left in the tank.

8. 2700 kg/hr of an acidic waste effluent stream flows from a process into an in-line
mixer, where 1950 kg/hr of dilute alkali is added, before the neutralised effluent is
discharged into a nearby river. The acidic stream contains 20% by weight (w/w)
H2 SO4 , the alkali contains 8% (w/w) NaOH. Show by calculating the percentage
composition (mass %) of the effluent that all the acid has not been neutralised.

9. A gas mixture is analysed as follows:


Component Mass %
Carbon Dioxide 15
Carbon Monoxide 8
Methane 57
Hydrogen 20

a) Calculate the mole fraction of each component.


b) The mass % of each element in the mixture.

2.12 Bibliography
Perry, Robert H. and Green, Don W. 2008. Perry’s Chemical Engineers’ Handbook. 8th
ed. New York: McGraw-Hill.
Himmelblau, David M. and Riggs, James B. Basic Principles and Calculations in
Chemical Engineering. 2013. 8th ed. London: Pearson.
Felder, Richard M. and Rousseau, Ronald W. 2008. Elementary Principles of Chemical
Processes. 3rd ed. India: Wiley

©H ERIOT-WATT U NIVERSITY
56 TOPIC 2. ENGINEERING CHEMISTRY

©H ERIOT-WATT U NIVERSITY
57

Topic 3

Physical Properties and Gas Laws

Contents
3.1 Introduction . . . . . . . . . . . . . . . . . . . . . . . . . . . . . . . . . . . . . . 59
3.2 Density, Specific Volume and Specific Gravity . . . . . . . . . . . . . . . . . . . 60
3.3 Specific Heat Capacity . . . . . . . . . . . . . . . . . . . . . . . . . . . . . . . . 61
3.4 Mixture Specific Heat Capacity . . . . . . . . . . . . . . . . . . . . . . . . . . . 63
3.5 Sensible and Latent Heat Effects . . . . . . . . . . . . . . . . . . . . . . . . . . 65
3.6 Phase Diagrams and Gibbs Phase Rule . . . . . . . . . . . . . . . . . . . . . . 67
3.7 Equations of State . . . . . . . . . . . . . . . . . . . . . . . . . . . . . . . . . . 70
3.8 Partial Pressure . . . . . . . . . . . . . . . . . . . . . . . . . . . . . . . . . . . 73
3.8.1 Dalton’s Law of Partial Pressures . . . . . . . . . . . . . . . . . . . . . . 73
3.8.2 Amagat’s Law . . . . . . . . . . . . . . . . . . . . . . . . . . . . . . . . . 74
3.9 Vapour Pressure and Antoine Equation . . . . . . . . . . . . . . . . . . . . . . 76
3.9.1 Non-Condensable Gas . . . . . . . . . . . . . . . . . . . . . . . . . . . 77
3.10 Henry’s Law and Raoult’s Law . . . . . . . . . . . . . . . . . . . . . . . . . . . 79
3.10.1 Henry’s Law . . . . . . . . . . . . . . . . . . . . . . . . . . . . . . . . . 80
3.10.2 Raoult’s Law . . . . . . . . . . . . . . . . . . . . . . . . . . . . . . . . . 80
3.11 Tutorial Topic 3 . . . . . . . . . . . . . . . . . . . . . . . . . . . . . . . . . . . . 82
3.12 Bibliography . . . . . . . . . . . . . . . . . . . . . . . . . . . . . . . . . . . . . . 84

Prerequisite knowledge

• Basic understanding of physical chemistry

• Some familiarity with basic physical properties

• Basic understanding of calculus

• Familiar with the notation for summing a sequence of terms

Learning Objectives
By the end of this topic, you should be able to:

• Use physical properties to solve Chemical Engineering problems.

• Find physical properties of a mixture.


58 TOPIC 3. PHYSICAL PROPERTIES AND GAS LAWS

• Distinguish between sensible and latent heat effects.

• Relate Gibbs phase rule to phase diagrams.

• Solve problems using simple equations of state (EOS).

• Distinguish between partial and vapour pressure.

• Calculate average properties of mixtures.

• Understand and solve simple vapour-liquid equilibrium problems.

©H ERIOT-WATT U NIVERSITY
TOPIC 3. PHYSICAL PROPERTIES AND GAS LAWS 59

3.1 Introduction
As stated in Topic 1 there are seven fundamental SI units - these fundamental quantities,
their associated dimensions and SI units are listed below:

Quantity Dimension SI Unit


mass [M] kg (kilogram)
length [L] m (metre)
time [T] s (second)
temperature [θ] K (kelvin)
amount of substance [N] mol (mole)
electric current [I] A (ampere)
luminous intensity [J] cd (candela)

Only the first five are commonly used in Chemical Engineering. All derived SI units can
be broken down into products and quotients of the above raised to some power.
Process streams - be they gases, liquids or solids - consist either of a single component
or are multi-component mixtures. However, even mixtures act like a single component
and single values are assigned to their physical properties.
Some common Chemical Engineering physical properties, which will be discussed in
this section, are listed below. Notice how these properties have derived SI units that can
be traced back to combinations of the seven fundamental quantities listed above:

Physical Properties Symbol Fund-SI units


area A m2
volume V m3
density ρ kg/m3
specific volume ν m3 /kg
specific heat capacity CP J/kg K or (m2 /s2 K)

©H ERIOT-WATT U NIVERSITY
60 TOPIC 3. PHYSICAL PROPERTIES AND GAS LAWS

3.2 Density, Specific Volume and Specific Gravity


These quantities have already been discussed in Topic 1, however, their symbols, SI
units and dimensions are listed below:

Quantity Symbol Fund -SI Units Dimensions


density ρ kg/m3 [M][L]-3
specific volume ν m3 /kg [L]3 [M]-1

Use density to find the physical space needed to accommodate a certain mass of
material.
The density of any substance depends on temperature and pressure: for liquids the
pressure-dependence is weak; for gases the pressure-dependence is strong. The
density of both liquids and gases is strongly temperature-dependent.
If the temperature of a fixed mass of gas is kept constant, the density will fall as the
pressure is reduced - due to expansion of fixed mass of gas to a larger volume.
In Thermodynamic Tables (physical property tabulations) specific volume "ν" (m 3 /kg) is
usually listed. Specific volume is simply the reciprocal of density, so that

Volume Occupied (m 3 ) 1
ν= = .......(3.1)
Mass of Species (kg) ρ

Specific gravity "sg" is simply the ratio of the density of any substance to the density of
a well-known material (water is used for liquids, air is used for gases):

Density of Liquid (or Gas) @ Temp T


sgsubstance = .......(3.2)
Density of Water (or Air) @ Temp T

"sg" has no units - thus, the number is same for in any set of units. To recover the density
of a substance from (3.2), accurate density tables for water or air are needed. [Perry
and Green 2008]

©H ERIOT-WATT U NIVERSITY
TOPIC 3. PHYSICAL PROPERTIES AND GAS LAWS 61

3.3 Specific Heat Capacity


There are two specific heat capacities in common use. One is heat capacity of constant
pressure CP (kJ/kg K) where pressure is kept constant. The second is heat capacity of
constant volume CV (kJ/kg K) where volume is kept constant - each defined as follows:

! "
∂h
CP = .......(3.3)a
∂T P

! "
∂u
CV = .......(3.3)b
∂T V
Where,
h = specific enthalpy (kJ/kg).
u = specific internal energy (kJ/kg).
T = absolute temperature (K).

Specific enthalpy and specific internal energy are simply the energy content of a fluid,
that is (kJ) of energy stored in each (kg) of the fluid.
Specific enthalpy h (kJ/kg) is used mainly for energy stored in flowing fluids (also non-
flow fluids where pressure is constant) while specific internal energy u (kJ/kg) is mainly
used for other non-flow processes.
Later it will be shown that equations (3.3) may be integrated, combined with other energy
conservation expressions and applied to heat exchangers "Hx":

Hx (either Heater or Cooler) Appropriate Expression


Steady- flow Hx Q̇ = ṁCPavg (T2 − T1 ) (kW)
Non-flow Hx (constant pressure) Q= mCPavg (T2 − T1 ) (kJ)
Non-flow Hx (constant volume) Q= mCVavg (T2 − T1 ) (kJ)

Both CP and CV have units of (kJ/kg K).

©H ERIOT-WATT U NIVERSITY
62 TOPIC 3. PHYSICAL PROPERTIES AND GAS LAWS

Example : 3.3
Problem:
Calculate the rate at which temperature will rise when a 0.5 litre container of water is
heated in a non-flow device, at constant pressure, from 20 ◦ C using a heater rated at 0.5
kW. Hence calculate the time (min) it will take for the water to reach 90 ◦ C (take average
Cp = 4.2 kJ/kg K for water [Perry and Green 2008]).
Basis: 1 second of operation
Solution:
Calculate the mass of water in the container, density of water is about 1000 (kg/m 3 ).

m = ρ V = 1000 × 0.5 × 10−3

∴ m = 0.5 kg

The heater is rated at 0.5 kW or 0.5 kJ/s. Now take a basis of "1 second of operation"
the total heat output of the heater will be 0.5 kJ. Use the second equation listed in the
table on the previous page. For pure water "C Pavg = CP " and

Q = mCP (T2 − T1 )

With a basis of "1 second of operation" the rate of temperature rise is

Q 0.5
(T2 − T1 ) = =
mCP 0.5 × 4.2

∴ ∆T = 0.238 K/s

The temperature being calculated here is a temperature difference; therefore, (K/s) are
the same as (◦ C/s) - remember, there are 100 ◦ C and 100 K between ice-point and
boiling-point of pure water. The elapsed time can now be found as follows:

90 − 20
t=
0.238

∴ t = 294 s = 4.9 min


..........................................

©H ERIOT-WATT U NIVERSITY
TOPIC 3. PHYSICAL PROPERTIES AND GAS LAWS 63

3.4 Mixture Specific Heat Capacity


There are a variety of correlations used to predict C P , a correlation is simply a
mathematical function with adjustable constants. In this case C P is a function of
temperature T - one such correlation is given below:

d
CP = a + bT + cT 2 + ............(3.4)
T

The constants a, b, c, d depend on the chemical species and are listed in standard
textbooks [Perry and Green 2008] - the temperature T is usually (K). Notice, function is
non-linear.
Most streams in a plant will be mixtures and it should be possible to estimate properties
of a mixture from properties of the pure components. Calculation of mixture properties
is tackled in "Thermodynamics" using so-called "mixing rules".
Some properties are additive but often complex interactions can occur between different
components in the mixture - the properties of such mixtures are generally harder to
estimate and will be dealt with later in the course.
Each species in a mixture will be at the same temperature but each will store a different
amount of energy as "T " changes. For a three component mixture

(# $ # avg $ # avg $ )
Q= mCPavg 1
+ mC P 2
+ mC P 3
∆T

* +
∴ Q = mT otal x1 CPavg
1
+ x 2 C avg
P2 + x 3 C avg
P3 ∆T ............(3.5)

Where,
xi = mass fraction of component "i" in mixture (mi /mT otal )
CPavg
i
= average heat capacity of component "i" in mixture (kJ/kg K).

Beware, often xi is used for both mole fraction and weight fraction but sometimes w i is
used for mass fraction.

©H ERIOT-WATT U NIVERSITY
64 TOPIC 3. PHYSICAL PROPERTIES AND GAS LAWS

Example : 3.4
Problem:
Calculate the heating power required to raise the temperature of a 30 kg/hr mixture,
75 wt% water and 25 wt% ethanol, entering a heat exchanger at 20 ◦ C and leaving the
heat exchanger at 60 ◦ C (for water average heat capacity is Cp = 4.2 kJ/kg K, while for
ethanol, the average heat capacity is Cp = 4.8 kJ/kg K).
Basis: 30 kg/hr mixture entering the heat exchanger
Solution:
Equation (3.5) was written in terms of a non-flow device operating at constant pressure.
For a steady flow device Q (kJ) becomes Q̇ (kW) and mT otal (kg) becomes ṁT otal (kg/s).
Thus for steady-flow heat exchanger (3.5) becomes

* +
Q̇ = ṁT otal x1 CPavg
1
+ x C avg
2 P2 + x C avg
3 P3 ∆T ............(3.6)

The term inside the brackets is mass fraction weighted average heat capacity of mixture
"CPavg
mix
" which leads to (apply maths notation for the sum of a sequence - in reality
instead of three components there may be any number)

'
3
CPavg avg avg avg
mix = x1 CP 1 + x2 CP 2 + x3 CP 3 = xi CPavg
i ............(3.7)
i=1

And,

Q̇ = ṁT otal CPavg


mix ∆T ............(3.8)

For the two components specified in problem, equation (3.7) becomes

'
2
CPavg
mix = xi CPavg avg avg
i = x1 CP 1 + x2 CP 2 = (0.75 × 4.2) + (0.25 × 4.8)
i=1

∴ CPavg
mix = 4.35 (kJ/kg K)

And,

Q̇ = ṁT otal CPavg


mix ∆T = 30 × 4.35 × (60 − 20)

∴ Q̇ = 5220 (kJ/hr) or Q̇ = 1.45 (kW)

The term "heating power" was used in problem statement. In Chemical Engineering
terminology "heat load", or exchanger heat "duty" is used more often.
..........................................

©H ERIOT-WATT U NIVERSITY
TOPIC 3. PHYSICAL PROPERTIES AND GAS LAWS 65

3.5 Sensible and Latent Heat Effects


For continuous heating/cooling use (3.8) below. For non-flow (batch) heating/cooling at
constant pressure use (3.5) below. In both cases the material temperature is changing
as the material is being heated or cooled:

Q̇ = ṁT otal CPavg


mix ∆T ............(3.8)

Q = mT otal CPavg
mix ∆T ............(3.5)

Whenever the material temperature changes, as the it is being heated or cooled, then
the term sensible heat transfer is used. Notice, the material must either remain solid,
liquid or gaseous throughout - there can be no phase change.

• For sensible heat transfer the energy being transferred into the material has the
effect of increasing its temperature.

For a phase change i.e. liquid to vapour energy is absorbed. It is needed to move
molecules apart, from the relatively closely separated liquid state, to the more distantly
separated vapour state:

• The temperature of the material will remain constant and this type of heat transfer
is called latent heat transfer - it varies depending on T & P .

• The energy added or released does not change the temperature; it only causes a
change in molecular separation.

Latent Heat Temperature @


Phase Change Value (kJ/kg) Material
Type 1 atm(◦ C)
Vaporisation Liquid-Vapour 2256.7 water 100
Fusion Liquid-Solid 333.6 ice 0
Sublimation Solid-Vapour 528 Carbon dioxide -75

At 1 atm pressure, the CP H2O for liquid water at 100 ◦ C is 4.22 (kJ/kg K) while for steam
at 100◦ C it is 2.01 (kJ/kg K) - notice the difference [Perry and Green 2008].

©H ERIOT-WATT U NIVERSITY
66 TOPIC 3. PHYSICAL PROPERTIES AND GAS LAWS

Example : 3.5
Problem:
Calculate how much energy must be added to water (per kg) at 20 ◦ C to raise 2 kg of
steam at 150◦ C. Assume that the operation is carried out at a constant pressure of 101
kN/m2 . Express the purely latent heat effect as a percentage of the total heat required
for the entire operation.
Basis: 2 kg of liquid water
Solution:
Take the following:
Average heat capacity of liquid water: 4.2 kJ/kg K
Average heat capacity of steam 1.93 kJ/kg K
Latent heat of vaporisation at 100 ◦ C 2256.9 kJ/kg
Energy to raise liquid water to boiling point = 2×4.2×80 = 672 kJ
Energy to boil the water (latent heat) = 2×2256.9 = 4,513.8 kJ
Energy to raise vapour to 150 ◦ C = 2×1.93×50 = 193 kJ
Total energy for entire operation = 5,378.8 kJ
Total energy per kg = 2,689.4 kJ/kg
% of total that is latent heat = 84%
..........................................

This example clearly shows that latent heat represents a very large part of the total
energy content in the vapour
It may be understood that raising steam, which is really very common throughout the
chemical industry, will be a costly operation.
For this reason energy efficiency is carefully monitored and minimised using such
techniques as metering, condensate recovery, minimising excess air requirements,
using efficient burner units and optimising water treatment and blowdown operations.

©H ERIOT-WATT U NIVERSITY
TOPIC 3. PHYSICAL PROPERTIES AND GAS LAWS 67

3.6 Phase Diagrams and Gibbs Phase Rule


Consider a typical T − v diagram for pure water. Points "1-2-3-4" all lie on an isobar (a
line of constant pressure, say 1 atm). Point "1" is liquid water at, say 20 ◦ C (subcooled
liquid). At point "2", 100◦ C, the water starts to boil (saturated liquid).
From "2" to "3" all the energy goes into boiling the water so there is no temperature
change in this region (this is called the two-phase region). Once point "3" (saturated
vapour) has been reached and all the liquid has boiled off - the water is now 100%
steam.
Heating beyond point "3" to point "4" the steam increases in temperature beyond 100 ◦ C
(superheated vapour). So, to the left of the two-phase curve "A-C-B" is liquid region, to
the right is vapour region and inside curve "A-C-B" is liquid-vapour region:

C Point C is the Critical Point where liquid and vapour


Isobar become indistinguishable. Denote the region above
“C” as supercritical fluid region.

Liq 4 A-C is the boundary between liquid and two-phase


region, it is called the Saturated Liquid curve;
on this curve liquid is about to boil
sat 2 3
Vap
B-C is the boundary between vapour and two-phase
Liq-Vap region, it is called the Saturated Vapour curve;
1 A B here vapour is about to become superheated

Heating from "1-2" or "3-4" involves sensible heat transfer - notice how temperature
changes along these section of the isobar. On the other hand, vaporisation from "2-3"
involves latent heat transfer - notice how T & P remain constant over this section.
Liquid and vapour properties behave differently - change in specific volume ν (m 3 /kg),
liquid phase, is much less (over same ∆T range) than change in ν within vapour phase.

©H ERIOT-WATT U NIVERSITY
68 TOPIC 3. PHYSICAL PROPERTIES AND GAS LAWS

In addition to T − v diagrams, P − T diagrams are also very useful. On this diagram


"TP" is the Triple Point and "C" is the Critical Point:

TP-C is the Boiling Point Curve and shows


how the boiling points vary with pressure.
LIQUID
TP-A is the Freezing Point Curve and shows
SOLID how the freezing points vary with pressure.

VAPOUR TP-B is the Sublimation Curve.

There is one phase in any area, two phase on the


lines and three phases in equilibrium at TP.

The Gibb’s phase rule relates F , the number of degrees of freedom to π, the number
of phases and N , the number of components in the system, as follows:

F = 2 − π + N ............(3.9)

Take a pure component, N = 1, and consider spaces between lines above where π = 1.
Thus, the number of degrees of freedom, F = 2. Now look at two further cases:

• On any of the lines on the above diagram (except TP), N = 1and π = 2. Thus, the
number of degrees of freedom, F = 1.

• At the Triple Point itself "TP", N = 1 and π = 3. Thus, the number of degrees of
freedom, F = 0 - the system is "invariant".

Degrees of freedom can also be written F = V − E, where V is number of independent


variables and E the number of independent equations. The phase rule variables are the
intensive variables T &P &zi . For a pure species variables are T &P , so V = 2:

• For one phase (any open area on diagram) there are no phase constraints so that
E = 0 and F = 2, which means both T &P must be fixed.

• For two phases (any lines on diagram except TP) there is one phase constraint so
that E = 1 and F = 1, which means either T or P must be fixed, not both.

• For three phases (the TP itself) there are two phase constraints so that E = 2 and
F = 0, which means T &P are fixed by two independent phase constraints (each
constraint take the general form µ αi (T, P ) = µβi (T, P )").

©H ERIOT-WATT U NIVERSITY
TOPIC 3. PHYSICAL PROPERTIES AND GAS LAWS 69

Example : 3.6
Problem:
Determine the number of chemical components, the number of phases and hence the
number of degrees of freedom for the following systems:

a) Steam and water.


b) Air which can be considered a mixture of oxygen and nitrogen only.
c) Ethanol and water mixed together in a single liquid phase.
d) Ethanol and water in vapour-liquid equilibrium.
e) A mixture of sand and salt.

Solution:
Use the Gibb’s phase rule which relates F , π and N as follows:

F =2−π+N

For a pure species (previous page) the variables are T &P . For a multi-component
system (above) the variables are T, P &Z1 , Z2 . . . ZN −1 where Zi is any composition.
Notice that only (N-1) of all Zi are independent - N is number of components:

Components Degrees of
Case Phases "π" Variables Fixed
"N " Freedom "F "
a 2 1 1 T or P
b 1 2 3 T, P plus zi
c 1 2 3 T, P plus zi
T or P plus xi or
d 2 2 2
yi
e 1 2 3 T, P plus zi

The Gibbs phase rule variables V are T, P &Z 1 , Z2 . . . ZN −1 which are all the intensive
properties of the system. If the species is pure then the variables are only T &P . There
are two ways of finding F , either F = 2 − π + N or F = V − E:

• V the number of variables also given by V = 2+ (N − 1) π, where the "2" are T &P
and other term are all of the independent mole fractions in each phase.
• Thus for pure species V is always 2 and restricted to T &P .
• E is number of phase constraints E = (π − 1)N . If N = 1 throughout; then for one
phase E = 0 (no phase constraints); for two phases E = 1 (one phase constraint);
and at TP there are three phases E = 2, (two phase constraints).

..........................................

©H ERIOT-WATT U NIVERSITY
70 TOPIC 3. PHYSICAL PROPERTIES AND GAS LAWS

3.7 Equations of State


Charts can be used to read of ν as a function of T &P , but they are not appropriate for
doing calculations on a computer. An equation is needed linking P − ν − T together,
these are called Equations of State - some apply to gases and others to liquids.
The simplest Equation of State (EOS) is the ideal gas law, which states:

P V = nRT ............(3.10)
Where,
P = absolute pressure (N/m2 ).
V = volume of gas (m3 ).
n = number of moles (kmol).
T = the absolute temperature (K).
R = Universal Gas Constant (8.314x10 3 J/kmol K).
Divide both sides of equation (3.10) by n the number of kmoles leads to

P v = RT ............(3.11)
Where,
ν = molar volume of gas (m3 /kmol).
Another EOS is "van der Waal EOS" (1873) - but now only of historical interest

RT a
P = − 2 ............(3.12)
(v − b) v

Notice the way ν is used here - it is not immediately apparent but this equation, after
some re-arrangement, is actually cubic in ν.
Where, a and b are parameters specific to each species: a accounts for non-zero
intermolecular forces; b accounts for non-zero molecular volume.
For any gas the parameters a and b are calculated by knowing the critical point:

27R2 TC 2
a= ............(3.13)
64PC

RTC
b= ............(3.14)
8PC
Where,
PC = critical pressure (N/m2 absolute).
TC = critical temperature (K).
This demonstrates the importance of the critical point for estimating physical properties;
this is also true for many other properties - to be discussed later.

©H ERIOT-WATT U NIVERSITY
TOPIC 3. PHYSICAL PROPERTIES AND GAS LAWS 71

Extensive tables of physical properties are available in the literature, including detailed
compilations of critical pressures and temperatures [Abbott, Smith and van Ness 2005].

Example : 3.7.1
Problem:
Calculate the molar volume of methane gas at a pressure of 5x10 5 N/m2 absolute (5 bar
absolute) and a temperature of 30 ◦ C (use the ideal-gas law) and take R = 8.314x10 3
(J/kmol K).
Solution:
Re-arrange equation (3.11) in terms of molar volume - be sure to use absolute units

RT 8.314 × 103 × (30 + 273)


v= =
P 5 × 105
∴ v = 5.04 (m3 /kmol)

As an exercise the student is asked to check that the units used on RHS of equation
(3.11) do simplify to (m3 /kmol) - this is always good practice.
(Hint: to carry out the above check you will have to remember 1 J = 1 N m)
..........................................

©H ERIOT-WATT U NIVERSITY
72 TOPIC 3. PHYSICAL PROPERTIES AND GAS LAWS

Example : 3.7.2
Problem:
Calculate the volume occupied by 1 kmole of an ideal gas at standard temperature and
pressure (use the ideal-gas law) and take R = 8.314x10 3 J/kmol K.
Take STP to be 1 atm and 0 ◦ C which is 101.325x10 3 N/m2 and 273.15 K.
Solution:
The problem can be solved either using equation (3.10) or (3.11) - take equation (3.10)
in this case

nRT 1 × 8.314 × 103 × 273.15


V = =
P 101.325 × 103
∴ V = 22.4 (m3 )

As before the student is asked to check that the units used on the RHS of equation (3.10)
do indeed simplify back to (m3 ). Of course if 1 kmol of any ideal-gas at STP occupies
22.4 m3 then 1 mol of any ideal-gas at STP must occupy 22.4 litres.
Consider when the ideal gas law can be applied to real gases. Take the ideal
gas assumptions first and then apply the "Kinetic Theory of Gases" to deduce the
circumstances under which real gases behave as ideal gases.
There are two basic ideal gas assumptions: first, the volume of the molecules is
assumed to be zero; second, the forces of attraction between the molecules are
assumed to be zero:

1. Volume: at low pressure, as P → 0, the volume of real gas molecules


becomes a small fraction of the volume that they occupy - thus at low pressure
this first assumption becomes more and more nearly true.

2. Intermolecular Forces: at high temperature, the real gas molecules move


fast and the action of inter-molecular forces will decrease. Thus, at higher
temperatures this second assumption becomes more and more nearly true.

In reality ideal T really depends on species, T /T C ≈ 2.5 is about right.


The a in van-der-Waals EOS accounts for non-zero forces of attraction, while the b
accounts for non-zero volume - hence, van der Waal can be used at higher P and lower
T than ideal gas law, while still retaining usable accuracy.
..........................................

©H ERIOT-WATT U NIVERSITY
TOPIC 3. PHYSICAL PROPERTIES AND GAS LAWS 73

3.8 Partial Pressure


The discussions around ideal gases so far have centred on a single pure component.
Now consider an ideal gas consisting of two components a&b. Take the total volume to
be VT , the total pressure to be PT and apply one of two laws to this system:

• Dalton’s Law of Partial Pressure.

• Amagat’s Law of Partial Volume.

3.8.1 Dalton’s Law of Partial Pressures


For a multi-component system (mixtures) one way to identify the composition, already
discussed, is to make use of mole fractions or mass fractions.
Yet another approach, for gases only, is to use "Partial Pressure" - defined as that
pressure a component would exert if it occupied the total volume V T alone.
So taking a and b to be two different ideal gas components - find the pressure that each
would exert, as an ideal gas, if they were to occupy the mixture volume by themselves;
these pressures are the partial pressures P̂a and P̂b - see below

na RT
P̂a =
VT
nb RT
P̂b =
VT

Add the two partial pressures together leads to

na RT na RT (na + nb ) RT
P̂a + P̂b = + =
VT VT VT
nT RT
∴ P̂a + P̂b = = PT
VT

That is, the sum of the partial pressure of each individual component P̂a + P̂b is equal to
the total system pressure PT .
Generalise the above result for any species and any number of components, where i is
the component of interest and n i the total number of the component in the mixture:

ni RT
P̂i = ............(3.15)
VT
And,

'
n
P̂i = PT ............(3.16)
i=1

©H ERIOT-WATT U NIVERSITY
74 TOPIC 3. PHYSICAL PROPERTIES AND GAS LAWS

Take species i, as the component of interest, and then divide the partial pressure of this
species by the total pressure to get

! "! "
P̂i ni RT VT ni
= = = yi
PT VT nT RT nT

Thus Dalton’s law becomes

P̂i = PT yi ............(3.17)

Which states that the partial pressure of any component is the total pressure times the
species mole fraction. Remember the convention in Chemical Engineering that y i refers
to a gas/vapour composition while x i refers to a liquid composition.

3.8.2 Amagat’s Law


Apply the same treatment to "volume" instead of "pressure". The "partial volume" is that
volume the component would occupy if subject to the total pressure P T alone:

ni RT
V̂i = ............(3.18)
PT

∴ V̂i = VT yi ............(3.19)

With Amagat’s Law the partial volume is the total volume times the mole fraction.

©H ERIOT-WATT U NIVERSITY
TOPIC 3. PHYSICAL PROPERTIES AND GAS LAWS 75

Example : 3.8
Problem:
The total pressure of a gas mixture is 0.1 MN/m 2 , the partial pressure of oxygen (1) is
measured at 0.03 MN/m 2 , nitrogen (2) at 0.065 MN/m 2 - the remaining gas is carbon
dioxide (3). Calculate the mol% composition of this mixture and the average molar mass
of the mixture.
Basis: 100 kmol of gas mixture
Solution:
For three components Dalton’s Law, equation (3.16), becomes:
'
3
P̂i = PT
i=1

Or,

P̂1 + P̂2 + P̂3 = PT


P̂3 = 0.1 − 0.03 − 0.065 (MN/m2 )
P̂3 = 0.005 (MN/m2 )

Thus, the partial pressure of carbon dioxide (3) in the gas mixture is 0.005 MN/m 2 . With
this information equation (3.17) may be used to work out the mole fractions.
Given the basis of 100 kmol of gas mixture, mole fractions can be converted into moles
(kmol) and then, using molar mass the mass (kg) of each component can be found as
follows - work from what is given to what is asked for in question:

Partial
Mole Fraction Molar Mass
Component Pressure Moles (kmol) Mass (kg)
(kg/kmol)
(MN/m2 )
O2 (1) 0.03 0.3 30 32 960
N2 (2) 0.065 0.65 65 28 1820
CO2 (3) 0.005 0.05 5 44 220
Totals 0.1 1.0 100 - 3000

The average molar mass of the mixture is then simply total mass of all components
divided by total kmoles of all components: thus, 3000 kg/100 kmol or 30 kg/kmol.
..........................................

©H ERIOT-WATT U NIVERSITY
76 TOPIC 3. PHYSICAL PROPERTIES AND GAS LAWS

3.9 Vapour Pressure and Antoine Equation


The vapour pressure (or saturated vapour pressure) is the pressure exerted by the
vapour in equilibrium with its own pure liquid. The vapour pressure depends only on
temperature, but varies from species to species. The symbols used are as follows:

P ∗ or P sat

For a pure species in a sealed container the P sat , at temperature T , is equal to the
partial pressure P̂ which is also equal to total pressure PT .
Remember that a liquid will boil if its vapour pressure is equal to the applied pressure.
The vapour pressure of water at 100 ◦ C is 1 atm, therefore, water boils in an open
container at 100◦ C, because its P sat at 100◦ C equals applied atmospheric pressure.
If it is necessary to find the pressure at which water would boil at a temperature of
120◦ C, then find P sat for water at 120◦ C - it is very close to 200 (kN/m2 absolute) which
is around twice the pressure exerted by the atmosphere at ground level:

• This would be the pressure P T inside a sealed container - the pressure recorded
on an absolute pressure indicator.

• The temperature at which any substance boils with a P sat of 1 atm is called the
Normal Boiling Point (NBP); this is to differentiate it from other boiling points at
different pressures - see boiling point (vapour pressure) curve below:

TP-C is the Boiling Point Curve and shows


how the boiling points vary with pressure.
LIQUID
TP-C is also the vapour pressure curve.
SOLID Vapour and liquid co-exist in equilibrium
at any point along this line.
VAPOUR
sat
Notice this line is not straight but if ln(P ) is
plotted versus 1/T then it is nearly straight.

©H ERIOT-WATT U NIVERSITY
TOPIC 3. PHYSICAL PROPERTIES AND GAS LAWS 77

The Antoine equation provides a good correlation for vapour pressure and shows that,
for any component, P sat is a function of temperature T only - note, there are more
accurate correlations, but the Antoine equation is widely used:

# $ B
ln P sat = A − ............(3.20)
T +C
Where,
P sat = vapour pressure (mm Hg absolute), but units vary.
T = absolute temperature (K), but units vary.
A, B, & C = constants that are specific to each species [Abbott, Smith and van Ness
2005] - sometimes the constant "C" can be ignored. Constants differ depending on
units used.

3.9.1 Non-Condensable Gas


In some cases, with two or more gases, one gas will not condense as T drops, but other
components in the vapour mixture may - a good example is water vapour in air:

• As the mixture is cooled at constant pressure, there will come a point when some
water may condense and change from vapour to liquid phase.

• However, over a moderate range of temperatures, the air will not condense. It is
said that the air is the non-condensable component in the mixture.

• A common example of this is with single-glazed windows on a cold night - water


may condense on the inside of the pane if its P̂ is high enough.

For the air/water system, the air has a particular P̂ , which really depends on number of
occupants in room and the size of the room:

• As the temperature falls during a cold night, the partial pressure P̂H2O of the water
stays constant, but P sat of the water vapour decreases.

• If the temperature drops low enough, P sat for water eventually becomes equal to
P̂H2O - the water vapour will then condense on the inside of the glass.

• So long as P̂H2O < P sat the component will remain vapour. Notice P̂H2O may be
lowered using a dehumidifier. The air of course cannot liquefy at this T &P .

©H ERIOT-WATT U NIVERSITY
78 TOPIC 3. PHYSICAL PROPERTIES AND GAS LAWS

Example : 3.9
Problem:
Use the Antoine equation to calculate the vapour pressure of water at 100 ◦ C. The
constants for water are A = 18.3036, B = 3816.44 and C = -46.13. It is important to
check the input and output units to be used with these constants - T must be in (K) and
vapour pressure P sat will have units of (mm Hg absolute).
Solution:
Take the Antoine equation, equation (3.20) and enter all the constants as given
# $ 3816.44
ln P sat = 18.3036 −
T − 46.13
# sat $ 3816.44
∴ ln P = 18.3036 − = 6.6332
373.15 − 46.13
∴ P sat = 759.9 mm Hg absolute

Converting these units (use conversion tables) from (mm Hg) to (N/m 2 ) leads to
! "
sat 133.32 N/m2
P = 759.9 (mm Hg absolute) ×
1 (mm Hg)

∴ P sat = 101.309 x 103 N/m2 or 101.309 kN/m2


..........................................

So the Antoine equation predicts the vapour pressure of water at 100 ◦ C to be 101.309
kN/m2 . Atmospheric pressure is usually taken as 101.325 kN/m 2 .
Since P sat for water at 100◦ C is equal (nearly) to the applied pressure (atmospheric
pressure) it shows that water will boil at 100 ◦ C under normal condition.
Thus, the Normal Boiling Point (NBP) of water is 100 ◦ C. It must be stressed again
that water will boil at both higher temperatures (pressure above 1 atm) and lower
temperatures (pressure below 1 atm), however, these boiling points are not the NBP.
Usually standard reference books will quote the NBP - the Antoine equation can be used
to find all the boiling points from Triple Point "TP" to critical point "C".

©H ERIOT-WATT U NIVERSITY
TOPIC 3. PHYSICAL PROPERTIES AND GAS LAWS 79

3.10 Henry’s Law and Raoult’s Law


So far only vapour-liquid equilibrium for pure species has been considered (pure liquid in
equilibrium with a non-condensable gas has also been discussed). Consider a partially-
filled water kettle below - the spout is open to atmosphere:

o 2
At 20 C the vapour pressure of water is 2.4 kN/m .
The total pressure is 101.3 kN/m2.
So the partial pressure of air is 98.9 kN/m2
Kettle and y water = 0.024.

o 2
Both phases at At 80 C the vapour pressure of water is 47.4 kN/m .
1 atm pressure The total pressure is still 101.3 kN/m2.
Temp “T” So the partial pressure of air drops to 53.9 kN/m2
and y water = 0.47.

These are conditions inside the kettle. Notice the P̂ of steam inside the kettle must equal
P sat at prevailing temperature T - air will continuously be displaced out of the kettle as
T rises. The student is asked to calculate what happens at 100 ◦ C.
The next question is how to deal with a situation where there are two components in a
liquid mixture and both components can vaporise - see boiling "pot still" below:

The boiling liquid-vapour phases must be at same "T & P"


and both components are now present in both phases.

Boiling ˆ " is no longer equal to " Pi ", since liquid phase is now a
"P
sat
i

“Pot Still” mixture.

However, Pˆi ∝ Pi and remember each component


sat

will have different Antoine constants.

If each component in the liquid has same composition, then the component with higher
P sat will have the higher P̂ and thus the larger vapour composition.
This leads to the concept of using temperature T as a tool to separate the individual
components from a mixture - the basis of distillation.
For a mixture, where two or more components split between vapour and liquid phases,
there are two laws that that describe the vapour-liquid equilibrium:

• Henry’s Law.

• Raoult’s Law.

©H ERIOT-WATT U NIVERSITY
80 TOPIC 3. PHYSICAL PROPERTIES AND GAS LAWS

3.10.1 Henry’s Law


Henry’s Law only applies only to a component in the liquid phase as x i → 0. This is
often the case when a slightly soluble gas dissolves in a liquid. A good example would
be "sparkling water" - i.e. the carbon dioxide/water system:

P̂i = xi Hi ............(3.21)
Where,

P̂i = partial pressure of component "i" in vapour phase (N/m 2 ).


xi = mole fraction of component "i" (solute) in dilute liquid phase.
Hi = Henry Law constant (N/m2 ) of solute component.

3.10.2 Raoult’s Law


Raoult’s Law only applies only to a component in the liquid phase as x i → 1; however,
some liquid mixtures (components that are very similar chemically, i.e. hydrocarbons)
obey this law fairly well throughout the composition range:

P̂i = xi Pisat ............(3.22)


Where,

P̂i = partial pressure of component "i" in vapour phase (N/m 2 ).

xi = mole fraction of component "i" in liquid phase.

Pisat = vapour pressure of component at prevailing temperature (N/m 2 ).

©H ERIOT-WATT U NIVERSITY
TOPIC 3. PHYSICAL PROPERTIES AND GAS LAWS 81

Example : 3.10
Problem:
A binary system (that is a system of only two components) acetonitrile (1) and
nitromethane (2) conforms closely to Raoult’s Law. The Antoine equations are given
below and the constants are consistent with temperature and vapour pressure units of
◦ C and kN/m2 respectively. If a liquid mixture, 50 mol% component (1) and 50 mol%

component (2), is at 75◦ C and if the two phases are in vapour-liquid equilibrium (the
mixture is boiling) calculate the following:

a) The partial pressure of each component in vapour phase.


b) The total pressure at which this mixture will boil (temperature 75 ◦ C).

c) The mole fraction of each species in the vapour phase.

Solution:
The answers to this question are given in outline form below

2945.47
ln P1sat = 14.2724 −
T + 224
2972.64
ln P2sat = 14.2043 −
T + 209
At 75◦ C:
P1sat = 83.21 (kN/m2 )
P2sat = 41.98 (kN/m2 )

a) P̂1 = 41.61 (kN/m2 )


P̂2 = 21.00 (kN/m2 )
b) PT = 62.61 (kN/m2 )
c) y1 = 0.665
y2 = 0.335

Students are required to check the vapour pressures, at 75 ◦ C for each component and
then use Raoult’s Law to calculate the partial pressures of each component shown
above.

• The total pressure should then be checked.

• Using this information the vapour phase mole fractions can then be verified.

(At this low total pressure of 62.61 kN/m2 , the vapour phase will behave as an ideal gas
and the molecules are so similar that the liquid behaves as an "ideal solution". When
using Raoult’s Law it is necessary that both of these assumptions are met).
..........................................

©H ERIOT-WATT U NIVERSITY
82 TOPIC 3. PHYSICAL PROPERTIES AND GAS LAWS

3.11 Tutorial Topic 3


1. A 30 m3 storage tank holds a solution with density 1050 kg/m 3 . Calculate the mass
of solution inside the tank.

2. 20 kg/s of water are fed into a storage tank with a capacity of 50 m 3 . Calculate
how long it will take to completely fill the tank.

3. The flow rate of liquid effluent from a chemical plant is measured to be 20 m 3 /hr
and has a density of 1000 kg/m 3 . After analysing the effluent it is found to contain
0.6 mole% Na2 SO4 and 6 mole% NaCl - the rest is water. Calculate the mass of
Na2 SO4 and the mass of NaCl discharged from the plant.

4. A small domestic heater is used to raise the temperature of 1.8 litres of water from
5 to 100◦ C determine the following:

a) The electrical energy needed, both (kJ) and (kWh), to complete this operation
(assume there is no change of phase, that heat losses can be neglected and
take the average specific heat capacity of water to be 4.2 kJ/kg K).
b) The power consumption of the heater (kW), given that the heating process
takes 4 minutes.
c) At 100◦ C the water will boil and, if 5% of the mass is allowed to escape
(as vapour) before turning the heater off, calculate the electrical energy (kJ)
needed to generate the vapour (take the latent heat of vaporisation to be
2256 kJ/kg).

5. Calculate the volume occupied by 10 kg of methane at a pressure of 200 kN/m 2 (2


bar) and at a temperature of 30 ◦ C. Calculate the new volume whenever:

a) The pressure doubles to 400 kN/m 2 (4 bar)


b) The temperature is reduced to 10 ◦ C.

6. A storage vessel of some 30 m 3 fixed capacity (not a "floating head” tank, where
vessel top can rise and fall) holds nitrogen gas at a temperature of 300 K and a
pressure of 2 x 105 N/m2 (200 kN/m2 ) calculate the following:

a) The mass of gas within the tank.


b) As a result of a fire outside the tank, the temperature inside rises to 400 K.

Determine whether the volume or the pressure of the gas will change and then
calculate the magnitude (size) of the change.

©H ERIOT-WATT U NIVERSITY
TOPIC 3. PHYSICAL PROPERTIES AND GAS LAWS 83

7. The maximum safe working pressure of a large spherical storage tank, 10 m


diameter, is 106 N/m2 (10 bar), at a temperature of 20 ◦ C. Calculate the maximum
mass of helium gas that can be safely stored in the tank (molar mass of helium is
4 kg/kmol).

8. The composition of air is taken to be 79 mole% nitrogen and 21 mole% oxygen.


Calculate the average molar mass of air. Also calculate the volume occupied by
20 kg air, at a temperature of 20 ◦ C and a pressure of 101.325 kN/m 2 (1 atm).

©H ERIOT-WATT U NIVERSITY
84 TOPIC 3. PHYSICAL PROPERTIES AND GAS LAWS

3.12 Bibliography
1. Perry, Robert H. and Green, Don W. 2008. Perry’s Chemical Engineers’
Handbook. 8th ed. New York: McGraw-Hill.

2. Abbott, Michael, Smith, J.M. and Van Ness, Hendrick. 2005. Introduction to
Chemical Engineering Thermodynamics. 7th ed. New York: McGraw-Hill.

©H ERIOT-WATT U NIVERSITY
85

Topic 4

Material Balance on Single-Stage


Systems

Contents
4.1 Introduction . . . . . . . . . . . . . . . . . . . . . . . . . . . . . . . . . . . . . . 87
4.2 Basic Principles . . . . . . . . . . . . . . . . . . . . . . . . . . . . . . . . . . . . 88
4.2.1 Steady-State Operation . . . . . . . . . . . . . . . . . . . . . . . . . . . 89
4.2.2 Unsteady-State Operation . . . . . . . . . . . . . . . . . . . . . . . . . . 89
4.3 General Balance Equation . . . . . . . . . . . . . . . . . . . . . . . . . . . . . . 90
4.3.1 Balances on Continuous Steady-State Processes . . . . . . . . . . . . 91
4.3.2 Balances on Batch Processes . . . . . . . . . . . . . . . . . . . . . . . 91
4.4 Non-Reacting Systems . . . . . . . . . . . . . . . . . . . . . . . . . . . . . . . 93
4.5 Combustion Reaction Material Balances . . . . . . . . . . . . . . . . . . . . . . 105
4.5.1 Combustion Reactions . . . . . . . . . . . . . . . . . . . . . . . . . . . . 106
4.5.2 Theoretical and Excess Air . . . . . . . . . . . . . . . . . . . . . . . . . 107
4.6 Extent of Reaction . . . . . . . . . . . . . . . . . . . . . . . . . . . . . . . . . . 118
4.7 Tutorial Topic 4 . . . . . . . . . . . . . . . . . . . . . . . . . . . . . . . . . . . . 121
4.8 Bibliography . . . . . . . . . . . . . . . . . . . . . . . . . . . . . . . . . . . . . . 123

Prerequisite knowledge

• Basic principle of mass conservation.

• Elementary algebra and arithmetic.

• Some familiarity with combustion reactions.

• Fundamental understanding of how to balance a chemical reaction.

• Basic understanding of reaction stoichiometry.

Learning Objectives
By the end of this topic, you should be able to:

• Carry out material balance on simple single-stage non-reacting process.

• Carry out material balances on simple single-stage reacting process.


86 TOPIC 4. MATERIAL BALANCE ON SINGLE-STAGE SYSTEMS

• Distinguish between differential and integral material balances.

• Carry out detailed calculations on combustion processes - solid and gaseous fuels.

• Develop systematic problem-solving skills.

• Identify independent equations and independent variables.

• Identify tie-components and develop efficient solution strategies.

• Understand how "the extent of reaction" can be applied.

• Handle multiple reactions and apply fractional conversion.

©H ERIOT-WATT U NIVERSITY
TOPIC 4. MATERIAL BALANCE ON SINGLE-STAGE SYSTEMS 87

4.1 Introduction
Material Balances are fundamental to chemical engineers: initially, as a student, at a
theoretical level; and later, as a practising engineer, at an operational level.
Material balances simply involve the application of the mass conservation principle,
together with the laws of chemical combination, and then applying them to chemical
transformation, mixing and purification processes - all carried out on an industrial-scale.
Material balances are a simple but powerful tool which help an engineer to understand
and analyse complex processing systems:

• "Material and Energy Balances" are a vital first step in the design of a complete
process as well as individual items of equipment.

• Once in operation, "Material and Energy Balances" help the engineer to


troubleshoot problems, optimise production and maximise plant efficiency.

This topic will concentrate on developing student problem solving skills, without any
complex Mathematics, Chemistry or Physics:

• A key requirement is to first understand each problem and then develop a


systematic approach to finding a solution.

• The final step is to implement the solution, both efficiently and accurately, so as to
obtain the correct answer.

• This involves no more than the application of simple arithmetic and basic algebra.

The worked examples provided in this section should not be regarded as "recipes". The
tutorial and exam questions may need to be tackled differently. However, the basic
principles will be the same - so concentrate on basic principles.
It is better to use the term "Material Balance" than "Mass Balance" because quantities
may often have to be expressed in molar or volumetric units - rather than mass units.
This topic starts with "Single-Stage" systems, such as reactors, dryers, filters and
distillation columns. The next topic then moves on to "Multi-Stage" flowsheets with
recycle streams - later in the course, complete plant flowsheets will be tackled.

©H ERIOT-WATT U NIVERSITY
88 TOPIC 4. MATERIAL BALANCE ON SINGLE-STAGE SYSTEMS

4.2 Basic Principles


The type of process being investigated will determine the particular form of the Material
Balance equations - these different process categories are as follows:

• The process may be batch, continuous or semi-batch and may either operate
under steady-state or transient conditions.

In the case of batch processes materials are first charged to a closed system, then left
until the physical or chemical change has taken place. Finally, the material is discharged
and the equipment is then prepared for the next batch - i.e. brewing and fine chemical
plants.
With continuous processes inputs and outputs enter and leave the processing unit
continuously - i.e. oil refining, gas production and most bulk chemical plants:

"Batch" "Continuous"

A semi-batch process, sometimes called a semi-continuous process, involves elements


of both batch and continuous plants:

• A batch reaction in a liquid mixture may produce both a gaseous and a liquid
product - the gas may be continuously removed from the batch liquid.

Batch processing is often used when different products, each in small production runs,
are to be hygienically produced - e.g. pharmaceuticals, cooked meats, and cheeses.
Continuous processing is often used when a plant is designed to produce the same
product over a long period of time - e.g. oil and gas, ethylene, and fibre.

©H ERIOT-WATT U NIVERSITY
TOPIC 4. MATERIAL BALANCE ON SINGLE-STAGE SYSTEMS 89

4.2.1 Steady-State Operation


A "steady-state" process is one where all conditions (temperatures, pressures, flows
and compositions), at any location within the process, remain constant with respect to
time (except minor fluctuations) - the conditions may change with position, but not time:

Process

Conditions

Time

4.2.2 Unsteady-State Operation


A "transient" or unsteady-state" process is one where the conditions within the batch
are continuously changing with time. Batch and semi-batch processes are intrinsically
transient operations. More details are available in general textbooks [Felder and
Rousseau, 2008].

Process
Conditions

Time

Continuous processes generally operate under steady-state conditions. However,


during start-up and shut-down - and following significant operating disturbances - they
will also exhibit transient behaviour.

©H ERIOT-WATT U NIVERSITY
90 TOPIC 4. MATERIAL BALANCE ON SINGLE-STAGE SYSTEMS

4.3 General Balance Equation


The diagram below represents any generalised processing unit:

Process Output
O
Input
Unit S
Streams
Streams

From a material balance point of view consider what can happen to any component or
species as it passes through such a process - the mass flow of any component can be
accounted for as follows:

a) It may enter the system in one or more of the input streams "INPUT".

b) It may exit the system in one or more of the output streams "OUTPUT".

c) It may be formed by reaction/reactions within the process "GENERATION".

d) It can be used up by reaction/reactions within the process "CONSUMPTION".

e) The amount of the species within the system may change, "ACCUMULATION"; be
aware that this may be a positive or a negative term.

A General Balance Equation can now be written for any component as follows:

Input + Generation = Output + Consumptio n + Accumulati on .…(A)

Enters Produced Leaves Consumed Build-up

Through Within Through Within Within

System System System System System

Boundary Boundary Boundary Boundary Boundary

©H ERIOT-WATT U NIVERSITY
TOPIC 4. MATERIAL BALANCE ON SINGLE-STAGE SYSTEMS 91

4.3.1 Balances on Continuous Steady-State Processes


For a continuous process operating under steady-state conditions, there can be no
"accumulation" of any species - notice, the conditions at any point within the process
would not be constant if a species were accumulating or depleting.
Setting the accumulation term to zero the general material balance, for a continuous
process where a chemical reaction can occur, becomes:

Input + Generation = Output + Consumption .................(B)

For a continuous process where no chemical reaction occurs:

Input = Output .................(C)

4.3.2 Balances on Batch Processes


In the case of batch operation there is no flow, neither flow into nor flow out of the
process, so the general balance equation reduces to

Generation = Consumption + Accumulation .................(D1)


Or

Accumulation = Generation - Consumption .................(D2)

In a batch process all of these terms will vary with time and each of the above terms
should really be expressed as follows: rate of accumulation; rate of generation; rate of
consumption.
Such an approach is called a "differential balance" - it is based on the instantaneous
rate at which a species is being generated or consumed.
However, if the changes take place in the system between two points in time (often the
start and finish of the batch process) the accumulation may be expressed as:

Accumulation = Final Output - Initial Input .................(D3)

Essentially the differential balance has been integrated over time to give an
"integral balance". Now substitute (D3) into (D1) - the new integral balance can be
written as

Initial Input + Generation = Consumption + Final Output.................(D4)

Comparing (D4) with equation (B), for a continuous system, shows there is very little
difference between solving balances for continuous (steady-state) systems and solving
integral balances for batch systems.

©H ERIOT-WATT U NIVERSITY
92 TOPIC 4. MATERIAL BALANCE ON SINGLE-STAGE SYSTEMS

The only real difference is that the units for each of the terms in equation (B), continuous
processes, will be (kg/s, kmol/s or tonne/hr) while the units for each of the terms in
equation (D4), integral batch processes, will be (kg, kmol or tonne).
The integral material balance only links initial and final material inventories of a batch
system (both overall material balance and material balances for individual species);
there is no information on how inventories are changing at any given moment.
To investigate these transient effects the rules of calculus would need to be applied to
differential balances - this will not be covered in this topic, see later modules.
To summarise: for continuous processes use equation (A) for reacting systems and
equation (B) for non-reacting systems.
For any batch process use the integral balance, equation (D4) whenever a reaction takes
place.
The same distinctions between differential/integral balances and transient/continuous
processes also apply to semi-batch processes.

©H ERIOT-WATT U NIVERSITY
TOPIC 4. MATERIAL BALANCE ON SINGLE-STAGE SYSTEMS 93

4.4 Non-Reacting Systems


Example : 4.4.1
Problem:
A continuous evaporator is required to concentrate 20 tonnes/hr of aqueous 10% (w/w)
NaOH solution to 50% (w/w). Calculate the amount of water vapour that must be
removed per hour.
Basis: 20 t/hr feed entering the evaporator
Solution:
A necessary first step is to produce a block diagram for the process. In this case the feed
solution F is to be separated into a water vapour stream V (no NaOH in this stream)
and the concentrate stream C (all the NaOH from the feed ends up in this stream):

WATER

VAPOUR
VA

V t/hr
FEED
Evaporator
F = 20 t/hr

10% (w/w) NaOH

CONCENTRATE

C t/hr

50% (w/w) NaOH

The diagram should include the following: a block representing the process itself; all inlet
streams; all outlet streams. The streams should be labelled and all other information
attached to either the streams or the process unit - see above.
The "basis" for the calculation should be clearly stated. The logical approach is to base
the calculation on "20 t/hr feed" entering the system. The calculated vapour stream flow
V will then be found "per hour" (t/hr) - thus, the answer need not be scaled.
This "basis" should be maintained throughout the calculation writing all the terms in the
balance equations in terms of (tonnes/hr). This is a continuous processing unit without
any chemical reaction - thus, the material balance that applies is as follows:

Input = Output .................(C)

©H ERIOT-WATT U NIVERSITY
94 TOPIC 4. MATERIAL BALANCE ON SINGLE-STAGE SYSTEMS

Proceed systematically by writing all the material balance equations: F is feed flow (t/hr),
V and C are vapour and concentrate flows (t/hr) respectively - take NaOH as component
(1) and H2 O as component (2):
Write down the overall material balance for a general continuous plant with no chemical
reactions - equation (C):

Input = Output ...........(tonne/hr)

Or, in terms of this evaporator

F = V + C .................(i)

Write down the individual component material balances for NaOH (1) and H 2 O (2) as
follows:

F xF 1 = CxC1 .................(ii)

F xF 2 = V + CxC2 .................(iii)

Not all components appear in both outlet streams - NaOH (1) is non-volatile and does
not appear in vapour. Thus vapour must be pure water with mole fraction equal to one.
NaOH is called the tie-component - it is the key to solving this problem.
It should also be realised that only two of the above equations are independent - check
this by adding equation (ii) and equation (iii) - remember mass fraction sum to one:

F (xF 1 + xF 2 ) = V + C (xC1 + xC2 )

∴F = V +C

{shows that (ii) and (iii) sum to (i)}

Thus, there are only two independent equations and any two equations may be used.
Looking at the variables in these equations, and remembering that mole fractions sum
to one, it follows that only C and V (t/hr) are unknown.
Therefore, there are two independent equations, two unknown variables and a solution
must be possible. NaOH is the "tie-component" - that is the key to solving this problem.
NaOH ties streams F and C together.

©H ERIOT-WATT U NIVERSITY
TOPIC 4. MATERIAL BALANCE ON SINGLE-STAGE SYSTEMS 95

Since NaOH is a tie-component a solution is easily obtained by solving equation (ii) first

F xF 1 20 × 0.1
C= = = 4 (t/hr)
xC1 0.5

Followed by equation (i)

V = F − C = 20 − 4 = 16 (t/hr)

This is the answer requested, i.e. ANS 16 tonnes of water vapour must be removed
per hour. This problem was solved using equations (i) and (ii) - because (iii) is not
independent. Equation (iii) can now be used as a final check:

F xF 2 = 20 × 0.9 = 18 (t/hr)

V + CxC2 = 16 + (4 × 0.5) = 18 (t/hr) (Checks OK)

Often a problem can be solved using a table in which case the algebra is used intuitively.

1. Start with "given" information (problem statement):

IN OUT
Feed (F ) Vapour (V ) Concentrate (C)
Comp- Mass Flows Mass Flows Mass Flows
onent Fraction (t/hr) Fraction (t/hr) Fraction (t/hr)
NaOH (1) 0.1 0 0 0.5
H2 O (2) 0.9 1.0 0.5

Total 1.0 20 1.0 1.0

2. Calculate component flows into evaporator:

IN OUT
Feed (F ) Vapour (V ) Concentrate (C)
Comp- Mass Flows Mass Flows Mass Flows
onent Fraction (t/hr) Fraction (t/hr) Fraction (t/hr)
NaOH (1) 0.1 2 0 0 0.5
H2 O (2) 0.9 18 1.0 0.5

Total 1.0 20 1.0 1.0

©H ERIOT-WATT U NIVERSITY
96 TOPIC 4. MATERIAL BALANCE ON SINGLE-STAGE SYSTEMS

3. All the NaOH - 2 (t/hr) goes to concentrate stream and its mass fraction is 0.5, so
that the total stream flow must be 2/0.5 = 4 (t/hr) and water flow must be 2 (t/hr) by
difference:

IN OUT
Feed (F ) Vapour (V ) Concentrate (C)
Comp- Mass Flows Mass Flows Mass Flows
onent Fraction (t/hr) Fraction (t/hr) Fraction (t/hr)
NaOH (1) 0.1 2 0 0 0.5 2
H2 O (2) 0.9 18 1.0 0.5 2

Total 1.0 20 1.0 1.0 4

4. If 20 (t/hr) feed enters the evaporator and 4 (t/hr) total leaves in concentrate, it
follows that 16 (t/hr) total must leave in vapour - all of which must be water:

IN OUT
Feed (F ) Vapour (V ) Concentrate (C)
Comp- Mass Flows Mass Flows Mass Flows
onent Fraction (t/hr) Fraction (t/hr) Fraction (t/hr)
NaOH (1) 0.1 2 0 0 0.5 2
H2 O (2) 0.9 18 1.0 16 0.5 2

Total 1.0 20 1.0 16 1.0 4

The problem was solved the same way in both cases - first equation (ii) was solved
followed by equation (i). As a check, students should check that equation (iii), the non-
independent equation, is also satisfied.
Any two equations may be used to solve this problem.
..........................................

©H ERIOT-WATT U NIVERSITY
TOPIC 4. MATERIAL BALANCE ON SINGLE-STAGE SYSTEMS 97

Example : 4.4.2
Problem:
Raspberries contain 15 mass% solids and 85 mass% water. During the jam making
process sugar and pulped berries are mixed in a mass ratio of 55:45, the batch is heated
to its boiling point and evaporation continues until an end point condition is reached. The
end point condition is that the final jam mixture must only contain 33.3 mass% water.
Calculate the quantity (kg) of raw ingredients (berries and sugar) required to produce
the final cooled batch of 2,500 kg jam. Take it that only pure water leaves throughout
the evaporation process.
Basis: 2500 kg of jam.

Solution:
Since this is a batch process it is appropriate to show "before" and "after" diagrams:

BEFORE AFTER

SUGAR (3) WATER


WAT
A ER

VAPOUR

BERRIES
Jam
5% (w/
Solids (1) 15% w w)
(w/w)
Processing 2500 kg
Water (2) 85% (w/w)
Unit JAM
SUGAR/BERRY
33.3% (w/w)
RATIO = 55:45

[Felder and Rousseau, 2008]


For batch process where there is no chemical reaction - generation and consumption
terms are zero and integral balance equation (D4) reduces to the following:

Initial Input = Final Output

Next step is to write down all the algebraic equations. If B is mass of berries added to
batch (kg), S is mass of sugar added to batch (kg), V is mass of water vapour produced
(kg) and J is mass of jam produced (kg).

©H ERIOT-WATT U NIVERSITY
98 TOPIC 4. MATERIAL BALANCE ON SINGLE-STAGE SYSTEMS

The overall mass of jam is known to be J = 2500 (kg) - hence the overall integral material
balance becomes:

B + S = V + 2500 .................(i)

There are three basic components - solids (1), water (2) and sugar (3) - thus there will
be three component balances. Apply a balance to "solids" component (1), remembering
that there are no solids in the sugar S or the vapour V . Also, the berries are 0.15 mass%
solids, thus the component (1) balance becomes

0.15B = 2500 xJ1 .................(ii)

Water is labelled as component (2). There is no water in the sugar, the berries are 85
mass% water and the jam is 33.3 mass% water. Thus, component (2) balance becomes

0.85B = V + 0.333J

∴ 0.85B = V + 832.5.................(iii)

Sugar is labelled as component (3) - the berries are analysed as water and solids,
so there is no sugar in the berries and there is no sugar in the vapour V and sugar
ingredient itself is pure- thus component (3) balance may be written:

S = 2500 xJ3 .................(iv)

S
Another constraint is that the = 55/45 - thus S = 1.22 B. Finally not all the "jam" mole
B
fractions are independent - they must sum to one. Thus, the four equations with the
additional mole fraction constraint (immediately below) are as follows:

xJ1 + xJ2 + xJ3 = 1

∴ xJ3 = 1 − 0.333 − xJ1

B + 1.22B = V + 2500.................(i)

0.15B = 2500 xJ1 .................(ii)

∴ 0.85B = V + 832.5.................(iii)

1.22B = 2500 (1 − 0.333 − xJ1 ).................(iv)

©H ERIOT-WATT U NIVERSITY
TOPIC 4. MATERIAL BALANCE ON SINGLE-STAGE SYSTEMS 99

Notice there are only three independent equations and three unknowns - there are
various solution strategies, one would be to solve (i) and (iii) simultaneously for B and
V and then equation (ii) for the final unknown x J1 doing it this way leads to

B = 1217 .2 kg (Solids = 182.6kg & Water = 1034.6kg)

V = 202 .1 kg

xJ1 = 0.073
Also,

S = 1.22B = 1485 kg

It is a good idea to fill out an "initial input" and a "final output" table to check all the results
both vertically and horizontally.

INITIAL INPUT Ingredients FINAL OUTPUT Products


Berries Sugar Jam Water Vapour
Component
(kg) (kg) (kg) (kg)
Solids (1) 182.6 - 182.6 -
Water (2) 1034.6 - 832.5 202.1
Sugar (3) - 1485 1485 -

Sub-Total 1217.2 1485 2500.1 202.1


Total 2702.2 2702.2

Equation (iv) can now be used as a final check, remember this is the non-independent
equation:

1.22B = 1485 kg sugar

2500 (1 − 0.333 − xJ1 ) = 1485 kg sugar (Checks OK)

Sometimes it is possible to solve the problem using only the table, but where equations
need to be solved simultaneously then the algebraic solution is safer. The answer is that
1217.2 kg of berries need to be charged with 1485 kg of sugar to make 2500 kg jam.
..........................................

©H ERIOT-WATT U NIVERSITY
100 TOPIC 4. MATERIAL BALANCE ON SINGLE-STAGE SYSTEMS

Example : 4.4.3
Problem:
The feed to a continuous distillation column comprises 50 mole% butane, 30 mol%
pentane and 20 mol% hexane. The distillate (top product) contains 90 mol% pentane
and 1 mol% hexane. The bottom stream contains 42 mol% hexane. Find the
composition of the two product streams and kmoles of each component in each product
stream.
Basis: 100 kmol of feed entering the distillation column.
Solution:
A general strategy should emerge from the last example: choose basis; draw a block
diagram; label the processing unit and all streams; fill all data into diagram; write down
all material balances; identify unknown quantities; solve the equations.
The mole% of each component in the feed stream is specified - thus "100 kmol of feed"
is a convenient basis, since this equates immediately to component kmoles in the feed.
The next step is to draw a block diagram and label all the streams (feed F , distillate D,
bottoms B), then enter all the data specified in the problem:

90%
9 butane

9% pentane
F (100 kmols)
1% hexane
50% butane (1)

30% pentane (2) B

20% hexane (3) 42 % hexane

There will be one overall and three component material balances - that is four in total, but
only three of them will be independent. For continuous steady-state operation, without
any chemical reactions,

Input = Output ...........(kmol)

Kmoles are also conserved (as well as mass) so long as there are no chemical reactions.

©H ERIOT-WATT U NIVERSITY
TOPIC 4. MATERIAL BALANCE ON SINGLE-STAGE SYSTEMS 101

Next step is to write down all the material balances as specified by the problem
statement. It is a good idea to number the components, say butane (1), pentane (2) and
hexane (3). The first equation below is overall and the rest are component balances:
Overall:

F = D + B.................(i)

Butane (1):

F xF 1 = DxD1 + BxB1 ................(ii)

Pentane (2):

F xF 2 = DxD2 + BxB2 .................(iii)

Hexane (3):

F xF 3 = DxD3 + BxB3 .................(iv)

Now fill in all the data given - the unknowns are D, B and one of the two missing bottom
stream mole fractions:

xB1 + xB2 + xB3 = 1

∴ xB2 = 1 − 0.42 − xB1

Thus the four equations now are listed below - remember only three are independent
and of course there are only three unknowns:
Overall:

100 = D + B.................(i)

Butane (1):

50 = 0.9D + xB1 B................(ii)

Pentane (2):

30 = 0.09D + (1 − 0.42 − xB1 ) B.................(iii)

©H ERIOT-WATT U NIVERSITY
102 TOPIC 4. MATERIAL BALANCE ON SINGLE-STAGE SYSTEMS

Hexane (3):

20 = 0.01D + 0.42B.................(iv)

Examination of these equations shows that the simplest solution strategy is to solve (i)
and (iv) simultaneously for D and B, and then to use (ii) to get x B1 .
The pentane component balance equation (iii) can be used to provide an independent
check. Students should check that solving (i) and (iv) simultaneously gives

• D = 53.66 kmol
• B = 46.34 kmol

Students should then check that substituting these values back into equation (ii) gives
the butane mole fraction in the bottom stream, from which pentane mole fraction in
bottoms can be found from mole fraction summation, as follows:

• Butane composition in bottoms stream: x B1 = 0.037


• Pentane composition in bottoms stream:x B2 = 0.543

Students are asked to check the answer by showing that the LHS of equation (iii) is
equal to the RHS - kmoles of pentane leaving (LHS) should add up to 30 kmoles of
pentane entering (RHS).
Knowing the compositions of all streams and total kmoles of all streams, it follows that
kmoles of each component in each product stream may be found - check all remaining
answers summarised in table below:

IN (F ) OUT (D + B)
F Mole D Mole B Mole
Comp- F Flows D Flows B Flows
Fraction Fraction Fraction
onent (kmol) (kmol) (kmol)
- - -
Butane (1) 0.50 50 0.9 48.29 0.037 1.71
Pentane
0.30 30 0.09 4.83 0.543 25.17
(2)
Hexane (3) 0.20 20 0.01 0.54 0.42 19.46

Total 1.0 100 1.0 53.66 1.0 46.34

When no chemical reactions occur, as above, then kmoles are conserved between inlet
and outlet. If a chemical reaction occurs then kmoles "in" and kmoles "out" may be
different.
..........................................

©H ERIOT-WATT U NIVERSITY
TOPIC 4. MATERIAL BALANCE ON SINGLE-STAGE SYSTEMS 103

Example : 4.4.4
Problem:
The feed to a continuous distillation column comprises 50 mole% butane, 30 mol%
pentane and 20 mol% hexane. The distillate (top) product, which accounts for 55 mol%
of the feed contains 90 mol% butane. The bottom product contains 42 mol% hexane.
Find the mol% of the two product streams.
Basis: 100 kmol of feed entering the distillation column.
Solution:
In this variation of the problem, the specification of the hexane content (and by
difference, the pentane content) in the distillate has been replaced by another
specification - that of percentage of the feed recovered as distillate product.
Sometimes percentages are used to denote a mole or a mass fraction of a component
(composition) in a particular stream, other times they can denote the percentage of the
feed recovered in a particular outlet stream - take care to read the question carefully.
Write down the overall balance and the individual balances as before. Notice that the
D
55% "recovery" specification means ×100 = 55, while means that D = 55 kmol.
F
Insert the mole fraction of butane in distillate and the mole fraction of hexane in bottoms,
together with known components entering in feed - the result is the following four
equations. Remember only three of these are independent.
Overall:

100 = 55 + B.................(i)

Butane (1):

50 = (55 × 0.9) + BxB1 ................(ii)

Pentane (2):

30 = 55 (1 − 0.9 − xD3 ) + B (1 − 0.42 − xB1 ).................(iii)

Hexane (3):

20 = 55xD3 + 0.42B.................(iv)

Not all the mole fractions are independent so place these dependent mole fractions in
the pentane equation, which will only be used at the end for checking purposes.

©H ERIOT-WATT U NIVERSITY
104 TOPIC 4. MATERIAL BALANCE ON SINGLE-STAGE SYSTEMS

Notice the equations may be solved sequentially (not simultaneously) - solve them as
follows:

• Equation (i) B = 45 kmol

• Equation (ii) mole fraction butane in bottoms x B1 = 0.01

• Equation (iv) mole fraction of hexane in distillate x D3 = 0.02

• Check answers by solving LHS of equation (iii) - ANS should be 30 kmol.

The missing pentane mole fractions can be entered (mole fraction in distillate and
bottoms must sum to one). Knowing mole fractions and kmoles for each product stream,
kmoles of individual components can be found - see completed table below:

IN (F ) OUT (D + B)
F Mole D Mole B Mole
Comp- F Flows D Flows B Flows
Fraction Fraction Fraction
onent (kmol) (kmol) (kmol)
- - -
Butane (1) 0.50 50 0.9 49.5 0.01 0.5
Pentane
0.30 30 0.08 4.4 0.57 25.6
(2)
Hexane (3) 0.20 20 0.02 1.1 0.42 18.9

Total 1.0 100 1.0 55 1.0 45

Notice how the three independent equations (i), (ii) and (iv) are solved for the three
independent unknowns, B, x B1 and xD3 .
The pentane mole fractions are not independent - these dependent mole fractions can
be left in the dependent component balance equation and used as a final check.
When solving these problems it is instructive to first sketch out a block diagram, label
streams carefully and write down all the flows and/or compositions - also numbering
components is useful. Below is not a "recipe" but is a useful guide:

• Write down all the material balances and insert specified information into balances.

• Specify dependent compositions in the dependent balance, solve the independent


equations, use the dependent balance as a final check - tabulate everything.

..........................................

©H ERIOT-WATT U NIVERSITY
TOPIC 4. MATERIAL BALANCE ON SINGLE-STAGE SYSTEMS 105

4.5 Combustion Reaction Material Balances


When carrying out material balances on any reacting system the generation and
consumption terms must be included, not only for the component balances, but also
for the overall balance. Remember that mass must be conserved, but moles may not be
conserved.
More details are available in general textbooks [Himmelblau and Riggs, 2013]
Combustion involves a chemical reaction between fuel and oxygen; thus, for continuous
combustion processes, the overall balance equation is given by (B)

Input + Generation = Output + Consumption

While for batch combustion processes the overall balance equation is given by (D4)

Initial Input + Generation = Consumption + Final Output

Reaction stoichiometry was discussed in earlier topics - thus, it is now appropriate to


apply these findings to solve material balances involving combustion reactions.
Combustion reactions involve the rapid reaction of a fuel with oxygen (usually as air).
Such reactions are of great practical and economic significance - they are used for heat
generation, electric power generation and pollution abatement control.
The design of power generation equipment is usually the job of mechanical engineers.
However, chemical engineers are involved in the analysis of combustion reactions and
in the design of and operation of burners, especially "thermal oxidation" processes:

• Another reason for studying "combustion" is that nomenclature, conventions and


material balance techniques involving combustion tend to be a little different.

• A fuel comprises varying amounts of carbon, hydrogen and sulphur, either as


elements or as specific compounds.

• Carbon oxidises to CO2 (total combustion) or to CO (incomplete or partial


combustion).

• Hydrogen forms H2 O

• Sulphur forms SO2 - sometimes SO3

• NOx (various nitrogen oxides) can usually be ignored in a first-look material


balance.

©H ERIOT-WATT U NIVERSITY
106 TOPIC 4. MATERIAL BALANCE ON SINGLE-STAGE SYSTEMS

4.5.1 Combustion Reactions


Often the fuel composition is given as an "elemental analysis" - that is the list of possible
compounds in the fuel is so extensive it is better to simply analyse the elements. Notice
that, with the individual reactions below, atoms of fuel are being consumed:

C + O2 → CO2 (Complete combustion)

C + 1/2 O2 → CO (Partial combustion)

2H + 1/2 O2 → H2 O

S + O2 → SO2

For a pure hydrocarbon fuel such as propane (unlike a composite fuel such as coal or
fuel oil), a balanced chemical reaction can be written as follows:

C3 H8 + 5O2 → 3CO2 + 4H2 O (Complete combustion)

C3 H8 + 7/2O2 → 3CO + 4H2 O (Partial combustion)

Similarly for a sulphur compound, such as carbon disulphide, the reaction for complete
combustion would be as follows:

CS2 + 3O2 → CO2 + 2SO2 (Complete combustion)

Air is most commonly used as the source of oxygen. It is normally adequate to take the
composition of air as being 79 mole% N 2 and 21 mole% O 2 .
Occasionally 78 mole% N2 , 21 mole% O2 , 1 mole% Ar is used. Sometimes the water
content of the air, which depends on atmospheric conditions, is a factor.
It is often better to clear fractions from balanced chemical equations and convert the
fractions into integers.

©H ERIOT-WATT U NIVERSITY
TOPIC 4. MATERIAL BALANCE ON SINGLE-STAGE SYSTEMS 107

4.5.2 Theoretical and Excess Air


In general there will be four material streams associated with a combustion process:

Stack Gas
Air Combus!on (Flue Gas)
Process
Solid Residue
Fuel
(Ash)

The stack gas will contain - usually as water vapour - any inlet water, together with the
water produced from the combustion of the fuel’s hydrogen content.
Stack gas compositions are expressed either on a wet-basis (water vapour included) or
on a dry-basis (water vapour excluded) - be sure to read questions carefully.
In order to ensure complete combustion of the fuel, oxygen must be supplied in "excess".
That is, in excess of theoretical stoichiometric proportions:

• Too little air (too low an air/fuel ratio) and the flue gases will contain smoke, soot,
and dangerous carbon monoxide fumes.

• Too much air (too high an air/fuel ratio) will result in excessive fuel consumption
and higher running costs which can be significant.

The theoretical oxygen is the amount of oxygen (usually in molar units) required for
complete combustion of all the fuel present.

C → CO2

H → H2 O

S → SO2

Any O2 that might be present in the fuel must be deducted when calculating theoretical
O2 requirement; i.e. it is assumed that any O2 in the fuel will be available to take part in
the combustion, thereby reducing the amount of air needed.
Any partial combustion of "C" to "CO" does not enter into the calculation of theoretical
oxygen required - it is a theoretical calculation at this stage.
"Theoretical air" is the quantity of air that contains the "theoretical oxygen" - it is easy
to convert from theoretical oxygen (kmol) to theoretical air (kmol) as follows:

Theoretical air (kmol) = (100/21) x Theoretical oxygen (kmol)

©H ERIOT-WATT U NIVERSITY
108 TOPIC 4. MATERIAL BALANCE ON SINGLE-STAGE SYSTEMS

For any chemical reaction it is important to work in kmoles (or moles) since fuel is
consumed and flue gases are produced in fixed proportions in accordance with the
balanced reaction equation.
The excess oxygen (or air) is the amount of additional oxygen or air needed over and
above the theoretical amount required for stoichiometric combustion:

% excess oxygen (air) = [(excess oxygen or air/(theoretical oxygen or air)]×100

Complete combustion requires good contact between fuel and air. Gaseous fuels, which
mix readily with combustion air, have lower "excess air" requirement than liquid fuels.
Combustion of liquids is improved by atomising them through a nozzle.
Solid fuels tend to require the greatest excess air. However, combustion of solids may
be improved by using them in a finely powdered (pulverised) form. Typical excess air
would be in the range:

• 5-10% for gaseous fuels.

• 50% for solid fuels.

The number of compounds in gaseous fuels is limited so that the composition (either
mass% or mole%) is usually given in terms of these compounds.
Heavy fuel oil and coal are chemically so complex that the analysis will usually be
specified in terms of elements "C, H, N, O, S", "moisture" and "ash".

©H ERIOT-WATT U NIVERSITY
TOPIC 4. MATERIAL BALANCE ON SINGLE-STAGE SYSTEMS 109

Example : 4.5.1
Problem:
Coal, with the composition shown below, is burned with 40% excess air and, while 98%
of the carbon burns to CO 2 , the remaining 2% of the carbon burns to CO. Determine the
stack gas composition (mol%) on both a "wet" and a "dry" basis.

SubstanceMoisture Ash S H C N O
Mass% 2.9 5.4 0.8 4.3 83.2 1.3 2.1

Basis: 100 kg of coal


Solution:
A basis of 100 kg of coal is convenient since the analysis is in terms of mass% -
thus, change the elemental analysis from (mass%) to (kg). Mass must immediately
be changed to kmoles to facilitate combustion reaction calculations.
The first step is to determine the theoretical oxygen and air required for complete
combustion; remember "excess" air is defined in terms of this quantity. Once the actual
quantity of air is found, incomplete combustion can then be allowed for.
To apply the excess air constraint, theoretical oxygen for complete combustion is
needed:
C + O2 → CO2
2H + 1/2O2 → H2 O
S + O2 → SO2

Calculate the theoretical oxygen demand as follows:

Mass Molar Mass Kmoles Theoretical O2


Component
(kg) (kg/kmol) (kmol) (kmol)
Moisture 2.9 18 0.161 0
Ash 5.4 - - 0
S 0.8 32 0.025 0.025
H 4.3 1 4.3 1.075
C 83.2 12 6.933 6.933
N 1.3 14 0.093 0
O 2.1 16 0.131 -0.066
Total 100 - 11.643 7.967

Hence, the theoretical oxygen requirement for complete combustion after subtracting
the O2 present in the fuel (-0.066 kmol) is 7.967 kmol total.

©H ERIOT-WATT U NIVERSITY
110 TOPIC 4. MATERIAL BALANCE ON SINGLE-STAGE SYSTEMS

Now apply the excess air constraint, which is defined in terms of theoretical O 2 , required
for complete combustion to get actual O 2 , actual N2 and actual air as follows:

Calculate
Using To Get
the Convert To
Formula Result
following:
(kmol) (kg)
Actual O2
= 7.967×1.4 = 11.154 = 356.93
supply :
Actual N2
= 11.154×(79/21) = 41.960 = 1174.88
supply:
Actual air
= 11.154+41.96 = 53.114 = 1531.81
supply:

Now apply incomplete combustion constraints - first find out how much carbon is
completely combusted and how much is partially combusted:

• Kmoles of C reacted to CO 2 = 6.933 x 0.98 = 6.794 kmol

• Kmoles of C reacted to CO = 6.933 x 0.02 = 0.139 kmol

Now calculate the amount of each compound in the exit gases:

Component Kmoles (kmol) Mass (kg)


H2 O 0.161 + (4.3)/2 = 2.311 41.60
CO 0.139 3.89
CO2 6.794 298.94
SO2 0.025 1.60
N2 (0.093)/2 + 41.960 = 42.01 1176.28
11.154 + 0.066 - (0.025 +
O2 1.075 + 6.794 + (0.139)/2) = 104.22
3.257
Total (wet) 54.536 1626.53
Total (dry) 52.225 1584.93

The overall material balance is easy to check as follows, remember kmoles are not
conserved but mass is always conserved:

Calculate the
Using Numbers: To Get Result:
following:
Mass
= 100 + 1531.81 = 1632 kg (IN)
(fuel + air):
Mass
= 1626.53 + 5.4 = 1632 kg (OUT)
(gas + ash):

©H ERIOT-WATT U NIVERSITY
TOPIC 4. MATERIAL BALANCE ON SINGLE-STAGE SYSTEMS 111

As can be seen, rounding figures to the nearest kilogram, the total mass into this solid
fuel combustion process is equal to the total mass leaving the unit - this is always a very
good final check of overall accuracy.
The stack gas analyses, on both a wet and a dry basis, are now easily found from
previous information, as follows:

Component Mol% (wet) Mol% (dry)


H2 O 4.2 -
CO 0.3 0.3
CO2 12.5 13.0
SO2 0.1 0.1
N2 76.9 80.4
O2 6.0 6.2
Total 100 100

Alternatively, the above stack gas analysis could also have been calculated, both wet
and dry, on a mass% basis.
It should also be noticed that for this solid fuel situation, the excess air requirement is
40% and this translates to around 6 mol% in the flue gas mixture.
If an oxygen analyser were fitted to this stack then the reading from the analyser should
be around 6 mol% O 2 .
Do not confuse percentage excess air with percentage O 2 in flue gases.
..........................................

©H ERIOT-WATT U NIVERSITY
112 TOPIC 4. MATERIAL BALANCE ON SINGLE-STAGE SYSTEMS

Example : 4.5.2
Problem:
A gas mixture containing 80 mol% CH 4 and 20 mol% C 2 H6 is burned completely with
10% excess air. Calculate the amount of air (kg) required per (kg) of gas and the mass
composition of the wet product gas. If an oxygen analyser were sampling the flue gas,
calculate the expected reading in mass% wet-basis.
Basis: 100 kmol of gas mixture
Solution:
Choosing a basis of 100 kmol of gas mixture, provides 80 kmol CH 4 and 20 kmol C2 H6 .
For complete combustion the reactions are as follows:

CH4 + 2O2 → CO2 + 2H2 O


C2 H6 + 7/2O2 → 2CO2 + 3H2 O

The last equation could be mutiplied by two so that stoichiometric coefficients are
converted to integers. Calculate the theoretical oxygen demand as before:

Kmoles Molar Mass Mass Theoretical O2


Component
(kmol) (kg/kmol) (kg) (kmol)
CH4 80 16 1280 160
C2 H6 20 30 600 70
Total 100 - 1880 230

The problem states that only 10% excess air is needed:

Calculate
Using To Get
the Convert To
Formula Result
following:
(kmol) (kg)
Actual O2
= 230×1.1 = 253 = 8096
supply :
Actual N2 253×(79/21)
= = 951.8 = 26,650.4
supply:
Actual air 253 +
= = 1204.8 = 34,746.4
supply: 951.8

©H ERIOT-WATT U NIVERSITY
TOPIC 4. MATERIAL BALANCE ON SINGLE-STAGE SYSTEMS 113

34,746.4 kg of air is required to completely burn 1,880 kg of gas fuel.


∴ air/fuel Ratio = 34746.4/1880 = 18.5 (kg/kg)

Now calculate the amount of each compound in the exit gases and the composition of
the stack gas - composition in terms of mass% on a "wet" basis:

Molar
Inlet Inlet Outlet Outlet
Comp- Mass Reacting
Moles Mass Moles Mass Mass %
onent (kg/kmol) (kmol)
(kmol) (kg) (kmol) (kg)

CH4 80 16 1,280 -80 0 0 0


C2 H6 20 30 600 -20 0 0 0
O2 253 32 8096 -230 23 736 2.0
N2 951.8 28 26,650.4 0 951.8 26,650.4 72.8
H2 O 0 18 0 +220 220 3,960 10.8
CO2 0 44 0 +120 120 5,280 14.4
Total 1,304.8 - 36,626.4 +10 1,314.8 36,626.4 100

It will be noticed that mass is conserved, some 36,626.4 kg entering and leaving the
combustion process.
It will also be noticed that kmoles are not conserved but can still be balanced as follows:

• Some 1,304.8 kmoles of reactants enter the system.

• Some 1,314.8 kmoles of products leave the system.

• However, some +10 kmoles of all species are generated by the reaction.

• Thus, while kmoles are not conserved they can be accounted for.

If an oxygen analyser were fitted to the exhaust stack then, by this calculation, it should
read about 2 mass% O 2 on a wet-basis:

• It is interesting to note that 40% excess air converts to about 6 mol% O 2 (solid fuel
combustion system).

• And 10% excess converts to about 2 mass% (gaseous combustion system).

These results do indicate a correlation between excess air and percentage of O 2 in stack
gases, but show that detailed calculations are needed to relate one to the other.
..........................................

©H ERIOT-WATT U NIVERSITY
114 TOPIC 4. MATERIAL BALANCE ON SINGLE-STAGE SYSTEMS

Example : 4.5.3
Problem:
C3 H8 is burned in air to give a flue gas containing: 0 mol% C 3 H8 , 0.9 mole% CO and
8.1 mole% CO2 ; the remainder is made up of N 2 , O2 and H2 O vapour. Calculate the
following:

a) The full "wet-basis" analysis of the flue gas (mass %).


b) The % excess air employed.
c) The air/fuel ratio on a mass-basis (w/w).

Basis: 100 kmol of flue gas mixture


Solution:
Choosing a basis of 100 kmol of flue gas mixture ensures 0 kmol C 3 H8 , 0.9 kmol CO
and 8.1 kmol CO 2 are in flue gas mixture. The fact that there is 0 mol% C 3 H8 in flue gas
means that all of the propane has reacted - this points to excess air being present.
The fact that both CO and CO 2 are present in the flue gases means that complete
combustion occurs with some of the fuel and only partial combustion with the rest:

C3 H8 + 5O2 → 3CO2 + 4H2 O (Complete combustion)

C3 H8 + 7/2O2 → 3CO + 4H2 O (Partial combustion)

All the C3 H8 has reacted (0 mole% in flue gas) and a total of 0.9 kmol of CO and 8.1
kmol CO2 has been produced. If x is kmole C 3 H8 reacted and if y is fraction completely
reacted, then (1 − y) the fraction partially reacted and we can write

3
xy = 8.1.................(A)
1

3
x(1 − y) = 0.9.................(B)
1

∴ y = 0.9

(1 − y) = 0.1

Solving equation (A) for x after substituting for y leads to


x = 3 kmol

©H ERIOT-WATT U NIVERSITY
TOPIC 4. MATERIAL BALANCE ON SINGLE-STAGE SYSTEMS 115

Then check that this value of x and known value of (1 − y) produce the correct result in
equation (B).
The amount of water in the flue gas is simply four times the amount of propane reacted
(either reaction) - thus, water in flue gases must be 12 kmol.
The amount of O 2 reacted must also be known since all the fuel has been consumed,
either by complete or partial combustion:

• O2 consumed by complete combustion = 3×0.9×5/1 = 13.5 kmol

• O2 consumed by partial combustion = 3×0.1×3.5/1 = 1.05 kmol

• Total O2 consumed by both reactions = 13.5 + 1.05 = 14.55 kmol

Fill in the table with what is known at this stage and remember there is excess air so
kmoles of O2 are unknown, call this z. However, kmoles of N 2 are linked to kmoles of
O2 (through the inlet air).
The kmoles of H2 O, CO and CO2 produced are known, so that the "Reacting" column
can be filled to get +3.45 kmoles of total reacted - thus the table becomes:

Component Inlet Moles (kmol) Reacting (kmol) Outlet Moles (kmol)


C3 H8 3 -3 0
O2 z -14.55 (z-14.55)
N2 z(79/21) 0 z(79/21)
H2 O 0 +12 12
CO 0 +0.9 0.9
CO2 0 +8.1 8.1
Total 96.55 +3.45 100

This problem can now be solved by adding all the elements either in the first or the last
column - take the last column, since this is basis of the calculation:

(z − 14.55) + 79/21z + 12 + 0.9 + 8.1 = 100

∴ 100/21z = 93.55

∴ z = 19.65 (kmol)

©H ERIOT-WATT U NIVERSITY
116 TOPIC 4. MATERIAL BALANCE ON SINGLE-STAGE SYSTEMS

Knowing z fill in rest of the table as follows:

Inlet Moles Reacting Outlet Moles


Component
(kmol) (kmol) (kmol)
C3 H8 3 -3 0
O2 19.65 -14.55 5.1
N2 73.90 0 73.90
H2 O 0 +12 12
CO 0 +0.9 0.9
CO2 0 +8.1 8.1
Total 96.55 +3.45 100

From the above students must complete the full balance table below and remaining
calculations for all parts of the question:

Molar
Inlet Inlet Outlet Outlet
Comp- Mass Reacting
Moles Mass Moles Mass Mass %
onent (kg/kmol) (kmol)
(kmol) (kg) (kmol) (kg)

C3 H8 3 44 -3 0
O2 19.65 32 -14.55 5.1
N2 73.90 28 0 73.90
H2 O 0 18 +12 12
CO 0 28 +0.9 0.9
CO2 0 44 +8.1 8.1
Total 96.55 - +3.45 100

Total mass IN = kg and mass OUT = kg so material


balance is satisfied.
As stated before kmoles can also be accounted for as follows:
kmol entering
kmol formed overall (all species)
kmol leaving system

©H ERIOT-WATT U NIVERSITY
TOPIC 4. MATERIAL BALANCE ON SINGLE-STAGE SYSTEMS 117

a) Students to complete the mass% flue gas analysis, on a wet-basis (last column)

b) The theoretical O2 requirement (complete combustion) = kmol.


The actual O2 at inlet = kmol.
% excess air (% excess oxygen) = %.
(Could be a little on the high side for a gaseous fuel)

c) The fuel/air ratio on a mass-basis can be found from the above table.
Air entering = kg.
Fuel entering = kg.
Air/fuel ratio = (kg/kg) or (w/w) use either notation for mass ratio.

An excess air of 31% ties in with a 5.1 mol% O 2 concentration in flue gases.
..........................................

©H ERIOT-WATT U NIVERSITY
118 TOPIC 4. MATERIAL BALANCE ON SINGLE-STAGE SYSTEMS

4.6 Extent of Reaction


If the "amount of any species reacted" is known, divide this quantity by its stoichiometric
coefficient to get the extent of reaction ε (kmol) - always positive with units of kmoles:

• The extent of reaction varies from zero to the "amount of limiting reactant that
can react (kmol) - amount initially present" divided by its stoichiometric coefficient.

• The same value of ε(kmol) can then be used to find how much of each species
has reacted using the reverse procedure - multiply ε by the species coefficient.

• Thus, υi ε is number of kmoles of any species i that has reacted; υ i is positive for
products, negative for reactants and zero for inerts.

• If more than one reaction takes place each reaction will have its own ε (kmol).

Example : 4.6.1
Problem:
100 kmol methane is burned completely to form water and carbon dioxide. Calculate
the extent of reaction and the quantities of water vapour and carbon dioxide formed.
Basis: 100 kmol of methane fuel
Solution:
The balanced chemical reaction is given below - the extent of reaction is based on the
information that 100 kmol methane is burned completely:

CH4 + 2O2 → CO2 + 2H2 O

kmol CH4 Reacted −100 (kmol)


ε= = = 100 (kmol)
υCH4 −1

Water vapour formed:

(H2 O kmol)out = (H2 O kmol)in + υH2O ε = 0 + 2ε


(H2 O kmol)out = 200 kmol

Carbon dioxide formed:

(CO2 kmol)out = (CO2 kmol)in + υCO2 ε = 0 + ε


(CO2 kmol)out = 100 kmol
..........................................

©H ERIOT-WATT U NIVERSITY
TOPIC 4. MATERIAL BALANCE ON SINGLE-STAGE SYSTEMS 119

Example : 4.6.2
Problem:
100 kmol of methane is reacted with 220 kmol oxygen, but the percentage conversion
of methane is 98% and, of the methane that reacts, 95% forms carbon dioxide while the
remaining 5% of the methane that reacts forms carbon monoxide. Calculate how much
of each species (kmol) are present after the reaction
Basis: 100 kmol of methane fuel
Solution:
The balanced chemical reactions are given below:

CH4 + 2O2 → CO2 + 2H2 O .................(Rxn 1)

CH4 + 3/2O2 → CO + 2H2 O .................(Rxn 2)

The fractional conversion is defined as:

moles reactant reacted


Conversion =
moles reactant in feed

|υi ε|
∴ Conversion =
moles reactant in feed

Now find both the extent of reactions ε 1 and ε2 - students are requested to fill out the
blanks below

ε1 = 0.98 × 0.95 × 100 = kmol.

ε2 = 0.98 × 0.05 × 100 = kmol.

©H ERIOT-WATT U NIVERSITY
120 TOPIC 4. MATERIAL BALANCE ON SINGLE-STAGE SYSTEMS

Then, given stoichiometric coefficients have magnitude and sign:


CH4 left unreacted at outlet:

(CH4 kmol)out = (CH4 kmol)in + υCH4 ε1 + υCH4 ε2

(CH4 kmol)out = kmol.

O2 left unreacted at outlet:

(O2 kmol)out = (O2 kmol)in + υO2 ε1 + υO2 ε2

(O2 kmol)out = kmol.

CO2 present at outlet:

(CO2 kmol)out = (CO2 kmol)in + υCO2 ε1

(CO2 kmol)out = kmol.

CO present at outlet:

(CO kmol)out = (CO kmol)in + υCO ε2

(CO kmol)out = kmol.

H2 O present at outlet:

(H2 O kmol)out = (H2 O kmol)in + υH2 O ε1 + υH2 O ε2

(H2 O kmol)out = kmol.

..........................................

©H ERIOT-WATT U NIVERSITY
TOPIC 4. MATERIAL BALANCE ON SINGLE-STAGE SYSTEMS 121

4.7 Tutorial Topic 4


1. 10 t/hr of a 50 mass% solution of NaOH in water is further diluted with water to 30
mass% NaOH the tank. Calculate the amount of water that must be added.

2. 2400 kg/hr of a liquid stream containing 35 mass% propane, 65 mass% butane is


to be heated so that a proportion of the stream is vaporised. The vapour produced
is analysed as 85 mass% propane while the remaining liquid is analysed as 90
mass% butane calculate the mass flowrate of each phase.

3. 3000 kg/hr of a mixture containing 20 mass% propane, 50 mass% butane and 30


mass% pentane is fed to a distillation column. The distillate contains 93 mass%
propane and is essentially pentane free. If pentane constitutes 38 mass% of the
bottom product calculate the following:

a) The flow and compositions of top and bottom products.


b) The fractional recovery of propane in the top product.

4. Two streams S1 and S2 containing a mixture of hexane and heptane are fed to
a continuous distillation column which produces a stream containing 98 mass%
hexane and a stream containing 97 mass % heptane. The compositions and flow
rates of the feed streams are:

Mass Flow (kg/hr) Mass% Hexane Mass% Heptane


Stream "S1" 3000 62 38
Stream "S2" 5000 55 45

a) Determine the mass flowrate of each product stream.


b) Calculate the fraction of the total hexane flow to the column that is recovered
in the hexane-rich product stream.

5. A new plant has been built to carry out the isomerisation reaction "A→B". During
the first month of operation, purchases of "A" were 150 tonnes and sales of "B"
were 70 tonnes. The amounts of raw material and product in store on site at the
beginning and end of the month were :

Mass "A" (tonnes) Mass "B" (tonnes)


Start 10 0
Finish 40 25

The contractor, installing the plant, has guaranteed an 80% conversion efficiency
of A to B. Check whether or not this level of efficiency has been achieved (it should
be found that the conversion is very close to guarantee level at 79.2% conversion).

©H ERIOT-WATT U NIVERSITY
122 TOPIC 4. MATERIAL BALANCE ON SINGLE-STAGE SYSTEMS

6. The two feed streams to a liquid-liquid extraction process are an aqueous solution
containing 3 mass % naphthalene-diamine and an extracting solvent which
consists of diethyl ether with 1.25 mass % water. There are two product streams;
one of these is an ether extract containing 15.2 mass % naphthalene-diamine and
3.67 mass % water, the remainder being ether, while the other is an extracted
aqueous solution (or raffinate) that contains a negligible amount of naphthalene-
diamine but consists of 3.24 mass % ether dissolved in water.
For the recovery of 1000 kg of naphthalene-diamine, calculate the following:

a) Mass (kg) of naphthalene diamine solution used


b) Mass (kg) of ether solution used
c) Percentage of the inlet ether contained in the extracted aqueous solution.

7. Methane gas, flowing at a rate of 1.5 kg/s, is burned completely and continuously
with 5% excess air, calculate the following:

a) Mass flowrate of air


b) Mass flowrate of stack gas
c) Mass % CO2 in stack gas (wet-basis)
d) Mass % CO2 in stack gas (dry-basis).

8. A burner is fed with 100 kg/hr of bituminous coal with the following composition:

C H O S N Ash
Mass% 76.6 5.2 6.2 1.3 1.6 9.1

a) Calculate the theoretical air requirement and the corresponding flue gas
composition (mole%) on both a wet and a dry-basis.
b) Calculate the mole% of CO 2 (wet-basis) if 25% excess air is used.

9. Ethane (C2 H6 ) is burned with 50% excess air. The percentage conversion of the
ethane is 97%. Of the ethane burned, 10% reacts to form CO and the balance to
form CO2 , calculate the composition (mass%) of the flue gas.

10. A supply of liquefied petroleum gas (LPG), containing 65 mole% propane (C 3 H8 )


and 35 mole% butane (C 4 H10 ), is fed to a burner, calculate the following:

a) The theoretical air requirement (kmol) for complete combustion of 100 kmol
of LPG.
b) If the LPG is burned completely using 10% excess air, calculate the
composition of the wet combustion products in both mole% and mass%.

©H ERIOT-WATT U NIVERSITY
TOPIC 4. MATERIAL BALANCE ON SINGLE-STAGE SYSTEMS 123

11. Butane (C4 H10 ) and air are fed to a burner where the butane is burned completely
to carbon dioxide and water. The air supplied is in excess of the stoichiometric
requirement. The mole fraction of carbon dioxide in the flue gas is 0.12 on a dry-
basis. Calculate the % excess air supplied to the burner.

4.8 Bibliography
1. Felder, Richard M. and Rousseau, Ronald W. 2008. Elementary Principles of
Chemical Processes. 3rd ed. India: Wiley

2. Himmelblau, David M. and Riggs, James B. Basic Principles and Calculations in


Chemical Engineering. 2013. 8th ed. London: Pearson.

©H ERIOT-WATT U NIVERSITY
124 TOPIC 4. MATERIAL BALANCE ON SINGLE-STAGE SYSTEMS

©H ERIOT-WATT U NIVERSITY
125

Topic 5

Material Balance on Multi-Stage


Systems

Contents
5.1 Introduction . . . . . . . . . . . . . . . . . . . . . . . . . . . . . . . . . . . . . . 126
5.2 Multiple Units with no Recycle . . . . . . . . . . . . . . . . . . . . . . . . . . . . 127
5.3 Recycle without Chemical Reaction . . . . . . . . . . . . . . . . . . . . . . . . 129
5.4 Recycle with Chemical Reaction . . . . . . . . . . . . . . . . . . . . . . . . . . 130
5.5 Material Balance Strategies . . . . . . . . . . . . . . . . . . . . . . . . . . . . . 131
5.6 Tutorial Topic 5 . . . . . . . . . . . . . . . . . . . . . . . . . . . . . . . . . . . . 143
5.7 Bibliography . . . . . . . . . . . . . . . . . . . . . . . . . . . . . . . . . . . . . . 145

Prerequisite knowledge

• Basic principle of mass conservation.

• Elementary algebra and arithmetic.

• Some familiarity with combustion reactions.

Learning Objectives
By the end of this topic, you should be able to:

• Carry out material balance on simple multi-stage non-reacting processes

• Carry out material balances on simple multi-stage reacting processes.

• Develop systematic problem-solving skills.

• Identify and solve problems involving recycle streams.

• Carry out material balance on processes with recycle streams.

• Master the terminology associated with processes with recycle loops.


126 TOPIC 5. MATERIAL BALANCE ON MULTI-STAGE SYSTEMS

5.1 Introduction
In the last section a number of material balance examples were considered, based on a
variety of process operations, such as combustion, evaporation, distillation etc.:

• Most processes however involve a number of these individual operations,


connected together, with material streams flowing between them.

• The objective in carrying out material balances on these multi-stage processes will
be to determine the mass or molar flows of all components in all of the streams
within the process.

• While this does not involve any fundamentally new principles, it will involve
improved problem-solving skills.

For a continuous process where no chemical reactions occur:

Input = Output

For continuous processes where any number of chemical reactions occur the above
equation must include "generation" and "consumption" terms as follows:

Input + Generation = Output + Consumption

With a multi-stage system these ideas can be applied either to a group of interconnected
units, or an overall balance over the whole process.
In order to develop effective problem-solving skills, it will be necessary at times to take
an overall view of the process, while at other times it may be necessary to concentrate
on a single unit.
What follows next are a sequence of examples that are designed to lead the student
through a succession of more difficult cases.
At all times concentrate on the approach adopted and the possible alternative solution
strategies.

©H ERIOT-WATT U NIVERSITY
TOPIC 5. MATERIAL BALANCE ON MULTI-STAGE SYSTEMS 127

5.2 Multiple Units with no Recycle


Example : 5.2
Problem:
1200 kg/hr of 20 mass% NaOH in water and 1500 kg/hr of 40 mass% NaOH in water are
mixed together. As much as possible of the mixture is passed continuously to "Process
A" which has a maximum capacity of 500 kg/hr (based solely on NaOH), while the
remainder is passed to "Process B". Calculate the flowrates and compositions of the
streams flowing to both "Process A" and "Process B".
Basis: 1 hr of operation
Solution:
This example involves the mixing of two streams and then the immediate splitting of the
combined stream - one going to "Process A" the other to "Process B".
It is better to think of the "mixing" and "splitting" as two distinct stages and to represent
the overall system as follows:

(S1) (S4) To “Process A”


(S3)
Mix Split
(S2) (S5) To “Process B”

Taking a basis of 1 hr, the table below can be partially completed with the information
from the problem specification:

S1 S2 S3 S4 S5
(kg/hr) (kg/hr) (kg/hr) (kg/hr) (kg/hr)
NaOH 240 600 500
H2 O 960 900

TOTAL 1200 1500

The NaOH flow in S1 is based on 20 mass% composition of NaOH. The water flow in
S1 can then be found by difference - the same logic applies to S2. Stream S4 flow of
500 kg/hr is specified by the problem description.

©H ERIOT-WATT U NIVERSITY
128 TOPIC 5. MATERIAL BALANCE ON MULTI-STAGE SYSTEMS

Now add S1 and S2 together to get mixed stream S3:

S1 S2 S3 S4 S5
(kg/hr) (kg/hr) (kg/hr) (kg/hr) (kg/hr)
NaOH 240 600 840 500
H2 O 960 900 1860

TOTAL 1200 1500 2700

The NaOH composition of S3 is 840/2700 = 0.3111 (w/w). It should be noted that when
one stream splits into two then each must have the same composition; thus, S4 and S5
must have same NaOH composition = 0.3111 (w/w).
The total flow of S4 must, therefore, be = 500/0.3111 = 1607 kg/hr. The water content,
by difference, must be 1107 kg/hr. S5 can then be found by difference since S4 - S3 =
S5
S1 S2 S3 S4 S5
(kg/hr) (kg/hr) (kg/hr) (kg/hr) (kg/hr)
NaOH 240 600 840 500 340
H2 O 960 900 1860 1107 753

TOTAL 1200 1500 2700 1607 1093

This example was solved by considering each of the process stages in turn. It is possible
to carry out a final check of the calculation accuracy by making sure that the overall mass
balance is correct.
Notice, total mass flow into mixer and out of splitter are as follows:

• Mass flow IN = S1 + S2 = 1200 + 1500 = 2700 kg/hr.

• Mass flow OUT = S4 + S5 = 1607 + 1093 = 2700 kg/hr.

..........................................

©H ERIOT-WATT U NIVERSITY
TOPIC 5. MATERIAL BALANCE ON MULTI-STAGE SYSTEMS 129

5.3 Recycle without Chemical Reaction


In chemical engineering it is common for unreacted or off-specification feedstock to be
recycled back to an upstream processing unit for further treatment:

Under- On-Spec Over-


- The exit stream from a
Mass Fraction Size Product Size crushing unit may show a wide
particle-size distribution.
Of Given

Size - Only part of that size range will


be suitable as product, or for
further processing.

Particle Size

Use a coarse-aperture screen to first separate oversize product then a fine-aperture


screen to separate undersize product. The "acceptable" stream is thus separated from
both the "coarse" and the "fine" product streams.
This process is extremely inefficient - should it be operated without any material being
recycled. With no recycle both oversize and undersize streams would either go to waste
or would be sold off cheaply as "off-specification" product.
Loss of oversize material is fairly easy to overcome - simply recycle it back to the crusher.
This simple step will significantly improve process efficiency, but there would still be a
problem associated with the undersize stream losses:

• If some process could be found to agglomerate this undersize material into


larger particles, then the overall process efficiency could be further improved by
incorporating such a device into a second recycle stream.

If a chemical reaction does not go to completion, due to chemical equilibrium constraints,


then the overall process efficiency and costs may be improved by recycling unreacted
feedstock back to the "fresh feed".

©H ERIOT-WATT U NIVERSITY
130 TOPIC 5. MATERIAL BALANCE ON MULTI-STAGE SYSTEMS

5.4 Recycle with Chemical Reaction


Consider the Haber ammonia synthesis process shown below - the reactor conversion,
to limiting reactant, is typically as low as 25%:

N2 + 3H2 ↔ 2N H3

After the reactor, the gas stream contains NH 3 plus unreacted H2 and N2 . If this stream
is cooled, then NH3 will condense to a liquid and can be separated - thus, the unreacted
gaseous H2 and N2 can be recycled back to the reactor:

Recycle: N2 & H2

Fresh Feed:
Feed
d: Product:
Reactor Separator
N2 H2 N2 H2 & NH3 NH3

The complexity of the recycle stream may be avoided by considering everything within
the recycle loop as an "overall process" - remember, fresh feed flowrate must equal
product flowrate no matter what happens within the recycle loop.

Fresh
esh Feed: Overall Product:

2 H2
N2 Process NH3

Viewed this way we see that only N 2 & H2 enter the system and only NH 3 exits.
Say, for example, 1000 kmol/hr of NH 3 product is needed and if a stoichiometric fresh
feed mixture is assumed, then the fresh feed flows of N 2 and H2 (fixed by the reaction
equation) must be 500 kmol/hr and 1500 kmol/hr respectively.
General Chemical Engineering textbooks are a good source of worked examples (Felder
and Rousseau, 2008) and (Himmelblau and Riggs, 2013).

©H ERIOT-WATT U NIVERSITY
TOPIC 5. MATERIAL BALANCE ON MULTI-STAGE SYSTEMS 131

5.5 Material Balance Strategies


The key to solving recycle problems is to recognise that the process is continuous and
operating under steady state conditions - therefore, basic material balance principles
can be applied to equipment as follows:

• An individual process unit may be selected.

• A group of process units may be selected.

• The entire process may be selected.

For all of the above, there can be no accumulation of material within the chosen system,
thus, total flows in/out must always be in balance - it helps to draw an envelope and all
flows entering and leaving this envelope must be considered:

A B

C E

In spite of the complex structure of the above process, envelopes may be drawn so that
the following material balance statements are all equally valid (applicable):

ṁC + ṁD = ṁE

Or

ṁA = ṁB

Where

ṁX = Mass flow of stream "X" in various units, often (kg/hr).

Notice different symbols may be used for mass flowrate. Here it is the ṁ S1 (kg/hr)
approach - at other times it can be simplified to just S1 (kg/hr).

©H ERIOT-WATT U NIVERSITY
132 TOPIC 5. MATERIAL BALANCE ON MULTI-STAGE SYSTEMS

Example : 5.5.1
Problem:
In the flow diagram shown below the mass flowrates (kg/hr) of various streams have
been marked. Calculate .the flowrates of the FIVE streams marked S1, S2, S3, S4, S5.
Basis: 1 hr of operation
Solution:
This example looks complicated but notice carefully how the problem is tackled:
S1
S2 S3 5840
A B C

2007 S4 2240
D
S5 E

1296

Start with a balance on Block C :

S3 = 5840 + 2240 = 8080 (kg/hr)

Similarly for Block E:

S5 = 2240 - 1296 = 944 (kg/hr)

S4 is now the only unknown stream connected to Block D, so S4 may be solved by a


balance over Block D:

S4 = 2007 - 944 = 1063 (kg/hr)

Both outputs from block B are now known, so the Block B balance now gives :

S2 = 8080 + 1063 = 9143 (kg/hr)

Finally S1 can be found from a balance over block A:

©H ERIOT-WATT U NIVERSITY
TOPIC 5. MATERIAL BALANCE ON MULTI-STAGE SYSTEMS 133

S1 = 9143 - 2400 - 2007 = 4736 (kg/hr)

From a previous topic it was found that whenever all the independent equations
had been used to solve for the available unknowns, then the final non-independent
expression could be used as a check.
The same approach can be used for material balances - whenever all the independent
material balances have been used to solve for the available unknowns, the remaining
non-independent balance can be used as a check.
In this case material balances have been completed over each of the blocks taken
individually - thus, the remaining non-independent balance can be used to check the
answer.
The one remaining - non-independent - balance is across the entire process, remember
to draw the envelope to exclude the recycle loop:

IN = 2400 + 4736 = 7136 (kg/hr)

OUT = 5840 + 1296 = 7136 (kg/hr)

Of course, this is not the only possible solution strategy for this problem. One other
feasible calculation sequence could be as follows:
Overall balance → S1
Balance on A → S2
Balance on E → S5
Balance on D → S4
Balance on B → S3

Now the balance on C is non-independent and can be used as a check. Notice that
various symbols can be used to represent flowrate - here "S1, S2", etc. was used. Be
prepared to recognise different symbols.
..........................................

©H ERIOT-WATT U NIVERSITY
134 TOPIC 5. MATERIAL BALANCE ON MULTI-STAGE SYSTEMS

Example : 5.5.2
Problem:
A closed-circuit grinding system is being used to produce 1.5 kg/s of powder finer than
200 mesh - to be used downstream (later) in a leaching process.
Material from the grinder is passed to a 200 mesh screen through which 80 mass% of
the material passes. The oversize material is recycled back to the inlet of the grinder.
Calculate the "recycle ratio" for the system.
Basis: 1 second of operation or 1.5 kg/s of product
Solution:
When Chemical Engineers are discussing large continuous plants, with recycle streams,
a key piece of terminology is the "Recycle Ratio" - defined below:

Recycle Flow
Recycle Ratio = ..........(5.1)
Fresh Feed Flow

With complicated systems it is important to state whether a mass or molar ratio is being
used - in this simple case it doesn’t matter. With multi-component systems recycle ratios
may be defined in different ways.
Although only the recycle ratio is asked for it is strongly recommended that a full material
balance is carried out; this will allow a final check to be carried out, before working out
the recycle ratio.
Feed
(S1)
- First consider an overall balance on the
Oversize
whole system – that is the envelope
(S2)
excludes the recycle loop.

(S5)
Grinder
- For this overall balance only stream “S1”

(S3)
enters, while only stream “S4” leaves.

- The entire process, streams “S2”, “S3”


Screen (S4)
and “S5” and both the grinder and the

Product screen become a single “overall” block.

©H ERIOT-WATT U NIVERSITY
TOPIC 5. MATERIAL BALANCE ON MULTI-STAGE SYSTEMS 135

Therefore, if 1.5 kg/s leaves this overall system in S4 then 1.5 kg/s must enter the overall
system in stream S1; so that, S1 = 1.5 kg/s.
The problem specifies that 80% of the material in S3 passes through the screen. Thus
S3 = S4 / 0.8 = 1.5/0.8 = 1.875 kg/s
Now apply a material balance over the screen itself - use the notation "SX" to represent
the flowrate (kg/s) of stream "X".

S3 = S4 + S5 (kg/s)

S5 = S3 - S4 = 1.875 - 1.5

∴ S5 = 0.375 kg/s

Now consider the point at which the fresh feed and recycle are mixed. The total feed to
the grinder is:
S2 = S1 + S5 = 1.5 + 0.375

∴ S2 = 1.875 kg/s
All of the streams have now been calculated - thus, the material balance table can be
summarised as follows :

Stream Mass Flowrate


Component
(kg/s)
S1 S2 S3 S4 S5
Powder 1.5 1.875 1.875 1.5 0.375

The only part of the system which was not considered in the solution procedure was the
grinder.
Previous balances around mixing point gave S2 = 1.875 kg/s and the screen gave S3 =
1.875 kg/s. Now a material balance over the grinder (non-independent balance) checks
out these answers.
The Recycle Ratio = 0.375/1.5 = 0.25.
..........................................

©H ERIOT-WATT U NIVERSITY
136 TOPIC 5. MATERIAL BALANCE ON MULTI-STAGE SYSTEMS

Example : 5.5.3a
Problem:
In an ammonia plant a fresh feed with a hydrogen/nitrogen molar ratio of 3:1
(stoichiometric ratio) is fed to the reactor along with recycled material. In the reactor,
25% conversion to nitrogen is achieved. Product ammonia is completely condensed
from the reactor outlet stream and the unreacted hydrogen and nitrogen are recycled
back to the reactor.
Fill out a complete material balance table based on production of 100 kmol of ammonia
product.
Basis: 100 kmol of pure ammonia product.
Solution:
A block diagram of a typical ammonia synthesis plant is shown below - each of the blocks
actually consists of a sequence of process units, but the overall plant can be thought of
as only two "blocks":

(S5)

Processing Separator
(S1) (S2) (S3) (S4)

Before starting this problem some common terminology should be defined. For the
generic (non-specific) block diagram shown above streams S1,S2,S3,S4 and S5, in
general, should be referred to as:

S1 Fresh (or Net) Feed


S2 Total (or Gross) Feed
S3 Total (or Gross) Product
S4 Net Product
S5 Recycle

A material balance over the whole system (outer dotted line) is called an "Overall
Material Balance".

©H ERIOT-WATT U NIVERSITY
TOPIC 5. MATERIAL BALANCE ON MULTI-STAGE SYSTEMS 137

A material balance over the main processing unit (inner dotted line), in this case the
reactor, is called a "Single-Pass, or Once-Through, Material Balance". The balanced
chemical reaction is of course very well known:

N2 + 3H2 ↔ 2N H3

There is rarely only one way (unique approach) of tackling these problems - to
emphasise this fact two entirely different strategies (approaches) will be used.
In the first case, take as a basis "100 kmol of NH3 product" - using this approach the best
way to proceed is to consider an overall material balance, i.e. the outer dotted overall
envelope - thus, the system can be simplified as follows:

(S1) (S4)

The H2 and N2 input via S1 must correspond to the NH 3 produced via S4 (taking account
of the reaction); there are no other streams passing through the outer envelope - and
notice that the recycle stream is within this envelope.
The Fresh Feed S1 is stoichiometric, consisting of pure N 2 and H2 , while only pure NH3
leaves in S4 - thus, there are no unreacted species in S4.
However, there must be twice as many kmoles of reactants in S1 as there are products in
S4 (taking account of the reaction stoichiometry and the fact that there are no unreacted
species in S4). Thus, 200 kmol of Fresh Feed must enter in S1.
Moreover, the ratio of H2 :N2 is 3:1 - hence, the 200 kmol of Fresh Feed must contain 50
kmol of N2 and 150 kmol of H 2 .
S4 consists of 100 kmol NH3 - this is pure ammonia, since it is condensed and separated
from the gaseous N2 and H2 which are all recycled. Therefore, there are 0 kmol of N 2
and 0 kmol of H2 in S4.
Notice that in terms of the overall balance, kmoles "IN" and kmoles "OUT" are not
conserved - this is because a chemical reaction takes place. However, kmoles can
be accounted for through the reaction stoichiometry (as was done above).
In addition, in the recycle stream S5 there can be no NH 3 since this is completely
condensed and separated from the recycled N 2 /H2 gas mixture.
Since there is 0 kmol NH3 in both S1 (fresh feed) and S5 (recycle) there must be 0 kmol
NH3 in S2 (total feed).

©H ERIOT-WATT U NIVERSITY
138 TOPIC 5. MATERIAL BALANCE ON MULTI-STAGE SYSTEMS

Filling in the material balance table leads to:

Material present in each Stream


Components
(kmol)
S1 S2 S3 S4 S5
N2 50 0
H2 150 0
NH3 0 0 100 0
Total 200 100

Since there is no NH3 in S5 and 100 kmol NH 3 in S4, then an NH 3 balance about the
separation block leads to the conclusion that there must be 100 kmol NH 3 in S3.

(S5)

Processing Separator
(S1) (S2) (S3) (S4)

Notice there is no chemical reaction in the separation block so that in this case kmoles
are conserved for all components:
There is 0 kmol NH3 in S2 and 100 kmol NH 3 in S3, therefore, +100 kmol of NH 3 must
have reacted (must have been produced) - knowing this allows the "extent of reaction"
to be calculated, since species reacting is υi ε.
Since the stoichiometric coefficient of NH3 is +2, it follows that the extent of reaction
must be ε = 50 kmol - this now applies to all reacting species.
Now take an envelope just across the reactor - not all the N 2 and H2 will react (unlike
the situation for the overall envelope), but knowing the extent of reaction, the amount of
N2 reacting = -1×50 = -50 kmol.
Using the same approach the amount of H 2 reacting = -3×50 = -150 kmol. However,
more N2 and H2 are present than are reacting due to the recycle stream, where
unreacted gaseous feedstock is returned to reactor inlet.

©H ERIOT-WATT U NIVERSITY
TOPIC 5. MATERIAL BALANCE ON MULTI-STAGE SYSTEMS 139

The key specification is that the conversion to N2 is 25% or 0.25 on a fractional basis.
The fractional conversion to a reactant is the amount of that reactant which reacts
divided by amount of same species in the feed (in this case S2).

(S5)

Reactor Separator

(S1) (S2) (S3) (S4)

Therefore, the N2 in S2 = 50/0.25 = 200 kmol. Since the ratio of N 2 :H2 must be 3:1 in S2
and, given that species also react in this ratio, it follows that this ratio must be preserved
through streams S2, S3 and S5.
Hence, the amount of H 2 in S2 must be 3 x 200 = 600 kmol. Now to find species in S3
simply take amounts of each in S2 and simply add or subtract the amount of each that
has reacted (change in kmoles between S2 and S3).
Using this approach N2 in S3 = 200 - 50 = 150 kmol and H 2 in S3 = 600 - 150 = 450
kmol, while NH3 in S3 = 0 + 50 = 50 kmol NH 3 .
From the above it will be appreciated that the "extent of reaction" is very useful since
it applies to all components taking part in the reaction. Now filling this additional
information into the table leads to:

Material present in each Stream


Components
(kmol)
S1 S2 S3 S4 S5
N2 50 200 150 0
H2 150 600 450 0
NH3 0 0 100 100 0
Total 200 800 700 100

©H ERIOT-WATT U NIVERSITY
140 TOPIC 5. MATERIAL BALANCE ON MULTI-STAGE SYSTEMS

The N2 and H2 from S3 then pass unchanged into the recycle stream S5 and the
completed material balance now becomes:

Material present in each Stream


Components
(kmol)
S1 S2 S3 S4 S5
N2 50 200 150 0 150
H2 150 600 450 0 450
NH3 0 0 100 100 0
Total 200 800 700 100 600

Three balances (overall, reactor and separator balances) have been completed:

(S5)

Reactor Separator

(S1) (S2) (S3) (S4)

The final balance across the mixing point (smallest envelope) can be used just to test
the answers; check that S2 = S1 + S5, for each component, and overall.
..........................................

©H ERIOT-WATT U NIVERSITY
TOPIC 5. MATERIAL BALANCE ON MULTI-STAGE SYSTEMS 141

Example : 5.5.3b
Problem:
There are a number of ways of doing these mass balances. To illustrate this once again
calculate the material balance table for all components in all streams for the previous
ammonia process, but this time choose a completely different basis.
Basis: 100 kmol of reactor feed (same composition).
Solution:
Whenever key information is available for the reactor (the conversion to nitrogen in this
case), then it may be better to start with the total feed stream to the reactor. Notice that
this is not fresh feed to reactor.
Initially, the feed flow needed to produce the desired product will be unknown, but if
some arbitrary basis is taken then the calculation may be scaled later
Since the previous ammonia example was fully worked out, students are required to
complete this variation themselves.
There is 100 kmol of total reactor feed and, because the H 2 /N2 ratio is specified at
3:1 (and this prevails throughout the system), then stream S2 must contain exactly
kmol of N2 and kmol of H2 .
The fractional conversion to a specified reactant is the amount of the species that reacts
divided by the amount of the same species in the feed (in this case S2).
Given a 0.25 conversion to N 2 it follows that amount of N2 in feed that reacts must be
kmol, the extent of reaction is thus kmol.
With this extent of reaction all other species reacting can be found; thus, H 2 in feed that
reacts kmol and NH3 that reacts is
kmol.
Knowing how much of each species is present in S2 and how much of each has
reacted, it is easy to calculate the composition of S3 by difference (subtraction): N 2
in S3 is kmol, H2 in S3 is kmol, NH3 in S2 is
kmol.
Now filling all this information into the table the student should get the following:

Material present in each Stream


Components
(kmol)
S1 S2 S3 S4 S5
N2 25 18.75 0
H2 75 56.25 0
NH3 0 0 12.5 0
Total 100 87.5

©H ERIOT-WATT U NIVERSITY
142 TOPIC 5. MATERIAL BALANCE ON MULTI-STAGE SYSTEMS

A balance over the separator S3 = S4 + S5 leads to the amounts in S4 and S5 as


summarised below - please check.

Material present in each Stream


Components
(kmol)
S1 S2 S3 S4 S5
N2 25 18.75 0 18.75
H2 75 56.25 0 56.25
NH3 0 0 12.5 12.5 0
Total 100 87.5 12.5 75

A balance over the fresh feed mixing point S2 = S1 + S5 leads to - please check.

Material present in each Stream


Components
(kmol)
S1 S2 S3 S4 S5
N2 6.25 25 18.75 0 18.75
H2 18.75 75 56.25 0 56.25
NH3 0 0 12.5 12.5 0
Total 25 100 87.5 12.5 75

If the material balance required 100 kmol of NH 3 rather than on the 12.5 kmol above,
simply multiply all the entries in the above table by 100/12.5 = 8. The student is required
to check this.
..........................................

©H ERIOT-WATT U NIVERSITY
TOPIC 5. MATERIAL BALANCE ON MULTI-STAGE SYSTEMS 143

5.6 Tutorial Topic 5


1. 2000 tonnes/day of dried solid containing 1 mass % water are produced from slurry
containing 30 mass % water in a two stage drier. If the quantities of water removed
in the first and second stage are in the ratio of 3:1, calculate the following:
a) Flowrates of all the streams.
b) The moisture content of the stream passing between the two stages.

2. Three streams of water containing suspended solids are mixed together as shown
below:
(S2) (S3)

(S1) (S4) (S5)

Measurements of flow and solids content for some of the streams are tabulated
below:

Stream S1 S2 S3 S4 S5
Flow (kg/hr) 3000 4500
Mass%
5 3 2 3.1
Solids

Complete the missing mass% solids and flowrates in this table.

3. The diagram below shows estimates of the flowrates (t/hr) throughout a plant,
based on measurements from flow meters which normally read to within 2% of the
correct value:
151 81

100 453 298 201

198 290
99

130

©H ERIOT-WATT U NIVERSITY
144 TOPIC 5. MATERIAL BALANCE ON MULTI-STAGE SYSTEMS

One of the above values however is grossly in error. Determine the location of
the faulty instrument and provide a better estimate of the corresponding mass flow
rate.

4. The fresh feed to an ammonia synthesis process is a 3:1 molar ratio of hydrogen
to nitrogen. The mixture leaving the reactor contains 30 mole% ammonia. The
ammonia is separated from the unreacted hydrogen/nitrogen mixture, which is
recycled and mixed with the incoming feed.
It may be assumed that a perfect separation is achieved so that no N 2 /H2 appear in
the final ammonia product stream and that no ammonia is recycled. If so calculate
the single-pass % conversion and the recycle ratio.

5. The diagram below shows a process which converts butane to butene according
to the reaction:

C4 H10 → C4 H8 + H2

REACTION
R SEPARATION
SEP

The feed to the process is pure butane and the overall conversion for the process
is 90%. No side reactions occur.
The final product stream from the process contains all of the hydrogen, 92% of the
butene and some of the butane leaving the reactor. The reactor feed contains 5
mole% butene. Determine the molar recycle ratio and the single-pass conversion
of butane.

©H ERIOT-WATT U NIVERSITY
TOPIC 5. MATERIAL BALANCE ON MULTI-STAGE SYSTEMS 145

6. The diagram shows a system for removal of water from natural gas using a liquid
phase drying agent "D":

DRY

GAS DRYING

DRYING AGENT, D

COLUMN

DISTILLATION

COLUMN

WET

GAS

The liquid fed to the top of the drying column contains 98 mass% D and 2 mass%
H2 O, while the liquid leaving this column contains 93 mass% D and 7 mass% H 2 O.
This stream is distilled to remove a distillate with a water content of 95 mass%.
The bottom product from the distillation is mixed with a 'make-up' stream of pure D
to provide the liquid feed to the drying column. It may be assumed that the natural
gas does not dissolve in the liquid in the drying column, and that this liquid does
not evaporate into the gas.
For the removal of 100 kg/hr of water from the natural gas stream calculate the
following:
(a) The flowrate of make-up "D".
(b) The flowrate and composition of the bottom product from the distillation column.

5.7 Bibliography
Felder, Richard M. and Rousseau, Ronald W. 2008. Elementary Principles of Chemical
Processes. 3rd ed. India: Wiley
Himmelblau, David M. and Riggs, James B. Basic Principles and Calculations in
Chemical Engineering. 2013. 8th ed. London: Pearson.

©H ERIOT-WATT U NIVERSITY
146 GLOSSARY

Glossary
Absolute Pressure
An absolute pressure measurement takes zero on the measurement scale to be
a complete vacuum whatever system of units is used: thus, absolute pressure is
measured relative to a complete vacuum.
Basis
The basis for solving a chemical engineering problem is the starting assumption
around which all the subsequent stream flows and compositions are based. The
basis is chosen for convenience usually depending on what is known or given
in the problem statement. If the result does not produce the quantities required
everything can be scaled to give the desired result.
Batch Processes
Batch processes are where reagents are charged to the unit, process changes
occur within the batch over time, products are removed and then the vessel is
prepared for the next batch. These are generally small-scale operations producing
different products.
Block Diagram
A block diagram is one or more process units represented by a rectangular block.
Coherent Systems
A coherent system of units is where the derived units are products and quotients
of fundamental units raised to some power without any proportionality factor. The
SI and CGS systems are both coherent, but the FPS system is non-coherent.
Combustion
Combustion is a process where some fuel reacts rapidly with oxygen (usually in
air) to release large quantities of heat. Typical fuels are methane gas, oil, coal or
hydrogen.
Complete Combustion
Complete combustion is where, in a combustion reaction, all the fuel is completely
consumed. Practically this means that the combustion equipment must be
supplied with more air than is theoretically needed based on the reaction
stoichiometry.
Continuous Processes
Continuous processes are where material flows continuously enter and leave the
process. They are associated with large-scale bulk chemical manufacture where
the same product is being produced in large amounts.
Critical Point
The critical point is where the saturated vapour curve meets the saturated liquid
curve. At the critical point liquid and vapour phases become indistinguishable as
the properties of both phases become identical. The critical point lies at one end
of the vaporisation curve and liquid-vapour phase splitting is not possible beyond
this point. The triple point lies at the other end of the liquid-vapour equilibrium
curve (vaporisation curve).

©H ERIOT-WATT U NIVERSITY
GLOSSARY 147

Critical Pressure
The critical pressure is the pressure at the critical point. It is a key parameter,
along with the critical temperature, in any predictive EOS. It also plays a central
role in predicting physical properties of both pure components and mixtures.

Critical Temperature
The critical temperature is the temperature at the critical point. It is the second key
parameter needed for any predictive EOS. It also, along with the critical pressure,
pays a central role in predicting physical properties of both pure components and
mixtures.

Derived Units
Derived Units are units that can be expressed as products or quotients of
fundamental units raised to some power: for instance the newton (N) in terms
of fundamental units is (kg m/s2 ); the joule (J) in terms of fundamental units is (kg
m2 /s2 ); the watt (W) in terms of fundamental units is (kg m2 /s3 ), and so on.

Differential Balance
When the material balance is differential then the important factors are the rate or
instantaneous flow of material in, rate of material flows being generated, rate of
material flows being consumed and rate of material flows leaving the process.

Dimensional Consistency
Any equation linking together variables with different units and dimensions must
be dimensionally consistent. The dimensions on the left hand side of the equation
must be the same as the dimensions on the right hand side once, that is, common
dimensions are cancelled. The same logic applies to units, irrespective of the
system being used, but all derived units will need to be reduced to fundamental
units before cancelling common units.

Dimensionless Groups
Dimensionless groups may be formed by grouping variables together so that all
the dimensions cancel completely. Sometimes these groups occur naturally in an
algebraic expression. However, very often dimensionless groups are deliberately
set up where they have real physical meaning - for instance, Reynolds's Number
is the ratio of inertial to viscous forces within a flowing fluid. Dimensionless groups
are used to scale small-scale trials physically so that the outcome represents
large-scale plant operation. They are also used to simplify correlations between a
large number of variables.

Dimensions
Any physical quantity can be measured using units and, depending on the system
of units used, a different number will be obtained for the same physical quantity.
However, any physical quantity will have the same dimensions, irrespective of the
system of units used. The fundamental dimensions are: mass [M], length [L], time
[T], temperature [θ], amount of a substance [N], electric current [I] and luminous
intensity [J]. Velocity in FPS is (ft/s) in SI it is (m/s) and for both its dimensions are
[L][T]-1 .

©H ERIOT-WATT U NIVERSITY
148 GLOSSARY

Distillation Column
A distillation column is an item of process equipment that separates two or more
components by exploiting the difference in volatility between the components. The
key factor in distillation is that the vapour phase will always be richer in the more
volatile component. Thus, if a sequence of vaporisations and condensation steps
are followed one after the other the more volatile components may be separated
from the less volatile components.
Duty
Duty is the same as heat load but usually applied to heat exchangers. Chemical
Engineers talk about “the duty of HX-1 or the duty of HX-2”. All items of equipment
are numbered and heat exchangers are often designated using letters “HX”
followed by a number.
Equation of State
An Equation of State (EOS) links molar volume "v" (m 3 /kmol) to absolute
temperature "T" (K) and absolute pressure "P" (kN/m 2 ). The simplest EOS is the
ideal-gas law; a more accurate expression is the van der Waal EOS. There are
now much better and more accurate EOS than the van der Waal equation.
Evaporator
An evaporator is an item of process equipment where a solvent is separated from
a non-volatile solute by vaporising the solvent through the application of heat. The
solvent is later recovered by heat removal and condensation.
Excess
Excess means that one reactant is supplied in excess of the theoretical
stoichiometric proportion (see "Reacts Stoichiometrically") - this is a practical
device to ensure that as much as possible of the limiting reactant actually reacts
(see "Limiting" in this glossary).
Excess Air
See excess oxygen in this glossary. In most cases the combustion, although
between fuel and oxygen, sources the oxygen from air. The excess air requirement
is easily found once the excess oxygen is known
Excess Oxygen
The minimum oxygen required for complete combustion of the fuel is the
stoichiometric oxygen requirement obtained from a properly balanced chemical
equation. In reality, because the fuel has a limited residence time in the
combustion system, excess oxygen is used to ensure complete combustion. The
amount of excess oxygen depends on how finely atomised and how well-mixed
the fuel and air can be arranged in the combustion system.
Extent of Reaction
The extent of reaction can be found from any species in the reaction, once the
amount of that species that has reacted is known. The extent of reaction is the
quantity of the species that has reacted divided by its stoichiometric coefficient.
The extent of reaction is always positive and has units of mol or kmol. It varies
between zero (no reaction) and the maximum amount (kmol) of limiting reactant
initially present.

©H ERIOT-WATT U NIVERSITY
GLOSSARY 149

Fractional Conversion
Fractional Conversion is defined by the following fraction: "moles of reactant
species that has reacted" divided by "moles of the same component in feed" - it
may also be expressed as percentage conversion. The conversion of any reactant
may be calculated, but it is usually the conversion of the limiting reactant that is of
interest.

Fractional Excess
Fractional Excess is defined by the following fraction: "amount supplied in excess
of stoichiometric requirement" divided by "stoichiometric requirement" - it may also
be expressed as percentage excess.

Fresh Feed
Fresh feed is the point in a process where fresh reactants or ingredients are
introduced into the process. Fresh feed implies that there are no recycled
components in the stream at that point.

Fundamental Units
Fundamental units are units that cannot be broken down into simpler units. In the
SI system there are seven fundamental units: kilogram (kg), metre (m), second
(s), kelvin (K), mole (mol), ampere (A) and candela (cd).

Gauge Pressure
Gauge pressure measurement takes zero on the measurement scale to be
atmospheric pressure whatever system of units is used: thus, gauge pressure
is measured relative to atmospheric pressure

Gibbs Phase Rule


The Gibbs Phase Rule links together the number of degrees of freedom (the
number of Gibbs Phase Rule variables that must be fixed) to the number of
components and the number of phases present in a mixture. It is also the number
of variables minus the number of equations linking these variables together. Note
that Gibbs Phase Rule variables are only temperature, pressure and composition.
For a pure substance the Phase Rule variables are only temperature and pressure.

Heating Power
Heating Power is the rate at which heat must be transferred into or out of a fluid,
usually moving under steady-flow conditions. It is expressed in watts or kilowatts.

Heat Load
Heat load is the same as heating power but is more widely used in a Chemical
Engineering context. The heat load is a term that can be applied to any type of
equipment where heat is being transferred in or out of a moving fluid.

Inert Species
An inert species has a stoichiometric coefficient of zero since it passes through
the reaction unchanged, neither being produced nor consumed. For example in
combustion reactions nitrogen passes through unchanged and is inert - ignoring
the small NOx component which, although important, does not materially affect
overall nitrogen flows.

©H ERIOT-WATT U NIVERSITY
150 GLOSSARY

Integral Balance
When the material balance is integral then the important factors are the total
material flows in (over time), total material flows being generated (over time), total
material flows being consumed (over time) and total material flows leaving the
process (over time). Integral balances are used for batch plant where total material
balance, over a period of time, is needed.
Latent Heat
Latent heat effects are the heat effects that occur solely when a substance
changes phase - these can be associated with vaporisation (liquid to vapour or
vice versa), fusion (solid to liquid or vice versa) or sublimation (solid to vapour or
vice versa). A feature of latent heat transfer is that heat is added or removed from
a substance without its temperature or pressure changing.
Limiting
Limiting means that if one reactant is supplied in excess (see "Excess"
in this glossary) of the theoretical stoichiometric proportion (see "Reacts
Stoichiometrically" in this glossary) then the other reactant must be limiting. For a
combustion reaction the fuel would be limiting and the air would be in excess. This
will be done to ensure complete combustion of the fuel
Manometer
A manometer is the transparent "U-tube" instrument used for measuring
manometric pressure; see "manometric pressure" in this glossary for further
details.
Manometric Fluid
The manometric fluid is the liquid that is displaced in the transparent "U-tube"
device, called a manometer; the displacement of the liquid is proportional to the
pressure difference between each leg. Common manometric fluids are water and
mercury, but other fluids can be used. The density of the manometric fluid must
be accurately known, so that the pressure difference may be calculated using a
formula.
Manometric Pressure
Manometric pressure is a pressure measurement taken using a transparent "U-
tube" in which the displacement of a liquid is directly proportional to the pressure
difference between each "U-tube" leg. The measurement is usually expressed as a
height of a liquid column (commonly water or mercury) and this can be converted
into standard pressure units using a formula. If one leg is open to atmosphere
then gauge pressure is being measured. If one leg is exposed to a vacuum then
absolute pressure is being measured.
Mass Balance
A mass balance can be applied to any item of process equipment. The general
principle is that mass is conserved and the mass entering a system must equal
the mass leaving a system whatever the system.
Mass Concentration
Mass concentration is defined as the following ratio: "mass of a component in the
mixture (kg)" divided by "total volume of the mixture (m3 )".

©H ERIOT-WATT U NIVERSITY
GLOSSARY 151

Material Balance
A material balance is very similar to a mass balance (see mass balance) except
the term material balance includes mass and moles and even volume. The
only one out of the three that must be conserved is mass - volume is rarely
conserved unless species are very similar. Moles are only conserved if the overall
stoichiometric coefficient is zero or if no chemical reaction takes place in the
process unit.

Molality
Molality is defined as the following ratio: "moles of a component in the solution
(mol)" divided by "mass of solvent (kg)".

Molar Concentration
Molar concentration is defined as the following ratio: "moles of a component in the
mixture (kmol)" divided by "total volume of the mixture (m3 )".

Molarity
Molarity is defined as the following ratio: "moles of a component in the solution
(mol)" divided by "total volume of the solution (l)". This is the same as kmol/m 3

Molar Mass
The molar mass is the mass of a substance divided by the amount of that
substance. In SI units it is expressed as (kg/kmol) - thus the molar mass in SI
units is the mass in (kg) of 1 kmole of the substance. The molar mass of a species
is found by adding the atomic masses of all the atoms that make up the molecule.

Mole
A mole is the amount of any substance that contains the same number of entities
(ions, electrons, atoms or molecules) as 12 g 12 C. This is the same as Avogadro's
number being 6.023×10 23 entities.

Multi-Stage
Multi-stage applies when multiple processing units are under consideration - this
could be two process units or an entire process plant.

Non-Condensable Component
The term non-condensable component arises when two gases are mixed together
and cooled. As the temperature is reduced one component may condense and
the other may not - the component that does not condense is said to be the
non-condensable component. An air/water system is a good example, under
reasonable temperatures water may condense out of the air, but the air is non-
condensable.

Normal Boiling Point


When the vapour pressure is equal to the applied pressure the liquid will boil and
this can happen at any temperature between the triple point and the critical point.
The Normal Boiling Point (NBP) is the temperature at which a liquid will boil when
the applied pressure is 101.325 kN/m 2 (atmospheric pressure).

©H ERIOT-WATT U NIVERSITY
152 GLOSSARY

Overall Stoichiometric Coefficient


The Overall Stoichiometric coefficient can be found by adding up all stoichiometric
coefficients across the reaction taking due account of sign - reactants being
negative, products being positive and inert species being zero. If the overall
stoichiometric coefficient is non-zero then more or less moles will be produced
than are consumed and there will be a change in number of moles across the
reaction. If the overall coefficient is zero then moles will be conserved.
Partial Pressure
The partial pressure is the notional pressure that a single gas species in a
mixture would exert if it occupied the total system volume, by itself, at the same
temperature as the mixture.
Partial Volume
The partial volume is the notional volume that a single gas species in a mixture
would occupy if it were present, by itself, at the same total pressure and
temperature as the gas mixture.
Phase Change
If a substance is in a solid phase (say ice), the molecules are bound tightly together
by attractive forces and oscillate about fixed positions close to each other within
a rigid structure. As the substance is heated (ice) the vibration of the molecules
will become large enough to allow molecules to break out of this solid structure
and move freely as a liquid. A phase change from solid to liquid occurs at this
temperature (melting point). Another phase change occurs at higher temperatures
when molecules can become freely separated from the liquid and move into the
vapour phase (boiling point). The temperature and pressure are both constant
during phase changes so long as the substance is pure.
Reactor
The reactor is an important item of processing equipment within which raw
materials are transformed chemically into desirable products and undesirable side-
products. The remainder of the plant is devoted to separating and purifying the
products leaving the reactor.
Reacts Stoichiometrically
The phrase "reacts stoichiometrically" applies to a theoretical situation where
reactants are fed into a reactor using only their stoichiometric proportions - that
is the proportions predicted by a properly balanced reaction equation. In practice
stoichiometric proportions are rarely used since some reactants would then leave
the reactor unreacted.
Recycle
The word recycle means to reuse a product. In Chemical Engineering the term
is used more specifically to mean that off-specification product or unreacted
feedstock is returned to near the beginning of the process to be reworked.
Recycle Ratio
The recycle ratio is defined as the ratio of the recycle flow to the flow of fresh feed.
It is a key piece of information when solving material balance problems that involve
recycle loops.

©H ERIOT-WATT U NIVERSITY
GLOSSARY 153

Recycle Stream
The recycle stream is the stream that carries the recycled materials back to the
inlet end of the process to be reworked

Selectivity
Selectivity is defined by the following fraction: "moles of desired product produced"
divided by "moles of undesired product". Selectivity enables side-reactions to be
quantified in the material balance.

Sensible Heat
If heat is transferred to or from a fluid and its temperature changes then this
is called sensible heat. The fluid does not change phase during sensible
heat transfer. Sensible heat transfer has the sole effect of changing the fluids
temperature.

Single-Stage
Single-stage applies when a single processing unit is under consideration - this
might be an evaporator, a jam processing unit, a combustion chamber, etc. The
basic point is that there is only one item of process equipment.

Steady-State Operation
Steady-state operation applies where the process is continuous and the flow rate
into the unit is the same as the flow rate leaving the unit. The properties, at any
position, are constant with respect to time.

Stoichiometric Coefficients
Stoichiometric coefficients are the numbers that appear beside each reactant or
product in a properly balanced chemical equation. The stoichiometric coefficients
have no units and act as ratios that show how reactants are consumed and
products are produced in fixed proportions. They do have a sign; reactant
stoichiometric coefficients are negative, while product coefficients are positive.
Any inert species will have a coefficient of zero and should not appear in the
balanced equation.

Theoretical Air
See theoretical oxygen in this glossary. In most cases the combustion, although
between fuel and oxygen, sources the oxygen from air. The theoretical air
requirement is easily found once the theoretical oxygen is known, since air can
usually be taken as 79 mol% nitrogen and 21 mol% oxygen.

Theoretical Oxygen
The theoretical oxygen is the minimum oxygen needed to completely burn all the
fuel available. It is the same as the stoichiometric proportion of oxygen needed.
In practical combustion equipment more oxygen than this theoretical minimum is
actually needed.

Tie-component
A tie-component is a component in a mixture that, through its behaviour, allows
the material balance to be solved by linking together two or more streams.

©H ERIOT-WATT U NIVERSITY
154 GLOSSARY

Transient Operation
Transient operation applies where the process is continuous and the flow rate into
the unit is different from the flow leaving the unit. It also applies to batch units
where the conditions are changing with time. The key difference between steady-
state and transient operation is that the former properties do not change with time,
at any point, whereas in the latter case the properties do change with time at any
point.

Triple Point
The triple point of a substance is when all three phases (solid, liquid and vapour)
coexist together in equilibrium. Using the Gibbs Phase Rule there are no degrees
of freedom. This means that both temperature and pressure are fixed and cannot
change without losing one or more of the three phases. This feature arises
because there are two phase constraints when there are three phases present
and these serve to fix temperature and pressure. For water the triple point occurs
and 0.1◦ C and 0.6115 kN/m 2 .

Unsteady-State Operation
See transient operation - basically the properties change with time at any point in
the process.

Vapour Pressure
The vapour pressure is the pressure exerted by a vapour in equilibrium with its
own pure liquid. The vapour pressure curve is the same as the vaporisation curve,
sometimes called the boiling point curve. For a given chemical species, the vapour
pressure depends only on the temperature. The more volatile a species the higher
will be its vapour pressure and the lower will be its Normal Boiling Point (NBP).

Yield
Yield is another way of being able to move from one side of a reaction to another - it
is defined as the following ratio: "moles of product produced" divided by "moles of
reactant provided". There are several definitions of yield so care should be taken
to check which one is being used at any given time.

©H ERIOT-WATT U NIVERSITY

Potrebbero piacerti anche